You are on page 1of 72

 
History
OBSTETRICS - I understand that you’re here because you wanted to
discuss about home delivery. Are you pregnant at this
PRE-PREGNANCY AND PREGNANCY COUNSELING stage? When was your LMP? How did you confirm
pregnancy? How were your periods before? Did you
Unplanned Pregnancy see any doctor until now? Did you take any folic acid?
Do you have any history of hypertension, epilepsy,
Case: Jenny is 32-years-old and has attended your surgery for diabetes or asthma? Any past history of admissions?
routine checks for the past 3 years. She was last seen 6 months Do you know about your blood group? Were you ever
ago for pap smear which was normal. At the time of the last infected with Rubella? Is this is a planned pregnancy?
consultation, the BP was 130/70 and breast examination was SADMA? Social history? Do you have enough
normal. CVS and respiratory examination were normal. Jenny is support? Financial problems? Do you have other
married and has 2 sons, 10 and 8. kids? How far do you live from the hospital? FHx?

Patient Profile Physical Examination


- Name: Jenny Smith - General appearance
- DOB: 10/04/74 - Vital signs
- No allergy - Neck and breast
- Occupation: Nursing home receptionist - Chest and Lungs
- Family history: Nil - Abdomen
- Medication: Nil - Pelvic
- PMHx: antidepressant given for 2 months at the age - Urine dipstick, BSL and urine PT
of 20 years old; contraception: partner has vasectomy
2004 Management
- I appreciate your concern. Before we discuss options
Task about home delivery, I would recommend for you to
a. History have regular antenatal care which is very important for
b. Physical examination (BMI 24, PT +, urine dipstick you and your baby. As part of the routine, we will start
negative, BP with blood tests: FBE, Iron studies, blood group and
c. Discuss essential issues with patient and Rh, TORCH, HIV, hepatitis B, syphilis, Pap smear if
management due, urine MCS, and BSL. At 18 weeks we will
organize an ultrasound to check the placenta and
History presence of abnormalities of fetus. Around 26-28
- Abdominal pain? SORTSARA? Reflux symptoms? weeks we will organize a sweet drink test for diabetes
N/V? change in bowel movements or urine? Vaginal mellitus and at 36 weeks we will do a swab to detect a
discharge? bug in the vagina. I would like to review you monthly
- Symptoms of depression? Symptoms of STD up to 28 weeks then every 2 weeks from 28 weeks up
(nocturia, pain, weight loss, unexplained fever) to 36 weeks then weekly until delivery.
- You would like to have a home delivery. It is a good
Physical examination idea because you will have your family members and
- General appearance would be more comfortable for you. Usually, there is a
- Vital signs 20-30% more chance of problems encountered during
- ENT: the first pregnancy and labor. During pregnancy, there
- Chest and lungs might be an increased risk of having increased blood
- Cardiac pressure, diabetes, antepartum bleeding, decreased
- Abdomen fetal movements of the baby, and chance of twin
- PV pregnancy. All these things are potentially risky and
- Urine dipstick and BSL can carry bad outcomes. That is the reason we are
doing antenatal care to pick them up early and
Management minimize the risk. Even with normal antenatal course,
- Offering appropriate treatment for nausea there are some unpredictable complications at the
- Medications, rest and fluids time of labor such as fetal distress,
- Explore patient’s attititude towards the situation intrapartum/postpartum hemorrhage, obstructed labor,
- Ensure support is available cord prolapse, shoulder dystocia, meconium
- Offer support aspiration, and such complications need urgent
- Followup management hospital setting with all medical staff and appropriate
- Plans for blood test and STD screening in the future equipments present. If you don’t like hospitals, there
are birth centers or family birthing suites or units which
Home Delivery are small and home-like, but they have midwife and
specialist if required. I would recommend you to have
Case: Your next patient in GP practice is a 24-year-old lady who a safe delivery at the hospital, but at the end, it is your
would like to discuss option of home delivery. choice. If you still want to go for home delivery, I
would advise you to stay near the hospital especially
Task towards the end of pregnancy. You must have
a. Relevant history (LMP 3months ago, confirmed by ambulance cover in case it is required and there
home PT) should be enough support at home. We will do regular
b. Examination findings antenatal care and if there are problems during the
c. Investigation and Management course of your pregnancy, then it is not
recommended.
- Reading materials. Review.

 
Pregnancy Counselling Regarding Timing Of Admission - Arrange for followup with MW and may arrange for
specialist consultation if requested
Case: G1P0 female at 24 weeks AOG asking when to go to
hospital for delivery Pre-pregnancy counseling regarding a patient with epilepsy

Tasks: Case: 26-year-old female presented in your GP who’s known to


a. Focused history be epileptic and is treated by sodium valproate. Over the last 2
b. Answer patient’s questions years, she had not fits and now in your GP clinic, asking for an
c. Counsel accordingly advice for her chances and preparation to be pregnant.

Focused History: Task


- Congratulate patient as it is her first pregnancy a. Counsel patient (include risks)
- Informed consent
- How is the pregnancy? Any problems? History:
- Any previous miscarriages (if yes: details on why, - When were you diagnosed? When was the last fit?
when, AOG) Description of fit (tongue bite, loss of consciousness,
- Is this a planned pregnancy? wetting of clothes, pre-warning signs-aura), any
- Regular antenatal checkups? known triggers (alcohol, excessive effort, drugs?)
- Workups: blood tests? USD? – results? when was the last assessment by her neurologist?
- PMHx: infections (esp TORCH), DM, HPN Any known complications? Any hospital admission?
- BLOOD GROUP All current and previous medications used and if any
- Location: how far do you live from the hospital? In complications? Any previous investigations (CT/EEG
emergency cases, can anyone drive you to the and drug serum level)
hospital? Do you have relatives? Who do you live with - Menstrual history: date of 1st period (menarche)?
at home? Regularity of period? Description of cycle/period (no.
- P/SHx: smoker? Alcoholic beverage drinker? of days of cycle? Days of period) any painful period?
Recreational drug use? Any heavy bleeding or clots?
- Any medications being taken? Allergies? - Sexual history: are you sexually active? In a stable
- Last pap smear? relationship? Any contraception used? Any known
- Gardasil vaccination previous STIs?
- Antenatal history: details of any previous
Counselling: pregnancies? Any previous miscarriages?
- Timing of delivery varies among women. Generally, at - PMHx: any other associated systemic illnesses? DM?
40 weeks, women experience backache, tummy pain, Hypertension?
and passage of mixture of water and blood from - Social hx: family hx? SMADMA? Previous pap smear?
vagina Gardasil vaccination (14-26)? Blood group?
- Labor pains result from strong uterine contractions
similar to period pain and are usually intermittent, Counselling tips:
initially after 20-45 minutes Æ over a period of several - Remember to be positive!
hours grows stronger and lasts longer Æ time to go to - Tell criteria to be eligible for pregnancy
the hospital and MW will measure the time for the pain o For DM: HbA1c <7 for last 3 months
- Sometimes towards the end of pregnancy there are o Epilepsy: free of fit for 2 years
UC that give a feeling of false pain and it is important o SLE: no active disease for the last 6 months
to recognize the pattern of labor pain o DVT/PE: thrombophilia screen negative
- If you develop serious symptoms (bleeding, passing of - Mention fetomaternal risks associated with pregnancy!
blood clots, reduced fetal movements, or trauma) Æ - The management should be by multi-disciplinary
report to the hospital ASAP approach.
- Sometimes PIH can occur during 2nd and 3rd trimester. - Are you alone? Would you like someone to be with
Sx are headache, visual problems, swelling Æ check us?
BP urgently and treat rising BP to prevent any
complications Counselling
- Duration of labor is not predictable because it - Although the outcome is successful for more than
depends on several factors: 90% of epileptic women to be pregnant, there is
o Size of the baby increased risk of fetomaternal risks during pregnancy.
o Position of the baby - For the mother, there is increased risk of vaginal
o Age of the female bleeding especially at the 3rd trimester, relapse of
o Size of maternal pelvis seizures more towards 3rd trimester and during labor.
o Any form of comorbid illness In 3rd trimester, level of absorption of medications is
o Usually: <12 hours for multiparous and 16- reduced hence, there are higher chances of relapse
18 hours for primipara and bleeding.
- Reassurance of support and pain relief throughout - For the baby, there is a risk of cleft lip, NTD, PTL, low
duration of labor by the MW, MDs and nurses birth weight
- Advise on regular antenatal checkups - But, you fit the criteria to be pregnant having no fits for
o Monthly up to 28 weeks the last 2 years.
o Fortnightly from 28-36 weeks - The management should be by multi-disciplinary
o Weekly >36 weeks until delivery approach. I will refer you to a neurologist for review
o Check BSL (OGTT) at 28 weeks AOG, and an obstetrician. I will also arrange for referral to a
vaginal swab to check for GBS at 34 weeks) high risk pregnancy clinic in a tertiary hospital to look
– important to predict a spontaneous and after you. The neurologist will review your medication
normal labor as I don’t think sodium valproate is the best
- Give reading materials and write a script for vitamins medication for you during pregnancy. I believe

 
carbamazepine is less risky. Meanwhile, I will refer We will start you on LMWH on the 14th week of
you to an obstetrician to make sure everything is gestation as a prophylactic measure until 6 weeks
alright and he will follow you up during pregnancy as post delivery.
well. ‐ It’s advised to wear elastic compression stockings
- All antenatal checkup tests will be done before during the day and avoid immobilization
pregnancy. ‐ Labor will be in a controlled manner at 38-39 weeks.
- We will start you with 5mg of folic acid from 1st visit On the planned date, we will withhold the morning
attempting pregnancy (3 months before pregnancy up dose of heparin. After labor, warfarin would be given
to 1st trimester). for 6 weeks (safe in breastfeeding) and we will monitor
- Vitamin K to prevent bleeding especially 26th week INR everyday to begin with (INR 2-3).
onwards ‐ If thrombophilia screen is positive: lifelong warfarin
- Post-delivery mother should nurse baby on the floor
surrounded by cushions. Breastfeeding is okay. Baby Critical issues: failure to do thrombophilia screen; failure to
will not be epileptic. Familial tendency doesn’t advise LMWH during pregnancy; and failure to advise about
increase. warfarin use in pregnancy
- Review of anti-epileptic medications will be done by
neurologist after delivery. Pre-Pregnancy Counseling of Obese Women
- High-risk pregnancy: combined 1st trimester screen
(blood plus usg looking for nuchal translucency and Case: You are a GP and a 30-year-old female came in because
nasal bone); if not high risk: 18-21 and 28-34 weeks she has been trying to conceive for the last 12 months. She
- All should deliver in tertiary hospital and shall have wants your advice on that matter. Height 1.5m, BMI 40, BP
planned labor when they have completed 37 weeks. Normal, BSL Normal

Pre-pegnancy DVT Counseling Task


a. History not more than 3 minutes (periods irregular, 5-6
Case: Your next patient is a 28-year-old woman. Her last weeks pain, stable partners, pap smear 1 year ago,
pregnancy was 18 months ago which was complicated by DVT junk food, no exercise)
and postpartum pulmonary embolism. She has come to see you b. Counsel regarding
for pre-pregnancy counseling. She has stopped warfarin 12 c. Advise Accordingly
months ago. There are no abnormalities on PE. She is not
overweight. Infertility
- >12 mos: investigation
Task - >24 months: infertile
a. Take relevant history (NSVD, episiotomy scar and
baby was normal; did not breastfeed; DVT happened History
postpartum and treated with warfarin x 6 mos) - I can see that you have been trying to conceive. Is
b. Management there anything in particular that concerns you? Do you
think you might be pregnant now? N/V/mood
History changes? Irritability? Breast tenderness?
‐ How was the previous pregnancy? When did the DVT - May I ask if you and your partner are aware of optimal
happen and how was it treated? Have you had any time for sexual activities? What contraceptives were
clotting episodes other than that? Do you have any you using before? How are your periods? Regular?
calf pains? Shortness of breath? Recent long Cycle? Any abdominal pain? Bleeding heavy?
immobilization? Any other bleeding problems (in the - Obstetric history: ever been pregnant? Miscarriages
family)? before?
‐ Contraception? Periods? Are they longer? Do you - I understand you are in a stable relationship. Any
bleed heavily during your periods? Blood group? history of STI in yourself or your partner? PMHx or
‐ Rubella status (vaccinate and avoid pregnancy for 3 Surgical conditions especially gynecologic surgery?
mos)? Thyroid problems? PCOS?
‐ FHx of bleeding disorders? - FHx: infertility? Gyne problems? Recurrent
‐ SADMA? miscarriages? Any pregnancy-related problems (CPD,
difficult delivery)
Management - Have you noticed any recent changes to weight? Hair
‐ Since you had a previous history of clotting during growth, acne? How is your appetite? Water work?
your first pregnancy, you have a high risk of having Bowel habits? How is your sleep?
another one. Pregnancy itself is a hypercoagulable - SADM (pills, steroid, anti-psychotic) A?
state because of the physiological and hormonal - I can see from your notes that your BMI is a bit high.
changes. Your pregnancy will be monitored by a Has it always been like this or is this a new change?
physican and obstetrician and GP. Anybody in family overweight?
‐ Before you get pregnant I would like to do some tests - Have you ever had BP, BSL, lipid level checked?
to exclude a group of disorders that can predispose to What was the result? Have you have ever had joint
clotting. This is known as thrombophilia screening. problem? How do you feel about your weight? How
There are 7 things in this screening: Protein C & S, does your weight affect your life?
factor V Leiden, antithrombin III, anticardiolipin,
antiphospholipid antibodies and anti-lupus Diagnosis
anticoagulant - First of all, it is very good that you have come for
‐ Rubella vaccination if not yet immunized some advice before falling pregnant. Apparently,
‐ Start taking folic acid 0.5mg OD 3 months before everything seems normal except your weight. The BMI
pregnancy and up to first trimester of pregnancy is an indicator of your healthy weight. The normal is
‐ During pregnancy, you would be managed by a team. between 18-24. If >35 it is morbid obesity that puts the
patient at a very high risk of developing obesity-

 
related problems (heart disease, hypertension, stroke, Task
joint problems, DM, stress or depression). a. Counsel the patient (steroids but no longer taking it
- I can see that you are already worried about your because she is symptom-free)
weight. The obesity affects out health generally as b. Answer her questions
well as related to pregnancy especially. Obese
females have higher chance of developing menstrual SLE in Pregnancy
irregularities, problems with ovulation that can ‐ Does not seem to cause exacerbations of SLE
sometimes lead to infertility. According to a study, ‐ Can adversely affect pregnancy according to disease
around 40% of obese females have problems severity
conceiving. Hence, it is very important for you to start ‐ Complications:
losing weight now. o Increased incidence of spontaneous
o Set a goal: 5-10% of BW in 6-12 mos abortions and stillbirth Æ related to lupus
o Make dietary changes – refer to dietitian anticoagulant and anticardiolipin antibodies
o Increase energy expenditure by exercising o Preeclampsia
regularly. I will give you some written o Prematurity
material regarding exercise o IUGR
o Please keep a diary of your diet and weight o Perital mortality
o Come for regular followup ‐ Neonatal lupus syndrome: blood disorders and
- I want you to be aware of certain obesity-related cardiac abnormalities in neonate
complications during your pregnancy, during labor and ‐ Increased maternal morbidity – kidney complications
afterwards. and pre-eclampsia
- During pregnancy, you are at risk of developing: ‐ Management
o GDM o Preconception counseling Æ symptom free
o Pegnancy-induced hypertension for 6 months
o Sleep apnea o Refer for review of drugs
o Problems with baby’s growth and o Corticosteroids
development (IUGR is common). o Low-dose aspirin
- We will check your BSL at 26 weeks and regularly at o Tests: lupus antibodies, APTI, FBE, RFTs,
each visit. You will have regular ultrasound to check ultrasound
growth of baby. Your antenatal visits will be more o LMWH
frequent than other females. At 28-34 weeks, we will o Timed delivery
send you to specialist for anesthetic assessment
because rate of CS is higher in obese females. We Questions:
want to be prepared for that. ‐ Can I become pregnant like other females?
- During labor obese females have higher risk of ‐ What are the risks for my baby?
developing: ‐ How will my SLE be affected by pregnancy?
o Shoulder dystocia ‐ Do I need some special medications during
o Non-progress of labor pregnancy?
o Obstructive labor
o CS and its complication History
o More difficult to monitor HR and activity (fat ‐ When was it diagnosed? What symptoms did you
obstructs signals) have? What treatment was given? For how long? Did
o Pain relief might be more difficult (more you have any side effects from these medications?
adipose, more unequal distribution) How many relapses have you had during the past 5
- What we will do is a planned delivery in a controlled years? Have you had regular checkups with
environment under close monitoring by the specialist specialist? When was your last checkup? When was
obstetrician. A normal vaginal delivery is encouraged the last blood test done? At the moment do you have
as much as possible, however, they will be prepared any symptoms like skin rash, joint pain, problems with
for CS waterworks? Are you on any medications at the
- After labor, there is a higher risk for you to develop: moment? Which one and what dose (prednisolone
o Wound infection 5mg)?
o Clotting problems ‐ When was your LMP? How are your cycles? Are they
o Postnatal depression (more common) regular? How many days of bleeding? How many
- We will give you some meds to prevent clotting. You days apart? Are you on any contraception at the
will be encouraged to breastfeed child that helps you moment? Is this your first pregnancy? Any
to lose weight and to develop good bond with baby. miscarriages before? How’s your general health? Any
Come back after delivery and get wound checked. other medical conditions? Any FHx or SLE or
Please be aware that elective CS is preferred because recurrent miscarriages? When was your last pap
it is hard to do emergency cesarean sections since it smear? What is your blood group? SADMA?
is difficult to move patient. It is more difficult to give
epidural anesthesia to predict effects of medication. Counseling
Please bring your partner next time to discuss further ‐ As you already know, SLE is an auto-immune disease
complications. which means that the body’s defense mechanism
- Reading material becomes active against its own tissues. There is
- Review usually inflammation of different tissues of the body
especially the skin, kidneys, and joints. The exact
Pre-pregnancy counseling of SLE cause is still not known but certain genes and viruses
have been implicated as stimulants. It is very common
Case: You are a GP and your next patient is a 24-year-old in females of childbearing age (20-45).
patient who is a diagnosed case of SLE for 5 years. She wants
to become pregnant and is seeking your advice.

 
‐ SLE unfortunately cannot be cured, but it can be very Any surgical/PM conditions? SADMA? What is your
well controlled with medications to prevent flare-ups. blood group? What is your partner’s blood group?
The good news is that majority of females with SLE Was the previous pregnancy with the same partner as
are able to have kids. It is important that they should now? Did you receive any anti-D injections at that
be symptom-free for at least 6 months before time? Any history of rubella infection before? Were
conception. you tested for rubella? When was your last pap
‐ There are certain risks associated with SLE: smear? What was the result? Are you vaccinated with
o 40% have exacerbations/flare-ups however gardasil?
10% have remissions
o Maternal risks: 20% develop pre-eclampsia, Management
2nd or 3rd (25%) miscarriages, ‐ From the history the only problem that I noticed is that
o Fetal risks: IUGR, prematurity (50%) you have a blood group that might carry some
o Lupus-like syndrome at time of delivery (5%) problems for you and your baby in the future. Let me
Æ rash and abnormalities of blood cell explain to you about blood groups. Usually in our
counts. This lupus like syndrome is not SLE. blood, there are blood cells that carry oxygen to the
This is a temporary response in the baby body. These cells carry proteins in the surface which
because of transplacental transmission of are named as A, B, O, AB as well as another factor
antiphospholipid antibodies from the mom to known as Rhesus factor (+ or -). The blood type is
the baby. It usually resolves within the 1st 4 determined depending upon the presence or absence
weeks; of these proteins. Around 85% of the population is
o congenital heart blocks: quite rare; only 2% positive for rhesus factor. The rest are negative. This
of pregnancies are complicated by this is important if your partner is carrying it in his blood.
‐ SLE: small-vessel vasculitis which also deposits in the There is a 50% chance that your baby will be Rh+.
placenta and small clots within the placenta Æ IUGR, Sometimes, the baby’s blood cells cross the placenta
prematurity, death either during pregnancy, miscarriage, with trauma, or
‐ We will consider this pregnancy to be a high-risk even without any cause. In that case, the mother’s
pregnancy. You will be managed by the specialist immune system produces antibodies against the
throughout the pregnancy. They will decide upon the baby’s cells. This phenomenon is known as
best medications for you during pregnancy. Usually, isoimmunization. If the mother does not receive any
steroids are safe but dose of steroids will be anti-D injections and she becomes pregnant again,
managed. Sometimes, azathioprine may be used. All there is a very high chance that these circulating
other cytotoxic drugs as we know are contraindicated. antibodies reach the baby causing: hydrops fetalis,
‐ We will do some blood tests and ultrasounds before hemolytic disease of the newborn, neonatal hemolytic
pregnancy and continue close monitoring throughout anemia. This results from breakdown of the baby’s
your pregnancy. To prevent the risk of clotting blood cells. The end result of the blood cell
problems or thrombophilia, the specialist might start metabolism is bilirubin which can be checked within
you on ASA or LMWH that you will need to continue the amniotic fluid to check the degree of hemolysis. At
after delivery (especially if anticardiolipin is positive). the moment, what we can do is to do regular antenatal
‐ The mode of delivery and timing will be best decided tests including your blood group and your partner’s
by the specialist according to the baby’s condition. If blood group.
they have any problems with his growth, they might ‐ You need to start taking folic acid 0.5mg OD from now
intervene earlier. onward. Once you become pregnant, at around 20
‐ I am going to write some blood tests for you: FBE, weeks of gestation, we will do a test that is called
UEC, Blood group, rubella antibody status, amniocentesis to check the level of bilirubin. If
anticardiolipin antibody, complete thrombophilia required, we will give you Rhogam or anti-D
screen. immunoglobulin, an injection to neutralize the
‐ Refer to obstetrician. antibodies. We will also test your blood for the level of
‐ Reading material antibodies to Rh group and titer. If titer goes beyond
‐ SLE association of Australia 1:8, we will do amniocentesis earlier, further followup
testing and anti-D injections.
RH-isoimmunization Counseling o Kleihauer test: determine how much
Rhogam is required. Tries to find out how
Case: You are a GP and a 25-year-old female comes to your many fetal RBCs are present within the
clinic. She had a miscarriage 2 years ago and she wants to mother’s blood.
become pregnant again. o Coombs test/antiglobulin test: done to check
the level of antibody in mother’s blood.
Task ƒ Direct (checks the antibodies that
a. History are bound to RBC)
b. Relevant management ƒ Indirect (check the circulating free
antibodies)
History ‐ Recommendations: For all RH (-) whose pregnancy
‐ When did you have it? What was the gestational age progresses to 28 and 34 weeks and postpartum within
of the pregnancy? Why was it terminated? What 72 hours will be given 625 Rhogam injections
method was used? Where was the termination done? irrespective of antibody titers.
Any complications afterwards? Any blood transfusions ‐ If bilirubin too elevated: exchange transfusion
or further procedures were required? Have you been ‐ Refer to obstetrician for possible assessment.
pregnant again since then? How are your periods? ‐ Reading materials regarding isoimmunization.
Are the cycles regular? Any bleeding in between? I ‐ Review
understand you’re in a relationship, what
contraception are using? Any history of STI in yourself
or partner? Any other medical or surgical conditions?

 
RH ISOIMMUNIZATION INDICATIONS was your last pregnancy? How did you miscarry? Any
All Rh(-) and unsensitized who requires or with: trauma? Did you have a D&C during any of the
INDICATIONS DOSE pregnancies? Did they do an autopsy on the products
Abortion or requires D&C (give within 72 hours to 250 IU IM of conception? During the last 3 pregnancies, did you
9-10 days)
suffer from any infections? Fever? Did you have the
CVS/amniocentesis 250 IU IM
antenatal blood tests done?
Threatened abortion <20 weeks: 250 IU
Antepartum hemorrhage IM ‐ How is your general health? Any history of diabetes,
Abdominal trauma >20 weeks: 625 IU thyroid problems, immune-related diseases like SLE?
External podalic version IM History of gynecological surgery? Blood group? Last
Bleeding during pregnancy pap smear? Were you on any contraceptives before
@ 1st trimester single 250 IU IM this pregnancy?
@ 1st trimester multiple 625 IU IM ‐ SADMA?
@2nd/3rd trimester 625 IU IM
@ Postpartum 625 IU IM Investigations
Pregnant women at 28 weeks 625 IU IM ‐ We need to do a pregnancy test on you to confirm if
34 weeks 625 IU IM
you are pregnant. If it positive, I will refer you to the
Rh (+) baby (give within 72 hours of delivery) 625 IU IM
high-risk pregnancy clinic. If negative, I will refer you
MISCARRIAGE AND ABORTION to a specialist clinic called recurrent miscarriage clinic
where they will do some tests on you to find out the
Recurrent Miscarriages possible cause of the miscarriages. They might ask
your partner to come in for a checkup as well.
Case: You are a GP and a young 26-year-old lady presents to ‐ I would ask the examiner for the results of the blood
you in your GP clinic. She has had 3 miscarriages before. She tests including FBE, Blood group, Ultrasound to check
thinks she is pregnant again because she has not had her any defects of the uterus, ovaries, and fallopian tubes.
periods for the last 6 weeks. She has a family history of alpha I would like to run a complete thrombophilia screening
thalassemia. (Protein C, S, antithrombin III, anticardiolipin antibody,
factor V leiden – most common deficiency, blood
Task homocystein levels), TORCH, Thyroid function tests,
a. Counsel the patient BSL, urea and electrolytes. At the clinic they will order
HLA and karyotyping for both partners.
History (miscarriages x 3 episodes around 8-10 weeks, had ‐ If PT (+): I will refer you to the high-risk clinic where
curettage once, irregular period 4-5weeks, Blood group B+) you will be seen by the specialist obstetrician.
Recurrent miscarriages affect 1% of all couples.
Case 2: You are a GP and a young 28-year-old lady presents to Sometimes, even with extensive investigations, no
you in your GP clinic. She has had 3 miscarriages before at cause can be found. You still have a very high chance
around 8-10 weeks and has had D&C done. You did some of a normal pregnancy. After the 1st miscarriages,
laboratory tests and she has come to collect the results. chances of successful pregnancy is 80%, 2nd (75%),
3rd (70%). I will ask the psychologist, midwife, and
Investigation: FBE, TORCH, chromosomal analysis, APAS, obstetrician to support you all this time whether or not
TFTs, PRL, LFTs, Hepatitis B&C, Urine microscopy and culture, you are pregnant.
FBS, HIV and STDs, thrombophilia, USD of uterus. ‐ One of my friend got cervical stitch, should I have it
too? It is usually done in cervical incompetence where
Causes: the miscarriage occurs in the 2nd trimester. We can do
‐ Immune-mediated: APAS, SLE, HLA incompatibility ultrasound earlier this time.
between partners, thrombophilias, SLE ‐ Referral letter
‐ Uterine abnormalities: cervical incompetence (2nd ‐ Written material
trimester), gynecological surgeries, birth defects
(septate uterus) Threatened Abortion
‐ Infections: TORCH and STDs, Hepatitis B&C
‐ Endocrine: DM and thyroid Case: You are a GP and a 28-year-old female comes to you
‐ Maternal ageÆ not a cause but risk factor; females with vaginal bleeding after 8 weeks of amenorrhea.
who become pregnant after 40 years has 50% chance
of miscarriage within the 1st trimester Task
a. History (2pads, clots, regular periods, B+)
Definition b. Physical examination (moderate bleeding, clot, os is
‐ >3 consecutive pregnancies lost by a female closed, uterus is normal and not enlarged, (+) CMT)
c. Investigation
History d. Management
‐ I can see from the notes that you have a history of
recurrent or repeated miscarriages. At the moment, Case: You are a GP in a suburban GP practice. Your next
you think that you might be pregnant. Have you done patient is a 24-year-old Mrs. Jones with heavy PV bleeding for
a test to check for pregnancy? Do you have any the last 24 hours. She is 7 weeks pregnant by date and she is
concerned and seeks your care.
symptoms like morning sickness, breast tenderness,
or irritability? I understand your LMP was 6 weeks
ago, any bleeding since then? Tummy pain or Task
discharge from down below? a. Focused history
b. Physical examination
‐ How are your cycles? How many days of bleeding?
c. Investigation
How many days apart? Please tell me more about
d. Diagnosis, management, and differential diagnosis
your previous pregnancies? Have you had any kids up
to now with this partner or previous partners? When

 
Differential Diagnosis Incomplete Abortion
- Ectopic Pregnancy: PV bleeding + b-hCG(+)+ os
closed + empty uterus Case: You are an HMO in ED and a 39-years-old female comes
- Threatened miscarriage: PV bleeding + b-hCG (+) + in complaining of vaginal bleeding and abdominal pain. LMP
os closed + intrauterine pregnancy was 8 weeks ago.
- Incomplete abortion + b-hCG(+) + os open +
intrauterine pregnancy + POC on examination Task
a. History (lower tummy, comes and go, started 12 hours
History ago; 4-5 pads/day; periods every 28-30 days, no easy
- Is my patient hemodynamically stable? bruisability or bleeding disorders)
- Please tell me more about the bleeding? When did it b. Physical examination (distress, pale and in pain; BP
start? How many pads did you use up to now? Did 80/50, os open with POC, PR:80 Æ vasovagal shock;
you pass any clots? Do you have any associated size of uterus is 8 weeks, mobile, no adnexal
pain? Have you felt N/V/breast tenderness? Do you masses/tenderness; no CMT)
feel dizzy at the moment? Any fever or discharge from c. Diagnosis and management
down below? Possibility you might be pregnant right
now? When was your LMP? Are you periods regular? History
How many days of bleeding? How many days apart? ‐ Is my patient hemodynamically unstable?
Have you ever had spotting in between? I understand ‐ When did the bleeding start? What is the color of the
you are sexually active and in a relationship, what bleeding? How many pads did you used since then?
method of contraception do you use? Are you Were they fully soaked? Did you pass any clots or
planning to fall pregnant? Have you ever been pieces of tissue? Did you bubbles or grape-like
pregnant before? Any miscarriages? When was your tissues? Do you have any dizziness, SOB or fever? Is
last pap smear? What was the result? What is your it the first time?
blood group? Any past medical or surgical condition ‐ Where is the pain? Is it there all the time or does it
especially any bleeding disorders, thyroid problems, come and go? Does it go anywhere? How severe is
gynecological conditions. FHx of bleeding disorders. the pain from 1-10? Anything that makes the pain
Have you or your partner ever been diagnosed with an better or worse? Any trauma or intercourse before the
STI? Any problems with waterworks like burning, bleeding?
frequency? How are your bowel habits? SADMA? ‐ Are your periods regular? When was your LMP? How
many days of bleeding? How many days apart? Do
Physical examination you have heavy periods?
- General appearance ‐ Are you sexually active? Are you in a stable
- Vital signs (postural drop) relationship? Any contraception used? Have you or
- Abdominal examination: distention, tenderness your partner ever been diagnosed with STDs? Any
especially on the RIF and LIF. Any visceromegaly, chance you could be pregnant? Do you know your
bowel sounds blood group? Have you ever been pregnant before?
- Pelvic examination: amount of bleeding, color of Any miscarriages? Do you have N/V/ or breast
blood, clots, discharge or signs of trauma? Sterile tenderness recently? When was your last pap smear?
speculum, check os whether open or close; POC; any ‐ How’s your general health? Do you have any FHx of
mass or lesion over the cervix; bimanual examination bleeding/clotting problems or miscarriages?
checking for size, shape and position of uterus;
adnexal tenderness or mass; cervical excitation; Physical Examination
- Urine dipstick, BSL, pregnancy test ‐ General appearance
‐ Vital signs
Diagnosis and Management ‐ Abdomen
- If pregnancy test positive: most likely your condition is ‐ Pelvic Æ remove POC immediately!!!
called threatened abortion/miscarriage. Your ‐ Urine dipstick
pregnancy test is positive, but because of your
bleeding, we need to admit you to the hospital to do Diagnosis and Management
some tests which include FBE, U&E, blood group, ‐ Admit the patient
USD of the pelvis to look for the presence of a fetal ‐ Start IV fluids and take blood for grouping and
sac within the uterus and to check for cardiac activity. crossmatching
Depending upon the results, the OB might advise you ‐ Give oxytocin or ergometrine or (Syntometrin) to stop
to take rest. Sometimes, because of the attachment of bleeding
the placenta to the womb, some bleeding can happen. ‐ Refer to OB&Gyne registrar for curettage
In majority of cases (90-95%), this bleeding is quite ‐ From history and examination, I am sorry to say that
harmless. It will stop on its own within a few days. this is a miscarriage. Most of the miscarriages occur
Your pregnancy will continue without any problems, without any obvious reason. Let me reassure you that
but you need to avoid stress, anxiety, and rigorous it is not your fault. You did not do anything wrong. So
physical activity for the rest of your pregnancy. We do please do not feel any sense of guilt. Most likely in the
not need to give you any medications as it has not first 14 weeks, the reason of miscarriage is due to
shown to alter the outcome in any way. If the bleeding chromosomal abnormalities. I have admitted you,
continues, we will repeat serial ultrasound to check for informed the registrar, and sent all the bloods for
fetal viability, but you will need to stay in the hospital necessary investigations. They will probably take you
until the bleeding stops. to the theater and do a procedure called curettage.
- If pregnancy test negative: Most likely, this is a They will empty whatever is left in the uterus to
delayed period. Sometimes, due to stress and with the prevent any complications. We will wait for your blood
use of the pill, your periods can become irregular. If it group report to come and if it is negative, we will give
continues for the next 2 or 3 cycles, you will need to you an injection called anti-D.
see the specialist gynecologist. She might decide to
start you on regular OCPs to regulate the cycle.

 
‐ Can I still get pregnant? Yes, you can still get which usually rises every 48 hours. If it doesn't, then
pregnant but it is advisable to wait for at least one we will do USG, progesterone (low) and CA-125 (rises
normal period before you get pregnant again. during impending rupture)
‐ I know it is a very hard time for you. Do you want me ‐ The gold standard for diagnosis remains to be
to call anyone for you? Do you have enough support? laparoscopy.
‐ Being 38 years old puts you at a higher risk of your ‐ If we find that the next one is ectopic as well,
child having Down syndrome. So in your next depending upon the fetal viability and damage to the
pregnancy, it is advisable for you to consider doing tube, the specialist obstetrician might decide to inject
Down Syndrome screening. MTX within the gestational sac that will help in
resorption of the fetus protecting the tube.
Critical error: ‐ If you develop tummy pain, vaginal bleeding, episodes
‐ Not considering anti-D of fainting or dizziness, or back pain (interscapular
‐ Not taking out POC immediately area), please come to the hospital right away because
‐ Doing unnecessary investigations like beta-hCG and these are symptoms of early ectopic pregnancy.
USD ‐ The best option would be IVF if your opposite tube is
removed. Please be optimistic. You still have a very
EXTRAUTERINE AND ECTOPIC GESTATION high chance of having a normal pregnancy.
‐ General risk for ectopic pregnancy: 1%; chance of
Ectopic Pregnancy recurrence: 10-20%

Case: A 23 years old female has recently been discharged from ANTEPARTUM AND OTHER COMPLICATIONS IN
the hospital after a procedure where the right Fallopian tube was PREGNANCY
removed because of an ectopic pregnancy. The left ovary on the
ultrasound showed the presence of corpus luteum. The patient Antenatal Care:
wants to know why it happened to her. - Do beta-hCg (quantitative or qualitative)
- Down Syndrome risk:
Task o @37: 1:200
a. Talk to the patient and explain about ectopic o @40: 1:100
pregnancy and its causes. o @45: 1:50
- Screening for down syndrome: HR: 1:200 or higher
‐ From the notes, I can see that you have recently o 1st tri: 80% predicted
undergone a procedure to remove a right ectopic ƒ 10-12 weeks: PAP-A and beta-
pregnancy. How are you feeling at the moment? How hCg;
are you coping with the loss of this pregnancy? ƒ 12-13 weeks: USG (nuchal
‐ I understand why you want to know why it happened translucency Æ aneuploidy)
to you. Do you know what ectopic pregnancy means? o 2nd tri: 60-70% predicted
Usually, the egg from the mom and the sperm from ƒ QUAD screen @14-20 weeks:
the dad meet within the tubes to form the fetus. This AFP, b-Hcg, estriol, inhibin A
fetus then travels and becomes attached to the wall of (ACEI)
the womb. Due to certain reasons, sometimes, the - May do dating usg during first visit
fetus implants within the tubes. It is then called an - Amniocentesis (0.5%)/CVS (1%): risk of miscarriage
ectopic or extra-uterine pregnancy. The size of the - Blood group
tube does not allow the fetus to grow therefore it may o If (-): repeat blood at 28 weeks; then give
rupture and leads to a lot of bleeding and other anti-D; repeat blood antibody screen at 34
complications. For you fortunately, such complications weeks (2nd injection of anti-D) Æ prevent
were prevented and the tube was removed. Please spontaneous transplacental hemorrhage
don't worry. You still have a chance of normal ƒ 2nd tri: 12-15% fetal RBCs can be
pregnancy. The risk factors for ectopic pregnancies found in maternal blood resulting
are: previous history of PID and STI (increases risk in isoimmunization
7x), previous surgeries of gynecologic nature ƒ 3rd tri: 20-30%
especially around the tubes, history of endometriosis, o Give anti-D after delivery
IUCD use, use of emergency contraception (causes - FBE: consider anemia (r/o hemoglobinopathy)
retrograde contraction of the Tubes), embryonal o Check the partner and check for trait
defects, previous history of ectopic pregnancy in the - Screen for infections: Rubella, HIV, Hepatitis,
opposite tube. hepatitis B&C, syphilis
‐ In most of the cases (97%), ectopic pregnancies are o If HbsAg (+)Æ check partner for hepatitis b
found within the tubes. Sometimes, they can be found antibody; talk about safe sexual practice
in the ovary, peritoneal cavity, and on top of the uterus o For hepatits b&c Æ refer to infectious
‐ For your next pregnancy, the chances of conception specialist
are around 50%. Please remember that even one tube - MSU for micro&culture: asymptomatic bacteriuria (or
can catch the eggs from the opposite ovary. You need in 6-8%) Æ(+) if >100,000 col/ml; tx because
to wait for at least 3-6 months before trying to increased likelihood of getting severe UTI (e.g.
conceive. Give yourself some rest and have a healthy pyelonephritis)
balanced diet. You can use OCPs but please avoid - Vitamin D levels: N: 70u; severe <20u
IUCDs, Emergency pill and POPs. - 18-22 weeks: morphology scan to check for structural
‐ When you miss your next period, please come and abnormalities
see me ASAP. We will do some tests including serial - 28 weeks: check for anemia (FBE) – physiologic
beta-hcg done starting day 5 of conception. We would anemia and GCT
like to record the quantitative increase in beta-hcg

 
- Average gestation: 40 weeks + 2 weeks; >42-43 ‐ Period questions: Do you have regular cycles? How
weeks perinatal mortality doubles; long is the cycle? How long is the bleeding time? Any
o Concern at 41-42 weeks: do fetal well-being spotting in between? Do you have excessive pain or
USD measuring umbilical artery flow (SD bleeding during the period? How did you confirm your
ratio: difference between peak systolic flow pregnancy (I did pregnancy test at home)? Good on
and end-diastolic flow), AFI and CTG you!
‐ Pregnancy symptoms: Do you feel tired, nausea?
Antenatal checkup Have you vomited? Breast tenderness? Tummy pain?
How’s your water work? Do you have regular bowel
Case: Your next patient in your GP practice is a 24-year-old function? Do you have unusual vaginal discharge or
female who is 8 weeks pregnant. You saw her last week as a bleeding?
part of her regular antenatal checks and ordered some blood ‐ What type of contraception did you use before you got
tests. Today she is here to know about the blood results. Her pregnant? Have you been diagnosed with STD?
health and pregnancy have been good so far. She is so excited When was your last PAP smear (If no for last 2years
about having a healthy baby by the end of her pregnancy. Her do it now!)? Do you know your blood group? Have you
results are as follows: had Rubella in the past or have you receive vaccine
FBE: Hgb 120, WBC 8000, Plt 170,000 for it?
UEC: Na 145, K 4.4, Cl 130 ‐ Any serious illnesses or surgeries in the past? (Heart,
LFTs: normal HTN, DM, anemia.) Is your husband generally
BSL: 4.3 healthy? Are you on any medication? Are you taking
Blood group: A-; Antibody screening test (-) folic acid? Are you allergic to anything? Smoking,
IgG (+) for Rubella and Varicella Alcohol and drugs? How many cups of coffee do you
Urine: MCS show GBS positive drink per day? What do you do for a living? When did
HBV and HCV: negative you migrate to Australia? Do you have any family
members or close friends here? Has anyone in the
Task family had twin pregnancies? Has anyone in the family
a. Explain result and advise on management had pregnancy complicated by DM, HTN, birth
defects?
Management
- Congratulate on her pregnancy Management
- Give anti-D at 28, 34 weeks and 72 hours after ‐ We need to order some routine lab tests to identify
delivery if child is Rh (+) and if there are bleeding any issue which needs to be addressed for the best
episodes outcome of your pregnancy.
- If antibody screening test positive: measure the titers o FBE exclude anemia. Hb. Iron deficiency
using ELISA (1:8 or 1:16 or 1:32 then check bilirubin ÆSupplement.
by doing o Blood group and RBC antibodies. If you are
o Amniocentesis: check bilirubin; Rh-you need anti-D immunoglobin
o Umbilical cord sampling: Hct (25%) prophylactically to prevent problem in future
o MCA ultrasound: check velocity of blood pregnancy. Repeat antibody test in
flow -- if there is hemolysis heart pumps 26weeks.
faster then velocity increases; less invasive o Rubella status if you are not immunized to
- Urine MCS: positive for GBS (asymptomatic rubella, I recommend you receive rubella
bacteriuria) -- treat with antibiotics because of risk of vaccination after delivery. (Contraindication
developing pyelonephritis during the pregnancy)
(Cefalexin/Augmentin/amoxicillin) o We will also do syphilis, Hepatitis B and C
‐ Repeat culture after 1 week and HIV screening.
‐ General advise for UTI o Vitamin D level.
‐ Check partner’s blood group o Midstream urine to check urinary tract
‐ Advise on antenatal checkup infection. Sometimes it can be
‐ Dietary advice, smoking and alcohol asymptomatic but need to be treated in
‐ Down syndrome screening Æ if older patient pregnancy. 30% of asymptomatic UTI can
become symptomatic.
First Antenatal Check Up o There’s another test which we offer in every
women in Australia. It’s a Down’s syndrome
Case: Mrs. Hasim a migrant from Sudan presents to your GP screening test. Would you like to do it?
clinic for her fist antenatal visit. ƒ 1st trimester: Pappa, beta HCG,
Ultrasound
Task ƒ 2nd trimester quad. Test(15-
a. Take History 18weeks): beta HCG, AFP,
b. Your management in pregnancy oestradiol, inhibin A
o You also need 18-20weeks mid pregnancy
She is a professional boxer for 10 years. “Can I do exercises?” ultrasound to make sure baby develops
“Can I eat sushi?” “How about weight gain?” properly and to look for position of the
placenta.
History o At 28weeks we screen for Gestational
‐ Mrs Hasim, Do you need interpreter? I understand you Diabetics: sweet drink test/glucose
have come to see me regarding pregnancy. Is it your challenge test.
first pregnancy (Yes)? Was it planned (Yes)? o At 36 weeks you will need to be advised to
Congratulations! do a low vaginal swab to check for a
‐ When was your LMP (8 weeks ago)? bacterial infection called GBS. If found you
will be given antibiotics prophylactically
during delivery.
10 
 
‐ You need to take folic acid 0.5mg for the 1st 3 months Counseling
of pregnancy because it decreases the occurrence of - The first pregnancy is usually longer as compared to
neural tube defects. succeeding ones. However, there are some warning
‐ Moderate exercise is good for you because it signs: if you have any contractions,any passage of
improves cardiovascular and muscle strength. Best mucus or water, vagina bleeding, any reduction in
exercises are low impact aerobics, swimming, walking fetal movements, any sort of tummy pain, headache,
and yoga. No contact sport because of risk of trauma. blurry vision, cloudy urine, or other warning signs, you
‐ Weight gain should be around 11-16kg during have to come to the hospital straight away.
pregnancy. But it all depends on your pre-pregnancy - The plan for your pregnancy is to come every month
state. Your diet is important, it should be well until your28th week, then every fortnightly from 28-36
balanced. Food rich in protein, dairy food, starch food weeks and weekly after 36 weeks and until delivery.
(potatoes) and plenty of fruits and vegetables. Best At 28 weeks, we will arrange a sweet drink test and
avoid a lot of sugary, salty and fatty food. Food around 34-46 weeks, we will do the vaginal swab to
delicacies: uncooked meat, egg, soft cheese, shell fish detect the bug called GBS.
and raw fish should be avoided as they are potential - If there are no warning signs as discussed before, it is
sources of Listeria and Salmonella. advisable to either relocate close to the hospital if you
‐ No smoking, alcohol and drugs. have friends or relatives or get admitted to the hospital
‐ What about my sexual life? Sexual life is acceptable a week or so before the due date.
and normal during pregnancy just follow your normal - Will I have a long labor this time as well? With regards
desire. to your delivery, the exact duration of your labor is not
‐ Can I see a dentist? See your dentist in case any easy to predict as it depends on several factors at the
dental care is required and it can be carried out in the time of delivery such as medical conditions, size of the
first half of the pregnancy. baby, size of the pelvis, presentation of the baby, and
strength of the contractions. But usually, the duration
Timing of Admission to Hospital of labor in 2nd pregnancy is shorter compared to the
1st.
Case: your next patient in GP practice is a 24-weeks pregnant - Right now everything sounds good. I will see you in
lady who has just moved into your town. She has come to see one month time and give you a few reading materials.
you as her first GP. She lives 80km from the main hospital
Down Syndrome Screening
Task
a. Relevant history (folic acid, regular checkup, normal Case: A young woman at 10 weeks’ gestational age comes to
USD and blood tests; history of prolonged labor see you in your GP practice. She is concerned about having a
because of poor contractions; instrumental delivery) baby with Down syndrome as recently, her sister had a baby
b. Advise when she immediately needs to attend the with Down syndrome.
hospital or midwife
c. Answer her question Task
a. Counsel patient
History
- Congratulations on your pregnancy. I can see that - Is this a planned pregnancy? Congratulations.
you’re concerned about when you should go to the - I understand from the notes that you are here to
hospital for delivery. I understand that you live 80km discuss about Down syndrome screening. I appreciate
away from the hospital. Before I address your your initiative to do that. I understand your anxiety. I
concern, is it okay if I ask you some questions? will give you all the information regarding the tests
- How is your pregnancy going so far? Was it a planned which can be done and how effective they are.
pregnancy? Are you attending regular antenatal care? - How is your pregnancy going so far? Are you getting
How were the blood test results? Anything significant? your antenatal care? Are you done with your blood
Do you know your blood group? What about the 18th tests? Any concerns or issues?
week USD? Is it a single baby? Is the placenta in the - Down syndrome is one of the common genetic
normal position? Any tummy pains or trauma so far? abnormality with trisomy 21. There are some
Any discharge or bleeding so far? Any leakage of fluid indications in doing Down syndrome screening in
down below? Any headache, BOV, N/V? Any urgency, pregnant women:
frequency or smelly urine? Did you take folic acid? Is o Increased maternal age (>30)
your baby kicking well? Any previous pregnancy or o Previous down syndrome baby
miscarriage? How was it? Was it term or preterm? Do o History of down syndrome in the family
you know the reason for the prolonged labor? How - We have screening tests and confirmatory tests. In the
was the baby after delivery? Any complications? What first trimester, there is a triple test a blood test which is
was the BW? Any previous medical or surgical issues done at 9-13 weeks AOG. We check free beta-hCG
like BP, DM? Any problem with your periods? Are you Pregnancy Associated Placental Protein-A. We
on any medications? SAD combine it with Ultrasound and it is done at 11-13
- Do you have enough support? I understand you live weeks AOG. Here we check for fetal nuchal
80km away, how long does it take to go to the nearest translucency. Screening tests can also be offered in
hospital by car? Can anyone drive you to the hospital the 2nd trimester between 15 and 17 weeks. These
in case of emergency? Do you have any friend or tests are not 100% confirmatory. In high-risk
relatives who live near the hospital where you could pregnancies, we can offer diagnostic tests: CVS or
relocate a few days before the due date? Is there amniocentesis.
anyone at home who will look after your first baby
when you are in the hospital?
11 
 
- CVS - Mechanical cervical ripening device Æmay be used
o done ideally at 9-11 (11-12 at clinical book) safely
weeks - 1/5 of patients end up having cesarean section
o results within 24 hours
o more accurate History
o 1% risk of abortion - Congratulations. Is it a planned pregnancy?
- Amniocentesis - What about first pregnancy? Was it your first? Was it a
o Done ideally at around 14-15 planned pregnancy? Did you have regular antenatal
o Longer (up to 3 weeks) and less accurate checkups? Complications of pregnancy (DM,
o 0.5% risk of abortion hypertension, bleeding)? Why was the CS performed?
- 3 regimens: Was it an emergency? Do you know the type of
o PAPPA and free hCG at 9-13 weeks cesarean section? Complications of surgery
o Nuchal thickness at 11-13 weeks (combined (infections, bleeding, DVT)? CPD (height of partner
tests raises detection rate from 70 to 90%) and patient)? How was the baby at birth? Any
o If calculated to be more than 1/200-250 Æ resuscitation needed?
woman is offered CVS if gestation between - History of previous uterine surgeries or rupture?
11 and 14 weeks or amniocentesis if at 15- - Are you taking folic acid?
16 weeks - How is your general health? Any medical condition
o Combined test: AFP, unconjugated estriol you have at this moment? Why do you want to have
and beta-hCG + Inhibin A at 15-20 weeks vaginal birth?
Æ increases detection rate from 65 to 75-
80% if inhibin A included Findings from Examiner
- If previous pregnancy was down syndrome, the risk of - Reason for cesarean section
having Down syndrome in the next pregnancy - Classical or Low-segment cesarean section
increases by 1%. - Age of gestation
- Complications: anesthetic, infection, hemorrhage,
Vaginal Birth After Cesarean Section (VBAC) damage to the adjacent organs like bladder, large
intestine etc, DVT
Case: You are a GP and a 28-years-old lady with previous - Baby: weight, apgar score, resuscitation done
cesarean section 2 years ago is in your GP clinic. She is now 7
weeks pregnant and she wants to have vaginal birth. Management
- At this stage we are not sure about the outcome of the
Task pregnancy as it depends on its progress. However, in
a. History (CS due to fetal distress, pap smear x 1 year majority of cases and in your case, successful vaginal
ago with birth can be achieved safely. The success rate ranges
b. Ask examiner for previous medical/surgical notes of from 55-85%. I will do antenatal screening tests and
the LSTCS (obstructed 2nd stage of labor hence will monitor you during your antenatal visits to look for
underwent CS, Apgar 6,8 BW 3kg, no CPD) certain conditions which can pose a risk during vaginal
c. Discuss possibility of vaginal birth to patient delivery or which can be an indication for cesarean
section. If any of these are present, you will be
Predictors of successful VBAC (55-85%): managed as a high-risk pregnancy.
- Non-recurring indication of CS (e.g. malpresentation) - I will arrange an appointment with an obstetrician at
- PIH 26 weeks for discussion about possible mode of
- Previous vaginal birth delivery and at 36 weeks for definite decision
- Institutions in which success rates is high regarding vaginal birth. The specialist will explain the
- Onset of labor is spontaneous risks and benefits of the mode of delivery to you and
the final choice will be made according to your wishes
Contraindication and advice of the obstetrician. If vaginal birth is
- Previous classic cesarean section birth decided, it will take place in a well-equipped hospital
- Some uterine surgery (hysterotomy, deep under supervision of an experienced obstetrician
myomectomy, corneal resection and metroplasty) because vaginal delivery can progress to cesarean
- Previous uterine rupture or dehiscence section in 1/5 of the cases.
- Maternal or fetal reason for elective CS in current - Folic acid prescription
pregnancy - Reading material
o Mother: PIH, Diabetes, Antepartum - Review
hemorrhage (previa/abruptio)
o Baby: Macrosomia, Multifetal gestation, Ovarian Cyst in Pregnancy
Malpresentation (breech, face, brow,
transverse lie) Case: You are HMO in ED. 25yo female 8weeks pregnant c/o
o Labor: Power, Passage, Passenger pain in the right lower abdominal pain.

Eligibility: 1 previous LSTCS and NO contraindication Task


a. Take history
Induction of labor: b. Ask for Physical Findings (All vitals stable. Healthy
- Risk of uterine rupture especially if induction of labor looking. Abdominal examination: Tender in the right
with prostaglandin E2, oxytocin + amniotomy and iliac fossa. No organomegaly. Per speculum: no
misoprostol is used discharge, no bleeding, no poc, os is closes.)
o Classic (5%)
o LSTCS (0.5%)
12 
 
c. Ask for one relevant investigation findings ((U/S: changes during pregnancy can sometimes be
Intrauterine pregnancy, Cyst in the right ovary 5cm in responsible. Rarely certain types of nasty growth may
size, no fluid in the pouch of Douglas) develop within that cyst however the chances are very
d. Talk about relevant management low at your age. The management depends upon the
size of the cyst, your symptoms, and the opinions of
Differential Diagnosis the obstetrician
- UTI - According to JM
- Ectopic pregnancy o If it’s a simple cyst <5cm reassess the
patient clinically and with a U/S in about 6
History weeks time.
o If it’s a simple cyst >5cm recommend a u/s
- Is my patient haemodynamically stable? guided aspiration.
- Pain questions: How bad? 5-6/10 dull kind of pain not o Complex cysts irrespectively to size,
radiated. Where? Go anywhere else? Does anything excision laparoscopically
make it better or worse? When did it develop? Is this o Any symptoms or U/S evidence of torsion of
the first time? Any associated symptoms eg fever, cyst: laparotomy and removal of cyst
N/V, bleeding from down below, discharge from down - For your case, because your cyst is still around 5cm
below? and your symptoms are controllable (pain killers
- Problems with water work: burning or frequency? given). I’ll ask obstetrician to come to see you. Most
Bowel habits: history of constipation? likely they will advice careful monitoring to lookout for
- I understand from notes you are 8 weeks pregnant. any symptoms of torsion which are: severe pain all
When was it confirmed? At the moment do you have over the tummy, recurring pain, symptoms of shock
symptoms like morning sickness, irritability, breast (fainting, low BP). The risk of torsion is around 10-
tenderness? Is this a planned pregnancy? Is this the 15%. At the moment once your pain settles down we
1st pregnancy? Any miscarriages before? History of will send you home. However, you need to report back
ectopic pregnancy? LMP? Are they regular? When to us if any symptoms develop most likely you will
was the last PAP? Result? need to undergo surgery in that case. Usually
- How’s your general health? PMHx: appendectomy. laparoscopic surgery doesn’t affect early pregnancies.
Have you or your partner ever dx with STD. History of However, slight increase risk of miscarriages. But we
pelvic infection or gynecological procedure done for will give you certain hormones to help maintain the
yourself? pregnancy (progesterone). Do the Surgery after
- SADMA? Blood group? Which contraception were you 15weeks with progesterone therapy. I want you to be
on before the pregnancy? Gardasil? aware of some other complication of ovarian cyst:
Infection: fever, and increasing pain, Cyst might
Physical Examination rupture, twist on its axis compromising the blood
supply to the ovary. However, around 80-95% of
- General Appearance: pallor jaundice dehydration ovarian cyst that presents to us resolves
- Vitals: ask all vitals. If suspect appendicitis ask for spontaneously.
Pulse and BP. - Review: in 6 weeks for U/S.
- Abdomen: Any visible distension, mass, scars?
Palpate any tenderness especially McBurney’s point. Alcohol Excess in Pregnancy
- Pelvic examination:
o Inspection: Any discharge, bleeding? Case: Your next patient is a 10 weeks old pregnant lady who
o Sterile speculum: discharge, bleeding, POC, came in for antenatal checkup. She is alcoholic beverage
OS drinker and a smoker for the last 10 years.
o Bimanual: Any tenderness, adnexal mass,
position and size of the uterus Task
a. History (planned pregnancy; first pregnancy; not a
Investigation: binge drinker; drinks with partner; cannot go without
- U/S: Ovarian Mass alcohol for one day
- Ask the examiner for Doppler U/S: To see the blood b. Advise management
flow to the ovary(torsion), To determine the nature of c. Focus on issues
the cyst: homogenous mass(simple cyst) or a complex
cyst (malignant in nature). History
- FBE, Serum beta HCG. U/S of pelvis and abdomen - Can you tell me a bit more about it? Is it a planned
looking at evidence of intrauterine pregnancy, rule out pregnancy? Is it your first pregnancy?
ectopic pregnancy, ovarian cyst, fluid in the pouch of - I would like to ask you a few more questions
Douglas. especially with your smoking and drinking habits. Is it
- Tumor markers: CA125, LDH alright with you?
- For how long have you been drinking? How much do
Diagnosis and Management you drink per week? What type of alcohol do you
drink? Do you drink a lot on the weekends? Do you
- From history and physical examination, most likely drink alone, with partner or with friends? Are you
your pain is coming from a cyst within the ovary. aware of the safe level of drinking? How long can you
Ovarian cyst is usually a benign condition where a go without alcohol? Do you need it to steady your
fluid filled sac is found near the surface of the ovary. nerves? Does it help you go to sleep? Do you take a
It’s quite common in female of reproductive age group drink in the morning when you wake up? Any
the exact cause is unclear. However, the hormonal symptoms of agitation, sweating, nausea, or shakes if
you don’t drink?
13 
 
- How’s your family life? Any problem at work or with - You can experience withdrawal symptoms like
family relations? Any financial issues? headache, shaking, sweating, N/V, anxiety, tummy
- CAGE? Have you ever tried to cut down? Ever been pain, diarrhea, problem with sleeping, high and low
annoyed? Do you feel guilty? Do you drink when you BP, craving for alcohol and smoking. When you
wake up in the morning? Do you know about its experience these symptoms, please immediately
effects in pregnancy? contact me so appropriate treatment could be given.
- How many cigarettes do you smoke per day and for - Lifestyle modification: Deal with stress in a healthy
how long? What is your pattern of smoking during the way like exercise, sports, meditation and yoga.
day? How soon do you have your first cigarette when - I will refer you to alcohol anonymous. It is an
you wake up? Do you find it difficult to smoke in non- organization composed of groups of people having
smoking areas? Have you tried to quit smoking in the problems with alcohol and who desire to stop it. I will
past? Does your partner smoke? also refer you to support groups – quitline for smoking
- Any medical condition such as liver, gastrointestinal, and give you some reading materials. I’m available for
heart? Any history of mental illness or depression? you for ongoing management and support for follow-
DM? Hypertension? ups.
- Are you on any medications? do you take folic acid? - RWH:
Have you used illicit drugs? o Sometimes it is not possible to stop
altogether.
Counseling o Avoid dehydration by drinking plenty of
- I would like to talk about the effects of smoking and water
alcohol in pregnancy and I would also like to do o Vitamin D, iron and calcium
investigations that we do during the first antenatal supplementation
checkup. o Folic acid for the first 3 months
- The effects of alcohol: In pregnancy, alcohol can pass o Nicotine replacement therapy shouldn’t be
through the placenta to the baby and is broken down used in pregnancy but may refer to
more slowly than in adults leading to fetal alcohol specialist for advice
spectrum disorders. On one extreme is fetal alcohol o Medications for withdrawal: Acamprosate
syndrome which is main cause of mental retardation in (champix) or naltrexone for 6-9 months;
babies. The other effects include vision and hearing
problems, learning, emotional and behavioral Pregnancy with IUCD
problems, speech or language delays, low BW, and
birth defects including heart, face, eyes and other Case: A-26-year old female comes to your GP clinic
organs of the body. complaining that her period is late. She has copper IUCD
- In pregnancy, there is increased risk of miscarriage inserted.
and premature birth. After the birth of the baby,
breastmilk production can also decrease. Task
- Unfortunately, smoking exposes the baby to some a. Take focused history
dangerous chemicals like nicotine, tar, and CO which b. Ask for physical examination (size of uterus is 7
decrease the amount of oxygen for the baby which weeks, no adnexal mass)
can affect his/her development. It can also damage c. Advice the management
baby’s lungs and can give rise to birth defects like cleft
lip and palate, low BW, and once baby is born, there is Case 2: Same Scenario You can see the thread of the IUCD
increased risk of chest infection like asthma,
pneumonia, and ear infections. History:
- In pregnancy, smoking is a risk factor for placental ‐ Hi. I know you are here to see me because you are
abruption and stillbirth. Also, there is an increased concerned about your period. When was your LMP?
chance of SIDS if parents are smoking and drinking. Was it normal or light? Do you have regular cycles?
- I know you are quite worried about hearing all this, but How long is the cycle? How long is the bleeding time?
the good news is that all of these can be avoided if Any spotting in between? Do you have excessive pain
you stop smoking and drinking alcohol. The ideal or bleeding during the period? When did doctor insert
situation is if you stop smoking and alcohol altogether the contraceptive device? Did your period change
if possible for you. The sooner you quit the better it is after the insertion? When was the last time you
for you and your baby. There is no known safe level of checked strings or thread?
alcohol use in pregnancy. (Limit to 1 SD per week but ‐ Pregnancy questions: Do you feel nausea, vomiting,
any reduction is important). breast tenderness? Increase urinary frequency? Any
- Suggestions on how to quit: It is important to abdominal pain? Have you notice usually vaginal
understand the effects of alcohol and smoking and discharge? Did you do pregnancy test? Have you ever
admit it as a problem for you and your baby. Strong been pregnant? Are you in a stable relationship? Have
motivation is the key to success. After making a you ever been diagnosed with STD or PID? When
decision, establish clear and realistic goals and I will was your last PAP smear? Do you know your blood
help you implement them to stop alcohol and smoking group?
altogether. Choose a quit date for both alcohol and ‐ Are you generally healthy? Any pelvic Surgery, C-
smoking to stop. section done before? Medication? Allergies? What
- I can arrange a family meeting to talk to your partner would be your intention if you are pregnant?
and advise him to stay away from alcohol and
smoking. Avoid situation where you usually drink Physical Examination:
alcohol like party and bars. Ask family and friends to ‐ General Appearance
help you quit. Let your family members, friends, and ‐ Vital Signs
coworkers know that you’re trying to stop drinking and ‐ Palpate abdomen: Distension, Tenderness, Masses
smoking.
14 
 
‐ Pelvic examination: History:
o Inspection ‐ IS MY PATIENT HEMODYNAMICALLY STABLE?
o Speculum: Appearance of cervix, OS, The ‐ Congratulations on your pregnancy.
presence of the string ‐ How many episodes of vomiting did you have per
o Check vaginal discharge day? Is it getting worse? Is it in the morning or
o Check if thread is present! throughout the day? What is the content? Do you
o Per Vagina: Size of the uterus (7weeks) have fever? Diarrhea? How is your appetite? Are you
Consistency (soft) Adnexal masses and still drinking eating or drinking? How is your
tenderness. waterworks? Any pain or burning sensation? Any
‐ Urine dip stick and Urine pregnancy test (+) increased frequency? Any change in color of urine?
Any loin pain? Did you eat outside?
Diagnosis and Management ‐ Pregnancy: is this a planned pregnancy? How did you
‐ Jacky your pregnancy test is positive. And I can see a confirm your pregnancy? Any family history of twins?
string which means contraceptive device is in place. Is the pregnancy natural or assisted? Any abdominal
How do you feel about it? cramps or vaginal bleeding?
‐ Unfortunately every contraceptive method has a ‐ Periods? Pills? Partner? Pap smear? Blood group?
failure rate. Effectiveness of IUCD is greater than 98% ‐ SADMA?
but there’s still a chance for being ineffective. You
don’t need to make a decision now. You can go home Examination:
and discuss it with your partner. I will organize a pelvic ‐ General appearance: tired, signs of dehydration
ultrasound for you to identify the exact position of (tongue, skin turgor, CRT?
pregnancy and position of the device. If you decide to ‐ VS: BP (check for orthostatic hypotension); PR
continue with pregnancy the device should be (tachy), RR, T – normal
removed. ‐ Chest, heart, abdomen – normal
‐ Is it safe? The procedure doesn’t increase the ‐ No pelvic exam needed.
miscarriage rate above that in population. (Every ‐ Urine dipstick – nitrites, ketones (+), leukocytes
pregnancy carries 15-20% risk of miscarriage.) o MSU: (+) for ketones! – admit!!!
However if it is left inside it will increase the risk of
miscarriage to up to 25% and increase risk of Investigations:
ascending infection. If you decide not to continue with ‐ MCU
the pregnancy, I will refer you to a specialist for ‐ Ultrasound examination
termination and the device will be removed during the ‐ Test for electrolytes, urea, LFTs
procedure.
Diagnosis and management:
Case 2: NO thread ‐ You have a condition called hyperemesis gravidarum.
‐ Jacky, your pregnancy is positive and I can’t see It means excessive nausea and vomiting in
strings of device. Two options are possible. Either pregnancy. These are common symptoms during
uterus expels the device (because device is a foreign initial pregnancy. However, 1 in 1000 women will have
body) or string loss and you are pregnant with device excessive vomiting and require hospitalization.
still in place. To find out I need to organize pelvic ‐ On examination, you are dehydrated and this was
ultrasound. How do you feel about pregnancy? confirmed in urine analysis, so we need to admit you. I
‐ What if device is still inside? We wouldn’t be able to will organize an ambulance. In the hospital they will
remove it if it still has strings but an attempt to remove secure 2 IV cannulas, take the blood for FBE, U/E/,
it will be made. But without strings it’s impossible to RFTs and LFTs because dehydration can affect the
remove the device safely without harming pregnancy. liver and kidney. We need to do MCS to rule out UTI
However pregnancy can be continued but there’s high and USD to confirm intrauterine pregnancy, rule out
risk of miscarriage and ascending infection. If multiple pregnancy and molar pregnancy.
pregnancy will be successful device will be delivered ‐ They will also give medications to stop the vomiting
with the placenta and membrane. If you decide not to (metoclopramide – mexalon, stemetil) and start IV
continue with pregnancy I will refer you to a specialist fluids and vitamin B6 (pyridoxine).
for termination. ‐ We don’t know the exact mechanism behind it.
However, it is usually due increased level of b-hCG
Hyperemesis Gravidarum which is one of the pregnancy hormones. Once the
body has become used to the new environment, the
Case: 38 year-old woman who came in with a 2-week history of nausea and vomiting settles and this usually happens
nausea and vomiting. She is 8 weeks pregnant and her by 14 weeks.
pregnancy is consistent with GA. She has no previous illness. ‐ You are a 38-year-old mother and that puts you at a
very high risk of having a baby with Down syndrome.
Task So I would like to offer you screening for Down
a. History syndrome (during your 10th week – blood plus USD).
b. Investigations (1 only) -- MSU
c. Diagnosis and management Critical Errors:
‐ Failure to recognize need for hospitalization
Differential diagnosis: ‐ Failure to do ultrasound and urine examination
‐ Multifetal pregnancy?
‐ Hydatidiform mole (complete/incomplete) Generalized Edema in Pregnancy
‐ UTI
‐ Infectious Gastroenteritis Case: A 35-years-old primigravida who is 32-weeks-GA is in
‐ Brain tumor/Addison disease your clinic complaining of increased swelling in the body for the
last few days.
15 
 
Task - It can be reduced by intermittently lying on the left
a. History (both legs and face; decreased when lying and side, elevating the lower extremities intermittently,
on left lateral position; no headache, visual problems, wearing elastic compression stockings, and
tummy ache, had regular antenatal checkups, one decreased salt intake in diet. It usually resolves after
pregnancy, normal placenta, folic acid, sweet drink birth of the baby as the uterus returns to pre-
test normal, no contraceptive, 1st pregnancy, last pap pregnancy size and the hormones return to normal.
smear was one year ago and normal, no general - At this stage there is no need for investigations but if
medical health condition, no HTN; social support is ok there are changes in your symptoms or if patient is
and no financial issues; no problem with waterworks concerned: Do investigations Æ FBE for Hb, infection,
or BM? No fever? platelet, U&E, LFTs, TFTs, RFTs,
b. Physical examination (generally well, mild generalized - Red flags: headache, blurring of vision, tummy pain,
pitting edema of the especially both legs, BMI is 27, increased blood pressure, feeling unwell, baby not
PR 80, T37.6, BP 120/80, RR: 12, neck for thyroid kicking
swelling, no LAD, FH 32cm, lie longitudinal, cephalic, - Reading materials. Review.
FHR 120, pitting edema, urine dipstick proteinuria
negative, BSL normal) Stillbirth
c. Diagnosis and management
Case: You are a GP and a 26-year-old lady comes to you 6
Pregnancy/planned pregnancy weeks after the delivery of a baby. The delivery was a stillbirth
- Antenatal checkup/infections/medications at around 22 weeks of gestation. The patient is still very upset
- 18 weeks ULD – placental, baby, anomalies, liquor about her baby’s death and she wants to know if this will happen
volume, fibroid again.
- Sweet drink test
- Hemoglobin Task
- Pre-eclampsia a. Take history (had fever x 3-4 days continuously at 20
- Heart disease, HTN, DM, heart (CCF), liver, kidney, weeks, and started bleeding/discharge from down
severe hypothyroidism below; ruptured BOW; did tests and baby was already
dead; )
History a. Physical examination
- Is this a planned pregnancy? Congratulations? Is this b. Counsel accordingly
your first pregnancy? Where exactly is the edema
(swelling)? Did it come suddenly or gradually? Any
pain in your legs? Anything increasing or decreasing Approach to Patient Who has had Stillbirth
it? How is your pregnancy going so far? Have you had - Emotional support must be ensured by offering
regular antenatal checkups? Any infections? What appropriate resources or referral
was the result of your midgestation USD (baby, - Take detailed history focusing on obstetric, medical
placenta, anomalies, liquor volume, fibroids)? What and family history and conditions surrounding previous
was the result of your sweet drink test? What about stillbirth
your BP? Have you checked it recently? - Discuss anomaly screening with patient
- How’s your general health? did you have history of - Discuss uterine artery Doppler studies at around 22-
high blood pressure or diabetes? Do you have any 24 week
recent headaches, visual disturbance, tummy pain, - Discuss dating USD in 1st trimester
SOB, chest pain, or racing of heart? Do you have - Discuss lifestyle advice (smoking, alcohol, weight loss,
problems with your waterworks or bowel motions? Is diet)
your baby kicking? Do you have a kick chart? Any - Discuss Serial USD for fetal growth monitoring (28
history of heart problem, liver, kidney or thyroid weeks onward)
problems? Any previous DVT, surgeries or previous - Discuss fetal movement surveillance
hospitalization? Are you on medications such as - Consider timing of birth
steroids? SADMA? FHx of DM or HTN?
History
Physical examination - I am really sorry about the loss of your child. It is quite
- General appearance: pallor, dehydration, jaundice understandable that you feel upset about this. Would
- Vital signs you like someone (a partner or a friend) to attend this
- Cardiovascular examination and JVP discussion? How are you feeling at the moment?
- Lungs - I understand that you pregnancy was at around 22
- Abdomen: FH, lie, presentation, FHT, tenderness weeks, did you have regular checkups until that time?
- Neurological examination: Reflex Did you have blood tests, and USD? What was the
- Peripheries: redness, warmth, tenderness in any result? Can you please tell me what exactly
areas happened? Did you have any symptoms like fever,
- Urine dipstick and BSL rash, vaginal discharge, bleeding? Any problems like
headache, visual changes, or high blood pressure
Management during the pregnancy? Any tummy pain? Were you
- From history and examination, most likely you have a feeling the baby’s movement at that time? What
condition called generalized edema of pregnancy. It happened afterwards? Where did the delivery occur?
typically involves the lower extremities but Did the pain start by itself or was it induced? When the
occasionally it can cause swelling of the face and baby was born, did they notice any abnormal
hands. There are a few reasons such as hormone- features? Did they do an autopsy of the child? Was it
induced sodium retention, increase of blood volume your first pregnancy? Did you ever suffer from a
by 50% during pregnancy, and enlarged uterus may
compress the veins (IVC) when you’re lying down
obstructing blood flow and causing the edema.
16 
 
gynecological problem before that? Did you have any Intrauterine Fetal Death (IUFD)
gynecological surgeries done? Any D&C done? Have
you had regular pap smears? When was the last one? Case: You are an HMO in the ED and a 34-weeks pregnant lady
What was the result? Are you having bleeding now comes in with abdominal pain.
after the delivery? SADMA? Blood group? Gardasil?
- Any FHx of birth defects or stillborn babies? Task
a. History (dull, 2/10, similar to menstrual pain, started 2
Physical examination days ago, no N/V, relieved by paracetamol, no fever,
- General appearance and BMI no burning sensation, first pregnancy, blood group A+,
- Vital signs normal pap smear)
- Thyroid enlargement b. Physical examination (anxious, normal BP and HR,
- Chest and heart FHR is absent with handheld doppler, no discharge,
- Abdomen: size of uterus (involuted), tenderness, water leakage, bleeding, urine dipstick negative, BSL
organomegaly 5.5 mmol/L)
- Pelvic exam: discharge, bleeding, speculum c. Management
- Urine dipstick and BSL
Differential Diagnosis
Causes of Stillbirth ‐ Placental abruption
- Unknown ‐ Preterm labor
- Infections (TORCH) ‐ Pre-eclampsia
- Cervical incompetence ‐ Urinary tract infection
- Fetal growth restriction (IUGR) ‐ Red degeneration of fibroid
- PIH ‐ Appendicitis
- Placental insufficiency
- Thrombophilia defects in mother History
- FHx ‐ I understand you have come to the hospital because
- Abruptio placenta you have abdominal pain. When did it start? Can you
- Chronic diseases in mom describe the pain? Does it come and go? Does the
pain travel anywhere? Can you recall any precipitating
Counseling factor such as trauma, exercise or sexual intercourse?
- Most probably, from the history and examination, the Do you have a fever, headache or blurred vision? Do
most likely cause of the stillbirth that you had was an you have N/V or back pain? Any burning sensation
infection that you developed at around 22 weeks. when passing urine? Have you noticed unusual
There are other causes as well like problems with vaginal discharge? Have you had any vaginal
placenta, poor growth of the baby, and certain bleeding or water leakage? Do you feel the baby’s
coagulation defects. It is important to find out the movements? When was the last time you felt the baby
exact cause before your next pregnancy. However, kick? Is this your first pregnancy? Have you had
sometimes, there is no cause that can be found. We regular ANCU? Any problems with your blood tests,
need to do some tests now after consultation with the midpregnancy USD, sweet drink test or blood
specialist gynecologist which includes FBE, cervical pressure? Do you know your blood group? SADMA?
smear and culture to rule out hidden infections, ANA Did anyone come with you today?
testing, VDRL, FBS to rule out diabetes, thrombophilia
screening, and USD to rule out any structural defects Physical Examination
of the uterus. Later on, the specialist might decide to ‐ General appearance and edema
do a hysterogram. This is an xray of the womb that ‐ Vital signs
helps to find out any defects of the shape of the uterus ‐ Abdomen: uterus, fundal height, lie, presentation,
as well as any growths within the uterus. For your next engagement, palpate uterus if tender or hard/tense,
pregnancy, we will manage you in the high risk FHT
pregnancy clinic. You will have an early dating ‐ Pelvic examination: discharge, bleeding, water
ultrasound at around 8-10 weeks. From then on, you leakage, cervical os, swabs (endocervical and high
will have serial ultrasound after every 2 weeks to vaginal)
monitor the baby’s growth. They will discuss with you ‐ Urine dipstick and BSL
regarding screening for Down syndrome and neural
tube defects. Your pregnancy will be monitored very Management
closely and around 12-14 weeks, the OB will assess ‐ I’m sorry to tell you but I can’t hear the baby’s heart.
you for possible cervical incompetence. We will make Most likely, your baby has died. I can see you’re very
sure that there is nothing that puts your next distressed, do you know what we can do for you now
pregnancy at risk. However, you need to make certain or do you need some time?
lifestyle changes like maintaining your weight within ‐ First of all, we need to confirm this with an USD. We
normal limits, quit smoking, stop drinking, avoiding will also look for signs of placental abruption which is
recreational drugs, healthy diet, and exercise. one of the possible causes for your presentation.
- The delivery will be in a controlled environment at ‐ Why did it happen? We will try to find a cause.
around 38 weeks in the presence of a specialist However, in majority of cases the death is
obstetrician where we will prepare for the possibility of unexplained. For now, I want to order some blood
emergency CS. tests for you. FBE, HbA1c, urine MCS, swabs, LFTs,
- The specialist/midwife will educate you regarding kick U&E, TORCH infection screening, TFTs, ANA and
charting for fetal movement and you need to inform us lupus anticoagulant
if you develop any symptoms like fever, vaginal
discharge, bleeding, rash, or reduced fetal movement.
17 
 
‐ Medical problems are unlikely at least in the first 3 due to fertility medications. Is it risky? Yes. A twin
weeks after fetal death has been diagnosed and pregnancy is usually slightly high risk than the normal
usually, labor will start during this time. You may pregnancy. There are risks to both mom and the
choose to await spontaneous labor or to have labor babies. The maternal complications are exaggeration
induced. Either way you can have a family member or of signs and symptoms of pregnancy, preeclampsia,
friend during the delivery and we will give you premature labor, gestational DM, malpresentation,
adequate painkillers to reduce the pain of childbirth. If antepartum hemorrhage, increased incidence of CS.
you choose to await spontaneous onset of labor, you Fetal complications include PTL, IUGR, twin-twin
will need frequent blood checks. If labor wouldn’t start transfusion (more in monozygotic), malformations.
within 3 weeks, you will need to have labor induction. ‐ Do not worry. You are in safe hands. We will do our
If your choice is immediate treatment, we can prepare best to manage you and prevent the complications. I
the cervix by using prostaglandin. If you or your will refer you to the high risk clinic. The specialist there
partner wish to bring clothes, see or hold the baby, it will follow you up. You will need more frequent visits
is possible. We strongly recommend an autopsy which (every 2 weeks until 28th week, weekly until delivery).
helps us to find a cause in up to 25% of cases. If You may need multiple ultrasounds starting from 28
you’re against autopsy, we can take a small sample of weeks (every 2-3 weeks). Babies will be monitored by
skin usually in the (axilla region) for chromosomal CTG from 34 weeks (2x a week). Aim to deliver the
study. Placenta will also be examined under the babies at 38 weeks.
microscopy and routine cord blood test. ‐ Increased supplements (iron/folic acid), nutrition
‐ To suppress lactation after delivery, you need to wear requirements and rest
tight bra and use simple painkillers and ice packs in ‐ Can I go for vaginal delivery? It is very early to
case of engorgement. comment at this stage, but it will depend on the
‐ We have a bereavement consultant and a social presentation of the first baby and your general health.
worker who can help you with funeral arrangements. If the first baby is cephalic, vaginal delivery is
‐ Technically, you can get pregnant when your periods possible. 70% (cephalic). If there are any
are back. However, it is important to be physically and complications, then specialist might consider doing
emotionally ready for the future pregnancy. cesarean section.
‐ We will also do active management of first stage of
MULTIFETAL PREGNANCY labor because of high chance of postpartum
hemorrhage due to overdistention of uterus.
Multifetal Gestation/Pregnancy ‐ Referral/Review/Reading materials (support groups)
‐ Red flags: bleeding, abdominal pain, water leakage,
Case: A 28-year-old primi who is 18 weeks pregnant comes to headache, blurry vision, urinary symptoms
your GP clinic to collect her ultrasound report that shows twin
pregnancy with 2 placentas and 2 amniotic sacs. ANTEPARTUM HEMORRHAGE

Task Placenta Previa


a. Tell patient about diagnosis and findings
b. Focused history Case: You are an HMO in a hospital OBs-and-Gyne unit and
c. Advise on management your next patient is a 26-year-old 28 weeks pregnant who came
in due to PV bleeding for 1 hour. She has been following up
Complications regularly and there were no remarkable findings up to now.
‐ Maternal: anemia, symptoms of pregnancy (morning
sickness, varicose veins), preeclampsia x3, Task
antepartum and postpartum hemorrhage, a. Relevant history (x 1 hour)
malpresentation, cord prolapse, CS b. Physical examination (pale and stressed, BP:
‐ Fetal: increased risk abnormalities, preterm delivery c. Investigation
(PPROM), IUGR in one of the fetus, twin-twin d. Management
transfusion, perinatal mortality x 5; prematurity,
malformations x 2-4 Risk Factors
- Smoking
Counseling - Previous placenta previa
‐ I’m not sure how this news will sound to you but the - Previous cesarean section
report says it is a twin pregnancy. Don’t worry. We will - Multiparity
take care of you. We will do a multi-disciplinary team - Advanced maternal age
approach which involves me as your GP, a specialist
obstetrician, and pediatrician for your babies. History
‐ History: how is your pregnancy going so far? Any - Is my patient hemodynamically stable?
abdominal pain, bleeding, or discharge? Any - When did the bleeding start? How many pads have
excessive N/V? any headache, blurring vision, burning you used? How soaked were they? Any clots? Any
in urine or leg swelling? Do you know what your blood tummy pain? Headache? Dizziness? Sweating?
group is? Did you take folic acid during the early Palpitations (assess severity of bleeding)? History of
pregnancy? Is it an assisted or natural pregnancy? trauma? Any bleeding disorders (menstrual history/are
Any PMHx? Any FHx of twins, DM, or hypertension? you bleeding from anywhere else)?
SADMA? - Pregnancy: how is your pregnancy going so far? Is it a
‐ We have two kinds of twins: dizygotic coming from 2 single pregnancy? Planned pregnancy? Significant
eggs and monozygotic which comes from one egg. In findings in 18 week ultrasound? How was your sweet
your case, it is a dizygotic pregnancy with 2 sacs and drink test? Is the baby kicking? Previous pregnancy?
placenta. Twin pregnancies run in families or might be How was placenta in previous pregnancy?
- Pills? Partner?
18 
 
- Period: are they too heavy or normal? Pap? Are you Mild Abruptio Placenta
aware of your blood group?
- Social history: how far are you staying from the Case: You are an HMO in ED and a young primigravida who is
hospital and do you have enough support? 30 weeks gestational age comes to see you because of vaginal
- SADMA? bleeding on examination, she is stable and vitals are normal.
Abdomen is not tense but slightly tender. FHT 140/min
Physical examination
- General appearance: pallor, dehydration and jaundice, Task
signs of trauma a. Focused history
- Vitals: sitting and standing BP, RR, PR, T, oxygen b. Explain condition to patient
saturation c. Management
o If with postural hypotension: I would like to
insert 2 IV bore cannulas, take blood for Features
blood group and crossmatching and start IV - Separation of the placenta from the uterus
fluids Revealed: bleeding
- Abdomen: FH (whether it corresponds to gestational - Concealed: severe hypotension
age), lie, presentation, tenderness of uterus, - Complications: IUFD, DIC (micro thrombi)
engagement/floating, FHR - Types:
- Pelvic exam: NO Per Vagina Exam!!! o Mild - blood loss <500ml; no fetal
o Inspection: discharge, blood, clot, signs of compromise; USG to exclude retroplacental
trauma clots; CTG; -- bed rest; ambulate slowly
o Speculum: discharge, blood, cervical os (if once bleeding stops; if term then might do
open or close); labor induction by amniotomy
- Urine dipstick and BSL o Moderate - about 1/4 of placenta has been
detached; blood loss >1L; severe abdominal
Placenta Previa: tenderness; shock; fetal compromise;
- Total placenta previa (completely obstructs the Admission and stabilize patient; if fetus alive
cervical os) then cesarean section
- Partial Placenta previa (partially obstructing the o Severe - more than 1500ml of blood, shock,
cervical os) severe tenderness, fetus is almost always
- Marginal (just at the beginning of the os) dead; DIC and coagulopathies are common
- Low-lying placenta - Risk factors
o Multiparty
Diagnosis and Management o Hypertension in pregnancy
- Most likely, you have placenta previa. At this stage, I o Smoking
would admit you, put 2 IV lines and take blood for o Cocaine abuse
FBE, blood grouping and crossmatching, and o Trauma
coagulation profile. I will call the OBS&Gyne Registrar
to come and have a look at you Differential diagnosis
- We need to organize an urgent USG to see the - Placental abruptio
position of the placenta and the obs and gyne registrar - Premature labor
might also consider doing CTG to check the status of - Red degeneration of fibroids
baby. - Trauma
- Placenta previa is an obstetric complication that - Placenta Previa
occurs in the 2nd half of pregnancy. It can cause
serious complications in both mom and baby. History
Complications are fetal malpresentation, postpartum - Is my baby okay? I understand that you are stressed.
hemorrhage, rebleeding, IUGR, isoimmunization Before I answer your questions, I would like to ask you
- Reassure a few details regarding your pregnancy. Are you still
- Further management: bleeding? When did it start? What were you doing
o Total or partial: send to tertiary hospital and when it started? How much is the blood loss? How
stay until delivery; most cases delivered via many pads did you use? Was it fully soaked? Did you
CS pass any clots? What was the color? Any gush of
o Marginal or low-lying and with minor water coming out with the blood? Any tummy pain?
bleeding and bleeding has stopped: go Do you feel dizzy or palpitations? Do you have any
home but needs to stay close to hospital; bleeding disorders in you or the family?
USG at 34 weeks; delivery: depends on How's the pregnancy so far? Are you regular with your
USG at 34 weeks and specialist will decide antenatal checkups? Do you remember your mid-
on that; CS organized at 38 to 39 weeks gestation USG? How was your sweet drink test?
Anything abnormal? Is your baby kicking? Is it
o If with severe bleeding and with fetal reduced? Do you know blood group? Any previous
compromise Æ immediate cesarean section pregnancies? Are you regular with your pap smears?
- Refer, Review and Reading materials Smoker? Illicit drugs?
- Red flags: bleeding, baby not kicking, water breaks,
tummy pain
19 
 
Diagnosis and management: ‐ Blood loss may also affect your baby’s condition in the
same way. Less blood will come to the baby and he
- You have a condition called a mild placental will experience hypoxia.
abruption. Draw diagram. I will need to organize some ‐ You are losing RBCs which have a very special
investigations to confirm the diagnosis and make sure function to carry oxygen to all organs and tissues in
the baby is alright. I will request for an USG to check your body. The only way to restore that is by blood
the degree of abruptio and request for CTG, full blood transfusion.
examination, UEC, crossmatching, coagulation profile, ‐ No doctor I still don’t want to be transfused. Miriam, I
indirect coomb test and kleihauer test. understand and respect your wish. Can you please tell
- Is this hospital a tertiary hospital? I would like to admit me if you fully understand all possible consequences
you. At this stage the bleeding has stopped but the which may arise without blood transfusion?
condition is risky. I will call the OB registrar to come ‐ We will do our best to save you and your baby’s life
and have a look. Meanwhile I will secure IV lines and without blood transfusion.
collect blood for investigations. Because you are RH-
we will give you anti-D injection. We will consider Examiner: Summarize legal and ethical issue in this situation
injection of Betamethasone 2 injections 12 hours apart ‐ Every competent patient has the right to accept/refuse
to help with the maturity of your baby's lung. treatment.
- What about the delivery? At this stage, we cannot say. ‐ In this case, there is the potential to damage the fetus.
You are stable now. We have to wait for the results of In Australia, the fetus has no rights.
the ultrasound and CTG. However, if your baby ‐ Born babies can be transfused without parental
becomes distressed or the bleeding recurs, the consent providing it is a life-saving procedure.
specialist might decide to do an emergency cesarean
section. How would you manage this situation?
- Reassure. ‐ I will continue monitoring mother and baby’s condition
- If in pain, IV pethidine. until emergency cesarean section has been arranged.
‐ I need to continue fluid transfusion.
Blood Transfusion Consent in APH ‐ I need to talk to senior doctor to help me.
‐ If husband is available, then can talk to husband.
Book case 123:
Pregnancy with Road Traffic Accident
Task
a. Ascertain patient’s view on blood transfusion Case: You are an HMO in ED and a 23-year-old female comes
b. Explain risks and benefits of treatment to patient and to you complaining of tummy pain. She is 32 weeks pregnant
baby and was in a car accident.
c. After six minutes, answer examiner’s question
Task
Counseling b. History
‐ Hello Miriam, I want to explain your condition and c. Physical examination (B-, stable VS pallor,
possible ways of management. You are having generalized tenderness, FH=GA, cephalic, FHS +, no
severe bleeding, and we call it antepartum bleeding, or contractions)
hemorrhage. An emergency USD confirms placental d. Management
previa. Placenta previa means the placenta is lying in
the way of the baby. This condition is a common History
cause of bleeding in pregnancy. Thirty percent of all - Is my patient hemodynamically stable?
APH is because of placenta previa. It usually presents - I would like to talk to my patient preferably in a
with causeless, painless and recurrent bleeding. resuscitation cubicle with all the necessary
Sometimes, it can present with severe bleeding like in resuscitation equipment.
your case. It is a dangerous condition and can - How are you feeling at the moment? Let me reassure
seriously affect you and your baby. you that you are in safe hands. If you want, I can call
‐ I know you are feeling unwell. Your BP is low and your someone to be with you.
pulse is fast. They are signs of hemorrhagic shock - I understand you have tummy pain? Where is it? How
due to blood loss. Most likely you lost at least 30% of bad is it? What is the type of pain? Does it come and
your blood volume and you are still bleeding. It is an go or is it constant? Does it go anywhere else (back or
indication for blood transfusion to increase you and towards genitalia – to r/o pelvic organ damage)? Do
your baby’s chance for survival. No doctor I still don’t you think it is getting worse?
want to be transfused. - Can I ask more about the accident? When did it
‐ Miriam, it is your right to refuse the treatment. happen? How? Who was driving? Were you in the
However, it is important for me to explain the possible passenger seat/backseat? How fast was it going?
outcomes so that you can make an informed decision. Were you wearing seatbelt? Do you have bleeding, or
Right now, we are doing important preparation for discharge from down below? Do you have headache,
emergency cesarean section. Bleeding will stop after N/V? Did you hurt your head? Is there a wound
we empty the uterus. However, the operation itself is anywhere on your body? Did you lose consciousness
associated with blood loss (500ml) and may worsen at any time? Do you feel the baby kicking?
your condition if blood transfusion is not started. We - Previous obstetric history? What is your blood group?
can replace fluids and use synthetic blood substitutes Partner’s blood group? Did you receive any injection
(Haemaccel). They will reduce shock. However, if of anti-D during pregnancy? Any past history of
blood loss exceeds 40% of blood volume, the biggest bleeding disorders, clotting problems, illnesses?
problem is hypoxia or oxygen deprivation, which may SADMA?
quickly lead to multiorgan failure/shutdown and death.
20 
 
Physical Examination Differential Diagnosis
- Full primary survey of the patient - Placental abruption
- Ensure her airway, breathing, circulation are not - Placenta Previa
compromised - Acute red degeneration of fibroid
- Inspect for any visible signs of trauma all over the - Preterm labor
body? Bruises? Pallor? Dehydration? - Appendicitis
- Vital signs especially BP - Bowel obstruction
- Secondary survey looking for signs of trauma to the - Torsion
bones, joints, vessels (pulses)
- Abdomen: palpate any tenderness, guarding, rebound History
and signs of ecchymosis, large bruise over lower - Is my patient hemodynamically stable? I would like to
tummy, fundal height, lie, presentation, FHS, uterine take a history and I would like to ask for IV access.
contractions - Can you please tell me more about the pain? When
- Pelvic exam: visible bleeding, discharge, signs of did it start? How bad is it on a scale of 1-10? Can you
trauma, nitrazine test (if pH >5 normal; >5-7 amniotic point out where exactly is the pain? Does it go
fluid) anywhere else like towards the flanks or downwards
- B/E Æ preferable done by obstetrician at tertiary care toward the pubic area? Did you hurt yourself in that
- Urine dipstick and BSL area? Do you think this pain is associated with
- Heart and chest N/V/headache/dizziness? Any bleeding from down
below? Vaginal discharge? Leaking of water? Do you
Management think the pain is continuous or does it come and go?
- I would like to ask for review by specialist obstetrician Do you think this pain was related to sexual activity
and I will arrange for blood tests such as FBE, U&E, (placenta previa)? Do you have any associated
BSL, crossmatching, D-dimer, coagulation profile, problems with waterworks? Any history of constipation
ABG, Kleihauer test to determine amount of or bowel-related problems? Is this your first
fetomaternal hemorrhage to decide the amount of pregnancy? Any miscarriages before? Have you had
antiD, ECG and xray of pelvic girdle. regular antenatal visits? Are you aware of the results
- Risk of fetal deformities are minimized during the third of your last USD? What was the position of the baby?
trimester and usual radiation exposure is very low Placenta? Can you feel the baby kicking? How often
o Typical pelvic xray – 0.10 mGyron during the last 1 hour? How is you general health?
o CT scan – 20-50 mGyron Any medical or surgical condition? Is this the first
o Proven risk - >50-100 mGyron episode of pain? Do you have any fever along with the
- I would also like to do USD to assess for fetal viability, pain? What is your blood group? What is your
size, gestational age, and position of baby, and any husband’s blood group? Have you received any anti-D
evidence of intraperitoneal fluid or hemorrhage. Also, I injections up to now?
would like to hook you to continuous CTG for 24 hours - FHx of HPN, DM, bleeding disorders
to look for any signs of fetal distress.
o Indications for CTG: bleeding, previous CTG Physical examination
is abnormal, trauma - General appearance
- Most likely, the obstetrician will advise anti-D IM - Vital signs: postural BP drop
dosage after the results of the Kleihauer test. - Abdomen: FH, lie/presentation, FHR, tenderness on
- Criteria for admission palpation, guarding, rigidity, tenderness especially
o FHR on CTG shows variable decelerations over the RIF/LIF?
o Serious trauma after second trimester where - Pelvic: inspect for signs of bleeding, discharge,
the patient requires fetal monitoring for 24 leaking; insert sterile speculum for bleeding,
hours or more discharge, pooling of fluid, nitrazine test; os if it is
o Abnormal obstetric findings like vaginal open or closed
bleeding - Bimanual examination is preferably avoided until an
- Keep admitted until all possible complications have ultrasound has been obtained
been ruled out such as fetal death, premature ROM
leading to PTL, abruptio placenta (can be delayed for Investigations
up to 48 hours), amniotic fluid embolism, and - FBE, blood grouping, crossmatching and hold. I would
chorioamnionitis like to do a CTG to assess fetal distress, coagulation
profile, LFTs, RFTs. If she is RH (-) I would like to do
Concealed Placental Abruption kleihauer test to check fetomaternal hemorrhage.
Also, I would organize an USD to check for fetal
Case: You are an HMO and a 25-year-old female who is 32 viability, position of placenta, and if there is any
weeks pregnant came in due to sudden onset of severe abruption of the placenta and amount of blood.
abdominal pain.
Management
Task - Unfortunately, what you have is a serious condition
a. History (pain is 7/10, sharp pain, started 45 minutes called placental abruption. Basically, a part of the
ago, bending over decreases the pain, baby kicking; placenta starts detaching from the wall of the womb
waterworks normal, ANC, USD, and sweet test for some reason. The exact cause is not known.
normal) However, there are certain risk factors like trauma,
b. Physical examination (pale, anxious, sweating, smoking, high blood pressure in the mom, diabetes,
increased HR, normal BP, RR, O2, Temperature, FH previous history of placental abruption, high parity,
appropriate for age, tender all over, mainly around poor nutrition, and sometimes, it is unexplained.
umbilicus, guarding +, cephalic, FHS+; pelvic no - This condition can be quite serious as there is a high
bleeding or discharge; os closed) risk of PROM, PTL, fetal distress, maternal shock,
c. Diagnosis and management acute renal failure and sometimes, IUFD.
21 
 
- I need to admit and you need to be urgently seen by a maintained. Aim is to keep the BP between 140/90
specialist obstetrician. We will start you on IV fluid and and 160/100; add beta-blocker if with tachycardia)
monitor your urine output with the help of catheter. We after the patient has been seen by the specialist.
will send blood for crossmatching. If required, we - At GP clinic Æ methyldopa or nifedipine spray to
might need to transfuse you. We need to prepare for lower BP
possible premature delivery. I will inform the theater to - I would inform the obstetric team to organize for
prepare for emergency cesarean section. We will give immediate delivery.
you steroids to help with the maturation of baby’s - I would like to monitor the patient by monitoring her:
lungs. urine output, continuous ECG, reflexes, vital signs.
- Usually, with moderate to large placental abruption, - Investigation: FBE, U&E, LFTs, coagulation profile,
there is a need to deliver the baby ASAP. We will also blood grouping and crossmatching, USD, CTG,
give you anti-D injections to prevent any thrombophilia screening
incompatibility of blood groups. - Aim of treatment: prevent development of fits
- If the baby is non-viable, if you are stable, we will - Aim of treatment if with fits: deliver the baby
induce and deliver the baby. But if not, emergency - I would like to call in obstetrician. If the pregnancy is
cesarean section is performed. less than 34 weeks, we will give the patient steroids
Betamethasone (Celestone) 11.4mg IM 2 dose 12
HYPERTENSIVE DISORDERS OF PREGNANCY hours apart, stabilize patient, and monitor all
symptoms. The patient remains at the hospital for
Pregnancy-Induced Hypertension/Pre-eclampsia observation. If symptoms worsen, we deliver by CS.
- If pregnancy is more than 34 completed weeks,
Case: You are a GP and a 30-weeks-pregnant primigravida deliver by induction or cesarean section.
comes to your clinic. She is complaining of headache. Her BP - If platelets are going low Æ give FFP
today is 170/110mmHg. It was the same on a previous - If patient develops symptoms of pulmonary edema Æ
occasion. give high-flow oxygen and diuretics.
- Complications: ARF, cardiac failure, cerebral
Task hemorrhages, DIC, IUD, HELLP syndrome,
a. Relevant History
b. Manage the case Counseling of mom
- What your daughter has had just now is a fit as a
History consequence of a very high blood pressure. This
- Is my patient hemodynamically stable? I would like to condition is known as PIH. This can happen because
know all the vital signs. of certain chemicals that are released by the placenta
- I would like to give the patient methyldopa now. that cause constriction of blood vessels and formation
- If she develops fits while talking rectal diazepam 5-10 of clots because of reduced supply to the brain
mg resulting to the fit. It is very important to control the
- I would like to ask some history from the patient. symptoms to prevent complications like liver failure,
Please tell me if you’re having symptoms like heart failure, and kidney failure. That is why we are
headache, BOV, tummy pain, or bleeding from down sending her to the hospital right away. She will be
below? Any abnormal feelings that you have seen by a specialist OB. They will lower her BP with
(derealization)? Do you feel the baby kicking? Have medications, but the cure is to deliver the baby.
you noticed any leaking from down below? Any
swelling of your ankles? Have you had all regular CARDIOVASCULAR, RESPIRATORY, HEMATOLOGIC,
antenatal checkups? USD? Blood tests? Sweet test? NEUROLOGIC, GASTROINTESTINAL CONDITIONS IN
Are you generally healthy? Any medical or surgical PREGNANCY
condition? Before this pregnancy, have you ever been
diagnosed with high blood pressure, kidney problems, Abdominal pain (Early Pregnancy) Æ Uncomplicated Cystitis
DM or any other conditions? FHx of similar condition?
SADMA? Blood group! Case: You are an HMO in the ED and your next patient is a 12-
week GA pregnant lady complaining of nausea and vomiting.
Physical Examination This is her first pregnancy. No complications so far.
- General appearance
- Vital signs Task
- Evidence of pedal edema (pitting or non-pitting) a. History (N/V with dull, nonspecific lower abdominal
- Funduscopy for bulging of the disc pain x 2 days; malaise; tolerate meals)
- Chest and heart b. Physical examination (T:37.6, BP: 115/80; soft, tender
- Abdomen (FH, lie/presentation, FHT, tenderness) in lower abdomen but not peritonitic signs; os is blue,
- Reflexes closed, no secretions or bleeding; free adnexas;
- Urine dipstick and BSL fundus expected high according to gestational age;
nitrites ++++, leukocytes +++, no blood, protein and
Management sugar)
- I would like to put the patient on the left lateral position c. Diagnosis and Management
and call for help. Check the airway. If there are
secretions wipe with clean cloth or suction. Put airway. History
Give oxygen by mask if possible. IV access. I would ‐ Was it a planned pregnancy? Congratulations. When
like to start her on magnesium IV LD 4gms over 15 did it start? Are you able to tolerate meals? Do you
minutes diluted with NSS and continue with 1 gram feel tired? Do you have abdominal pain? What type of
divided over 24 hours. IV hydralazine (5-10mg bolus pain? Any discharges or bleeding? Fever? Pregnancy
given over 5-10 minutes then an infusion of 5mg/hr is checks? Did you have any tests done? Are you taking
folic acid? Did they mention your blood group?
22 
 
‐ PMHx: any history of UTI; Diagnosis and Management
‐ 5Ps: previous miscarriages; twin pregnancies? - You have a condition called iron-deficiency anemia. It
is the most common cause of anemia in pregnancy. It
Physical examination: is often asymptomatic and detected on screening as in
‐ General appearance: pallor, dehydration your case. There is high demand of iron during
‐ Vital signs: stable except temperature 37.5 pregnancy and in your case, most likely the reason is
‐ ENT/CVS/Lungs due to the inadequate gap.
‐ Abdomen: tenderness of lower abdomen - There are some risks to you and your baby because
‐ Pelvic: of this. Anemia can predispose you to infections,
o Inspection of genitalia: bleeding, discharge; excessive blood loss during pregnancy, and can affect
o Speculum: no bleeding or discharge; os is your heart. Because of this, there is reduced oxygen
close; supply to the baby which can lead to IUGR, fetal
o Bimanual examination: adnexa are free; distress and in severe cases, stillbirth.
uterus is expected high - We need to give you iron supplements. 200 mg 2x
‐ Urine dipstick and BSL daily. The hemoglobin level should increase by 1gm/L
per week. There are some side effects like nausea,
Diagnosis and Management tummy pain, black stool, and constipation. We will
‐ You have a condition called UTI most likely what we stop the medications 3 months after your hemoglobin
call cystitis. Do you know what it is? At this point, I am levels become normal. I would also advise you to eat
happy to send you back home, but if the condition more iron-rich foods such as iron-fortified cereals,
persists and you cannot tolerate foods or drinks, fever, legumes, nuts and nut butters, seeds, wholegrain
chills and pain in the back, then go to the ED. breads, green leafy vegetables, dried fruit, iron-
‐ I will need to send your urine for culture and sensitivity enriched breakfast cereals, milo and ovaltine and liver.
and I am going to start you on amoxicillin 500 mg TID Eating a lot of vitamin C rich foods to increase
or cephalexin 500 mg BID or nitrofurantoin 50mg QID. absorption of iron;
If pyelonephritis: ceftriaxone IV 1g OD. - Parenteral iron indications: if close to delivery and if
‐ I will review you in 3 days and we will either continue cannot tolerate oral iron and Hgb <7g/L
your medication or change it according to the results - Anemia: Hgb <110g/L in 1st trimester and <100g/L in
of your culture. I will prescribe paracetamol for the late second or third trimester. Iron requirements
abdominal pain and metoclopramide for vomiting. increased to 1300mg/day.
- Review after 2 weeks. Refer if not increasing.
Anemia in Pregnancy - Reading material.

Case: You are a GP and a 28-year-old G4P3 20 weeks’ Asthma in Pregnancy


pregnant lady has come to see you to know the results of the
recent blood tests. The blood tests hemoglobin is low, MCV is Case: A 26-year-old female who is 20 weeks pregnant comes to
low, transferrin is high, ferritin is low. your GP clinic complaining of SOB.

Task Task
a. History a. History (had mild asthma and use ventolin PRN;
b. Physical examination sudden, fever; with wet cough; greenish or yellow; ex-
c. Diagnosis and Management smoker and partner is a smoker)
b. Physical examination (in distress; audbible wheeze,
History BP 120/80; T:38, RR26, O2 93, HR 100; RR
- I understand that you’re here for your blood results increased; increased work of breathing, retractions,
and it was found that you have iron deficiency anemia. increase vocal fremitus in right lung base; dullness on
This means that there is less oxygen delivered to the right lung base and decreased air entry on right lung
tissues. base and diffuse wheezing; FH 20cm, FHT 140,
- Do you feel tired? Is there any dizziness, palpitations uterus soft and non-tender)
or SOB? c. Diagnosis and management
- How is your pregnancy so far? Have you had regular
antenatal checkups? How are your blood tests and Differential Diagnosis
ultrasound? How about your pregnancies? Have you - Pulmonary embolism
had blood loss? When was your last pregnancy? - Asthma exacerbation
- What about your periods? Did you have abnormal - Spontaneous Pneumothorax
bleeding? What about your diet? Any bleeding - Heart failure
disorders? Are you on any special diet? Is the baby
kicking? Any other health problems? History
- Blood group? - I understand you have come to see me because of
SOB? When did it start? Did it happen suddenly or
Physical examination gradually? Do you feel SOB at rest or only on
- General examination: pallor, bruising, lethargy, exertion? Does anything make it better or worse? Is it
- Vital signs: postural drop the first episode? Do you have a fever, shivers? Do
- Lungs you have a cough? Is it dry or wet? What’s the
- Cardiac: murmur (systolic) sputum? Did you notice blood in the sputum? Do you
- Abdomen: FH (check for IUGR), abdomen soft or have chest pain or tightness? Is it worse with deep
tense, FHT inspiration? Do you feel your heart is racing? Do you
- Pelvic examination: bleeding, discharge feel nauseous? Have you been vomiting? Do you
- Urine dipstick and blood sugar have abdominal pain? Have you felt the baby kick?
23 
 
Any Vaginal discharge or bleeding? Waterworks and b. Physical examination (vital is normal, no thyroid
bowel? Calf tenderness or swelling? enlargement, no carotid bruits, no rashes or jaundice,
- Is it your first pregnancy? How’s your pregnancy so chest normal, apex is normal, tapping, auscultation
far? Any problems with blood tests? Any problems low-pitch, rumbling diastolic murmur best heard with
with USD? Do you know your blood group? bell on the left lateral position; soft, nontender, no
- Can you please tell me more about your asthma? hepatosplenomegaly; ankle edema)
When were you diagnosed? How often do you get c. Diagnosis and management
attacks? Do you have symptoms between attacks?
Which medications are you on? Have you ever been History
admitted to the hospital with severe asthma attack? ‐ I know you have come to see me because you are
Have you ever been admitted to the ICU or ever been short of breath. When did it start? Did it happen
intubated? Do you know any precipitating factors suddenly or gradually? Do you have SOB at rest or
(smoking, cold air, exercise, dust, pollen, infection)? only during physical activity? How far can you walk
Other medical or surgical conditions in the past? Are (block)? Is it the first episode? Associated symptoms:
you on any medications? Allergies? Smoking? fever, cough, noisy breathing, chest pain or tightness?
- What do you do for a living? Any recent history of Do you notice your heart is racing or beating
travel? irregularly? Do you sleep flat? How many pillows do
- FHx of asthma you use when you go to sleep? Have you ever woken
up at night SOB? Have you noticed swelling of your
Physical examination ankles? How’s your appetite? Do you feel tired? Do
- General appearance and peripheral cyanosis; signs of you have N/V? Do you have abdominal pain? How’s
respiratory distress your waterworks? Any unusual vaginal discharge or
- Vital signs bleeding? History of travel?
- ENT ‐ I know it’s your first pregnancy, is it planned? Did you
- Chest: see any doctors regarding your pregnancy? How did
o Inspection: use of accessory muscle; chest you confirm pregnancy? Do you know your blood
expansion; group? When was your last pap smear?
o Palpation: chest expansion; vocal fremitus ‐ PMHx: Are you generally healthy? Any serious
o Percussion: dullness or hyperresonance condition or surgeries in the past? Any heart or lung
o Auscultation: air entry; disease? Can you remember what type of treatment
o Peak-flow meter did you receive? Did you have regular follow-ups?
- Heart Medications? Allergy? Smoking? Alcohol? Do you
- Abdomen: FH, FHT, uterus is soft and nontender have enough support? FHx of heart or lung problems?

Management Physical Examination


- According to your history and PE, it is most likely an ‐ General appearance
acute asthma attack secondary to chest ‐ Vital signs: PR (regular)
infection/pneumonia. You need to be admitted in the ‐ CVS: peripheral/central cyanosis and pallor, JVP,
hospital. You will receive treatment to control the inspection and palpation of precordium, apex beat,
asthma attack. Most likely you will be nebulized with thrills, murmur
ventolin, ipratropium and IV steroids and treatment of ‐ Lungs: evidence of pulmonary edema or pleural
infection with IV antiobiotics. effusion
- Which antibiotic would you like to use? Benzypenicillin ‐ Abdomen: hepatomegaly, tenderness
or amoxicillin or erythromycin if with allergy ‐ Legs: peripheral edema
- You will also be hydrated with IV fluid and have
oxygen. Diagnosis and Management
- You will have some tests: FBE, ESR/CRP, blood ‐ According to your history and PE, I suspect heart
cultures, and U&E, sputum for microscopy and valve disease which is called mitral stenosis. Your
culture; CXR if indicated heart has 4 chambers. Mitral valve separates the
- How long will I stay in the hospital? You will probably upper and lower chambers on the left side of the
stay for a few days. We need to control your heart. Stenosis means the valve doesn’t open fully
pneumonia and asthma. If your temperature has restricting blood flow. Most likely, it’s the complication
returned to normal for 48 hours and you are free of of rheumatic fever.
asthma symptoms, you will be discharged and treated ‐ I will refer you to the cardiologist for further
as an outpatient. assessment. You need to have ECG and echo to
confirm the diagnosis and assess the severity and
Pregnancy plus Cardiac Murmur heart function.
‐ Normal pregnancy is associated with significant
Case: You are a GP and a 32-year-old 10- week (or 20-week) hemodynamic changes (increased blood volume) and
pregnant lady came in due to shortness of breathing. She is a your heart will need to work harder and may worsen
primigravida. your condition. That is why, for the best outcome of
your pregnancy, you will be managed in a high-risk
Task pregnancy clinic. You will be seen by an OB,
cardiologist, midwives and GP.
a. History (1st pregnancy; noted SOB x 1 month ago with ‐ During the pregnancy you will have more frequent
occasional palpitations, no chest pain, especially with follow-ups and the cardiologist will make a decision
walking, + history of RF 6-7 years old, about treatment
24 
 
‐ NYHA (Heart Failure) Diagnosis and management
o I – no symptoms but with signs of cardiac ‐ I would like to organize some confirmatory tests
damage because the GCT we did 2 days ago is one of the
o II – symptoms comes with ordinary physical screening tests so I would like to do the oral glucose
exertion; dyspnea, SOB, palpitations, tolerance test. In that test, you have to fast and we’ll
tiredness; usually treatment not required in give you oral glucose and measure the blood sugar
pregnancy; monitor for deterioration levels at 1 and 2 hours. In addition, I would also
o III – asymptomatic at rest but symptomatic organize FBE, HbA1c, urine MCS, USD and CTG.
with minimal physical exertion; treatment
required ‐ From the history and examination, you have a
o IV – symptomatic at rest; admission to condition called gestational diabetes. This means that
hospital for treatment during the pregnancy your blood sugar has increased
‐ Will I be able to deliver vaginally? Most women who too much. Gestational diabetes is the result of the
have heart disease have an easy spontaneous labor. hormone called Human placental lactogen (HPL)
There is no indication for inducing labor. During labor, produced by the placenta, progesterone, beta-hCG,
you will need to be on your side or well-propped up to and cortisol. All these substances/hormones have
avoid compression of the major vessels (aorta) which anti-insulin effects.
may cause marked decrease in BP. If there is a delay ‐ Gestational diabetes increases the mother’s risk and
at the 2nd stage of labor, instruments (forceps/vacuum) puts the baby at risk as well. Mother’s glucose crosses
will be used. If there is a need for medication to the placenta, but insulin cannot and fetal pancreas
stimulate uterine contractions, oxytocin is the gets activated and starts secreting additional insulin
preferable one. You will be closely monitored during and because of that baby’s can become macrosomic
the delivery and after. (large babies), hence we will do frequent USD. The
‐ If heart failure needs to be treated, same drugs (beta- baby may also develop jaundice and there is
blocker, digoxin, diuretics) are given as for non- increased chance of premature delivery, which leads
pregnant apart from ace inhibitors. to prematurity and hyaline membrane disease. There
‐ Do you have any questions? is also a risk of neonatal hypoglycemia because of
increased insulin, as well IUGR and IUFD.
ENDOCRINE DISORDERS OF PREGNANCY ‐ Do not worry. You are in safe hands, but with good
monitoring done by the MDT these risks can be
Gestational Diabetes minimized dramatically. I need to refer you to a
diabetic physician/endocrinologist, obstetrician,
Case: Your next patient in GP practice is a 28-year-old woman dietitian and diabetic educator. Our main aim is to
who is 28-weeks pregnant. She returns to you for the results of maintain the BSL to <7mmol/L by dietary
the GCT with 75 grams of oral glucose load done 2 days ago. modifications. You need to measure you BSL at least
The plasma glucose level was 9.2mmol/L (N<8mmol/L) after 1 3x a day. If the BSL is not controlled with the diet we
hour. will start with insulin. I would advise you to maintain a
diary of your BSL. You might also need to be reviewed
Task by ophthalmologist and kidney specialist. We will
a. Further history (FHx of DM, regular PNCU, no monitor you by doing the HbA1c and urine protein
symptoms of DM) (microalbuminuria).
b. Explain the results ‐ From 32 weeks of pregnancy, we will start doing CTG
c. Examination (FH, FHT +, cephalic) to monitor the baby. If your sugar is well controlled, we
d. Diagnosis and outline management will do it once a week but if not, we will do 2x a week.
If euglycemia is achieved, the specialist may give you
History a trial of labor/normal delivery. They will also organize
‐ How’s your pregnancy going so far? How was your USD in the 3rd trimester to assess the growth of the
midpregnancy ultrasound? Is this your first baby and we do a planned delivery at term (38
pregnancy? Have you ever been diagnosed with weeks). Depending on how your glucose levels are,
diabetes before? Recurrent thrush/candidiasis? you might need intermittent insulin injections during
Polyuria, polyphagia? Any other previous illnesses or labor. If baby is big or any other complications
surgeries? Any FHx of diabetes? Blood group? Are happen, the specialist might consider doing cesarean
you regular with pap smear? Weight before pregnancy section.
and weigh now? ‐ Don’t worry. We just have to control your blood sugar
‐ Do you think your tummy is more distended than what level.
you expect it to be? Any previous pregnancies or ‐ Will I remain diabetic? Usually, the diabetes will
miscarriages? Do you have headache, frothy urine, or resolve after delivery. However, there is an increased
blurred vision? SADMA? Social history? chance of recurrence in succeeding pregnancies and
30% risk of developing DM later in life. Hence, we
Physical examination need to organize a followup GTT 6-8 weeks after
‐ General appearance: edema, BMI delivery and to be checked 2 yearly (every 5 years in
‐ Vital signs clinical book).
‐ Chest and heart ‐ Red flags: uterine contractions, leaking of water, etc.
‐ Abdomen: FH, lie or presentation, floating/engaged,
FHT
‐ Pelvic examination: discharge, spotting/blood, os,
25 
 
INFECTIONS IN PREGNANCY ‐ Complete damage of baby (>45%) if mother exposed
during the 1st trimester
Rubella exposure in pregnancy ‐ Further damage can result to deafness and cataract
‐ Offer termination of pregnancy
Case: 28-year-old schoolteacher presented in your GP clinic ‐ NO vaccination in pregnancy
concerned she was exposed to an 8-year-old student who was ‐ Can cause abortion, miscarriage, stillbirth, IUGR, fetal
confirmed to have Rubella infection. She is not sure if she is infection
pregnant or not. LMP was 10 weeks ago. ‐ Congenital rubella:
o Cataract, deafness, developemental delay,
Task: irritability, mental retardation, microcephaly,
a. Talk to the patient neurologic (meningoencephalitis)
b. Discuss her concerns o Heart: patent ductus arteriosus, tricuspid
c. Answer her concerns stenosis 

History: Chickenpox in pregnancy


- I know that you are concerned about being exposed to
a child with rubella and being pregnant. How long Case: 25-year-old G3 female who is currently 10 weeks
have you been exposed to this child? Have you had pregnant. She is worried because her son has chickenpox.
any fever? Rash? Body ache? Did you have any
previous vaccination against rubella or any chance Task:
you’ve been infected with rubella before? When was a. Counsel the patient
your LMP? How frequent were your periods? Did you
check PT? Do you have signs of pregnancy? N/V? - Congratulate on pregnancy
morning sickness? Tender breasts? Are you in a - I understand from the notes that your son is having
stage of having a planned pregnancy? chickenpox. How is he? Is he feeling better? When
- Do you have any other systemic illnesses? Are you exactly did he have the rash? Who diagnosed the
using folic acid? Meds? Pap smear? Blood group? chickenpox? Is the rash becoming dry by now? Have
SADMA? you had chickenpox before? At the moment are you
suffering from fever, rash or any other symptoms?
Management Were you vaccinated against chickenpox. Have you
had regular antenatal checkups up till now? Did you
Immunized; safe to continue have the blood tests? USD? Results?
IgG +, IgM -
pregnancy - Do I have any examination findings?
- How much do you know about chickenpox?
- Chickenpox is a viral infection caused by varicella
zoster virus. It is a very common infection especially in
Pregnant Avoid further contact; can continue school-going kids. From statistics, we know that 80%
patient IgG -, IgM - of pregnant females are found to be immune or
pregnancy and repeat test in 2-3 weeks
(b-hCg +) protected either as a result of exposure in childhood or
from immunization. This immunity is lifelong. If you
have had it before, the risk is minimal for you.
This is the worst scenario with high - I will arrange some blood tests if you have antibodies
chance of fetal damage in 90% in the in your blood. There are two types of antibodies that
1st 8-10 weeks of gestation and this we check: IgG (if + that means you are immune and
IgG -, IgM +
condition is called congenital rubella can continue with pregnancy without any problems);
syndrome with lots of congenital IgM (if + it indicates that you have been recently
malformations. Offer patient referral to exposed to this infection, but let me reassure you that
specialist and discuss termination upon the risk to the baby within the first trimester is only
patient preferences. 0.4% and later on goes up to 2%.
- The period of highest risk both for the baby and for
you is 1 week before and after delivery. Especially for
Immunized; safe to be pregnant
IgG +, IgM - the mother with an active chickenpox infection, there
is a 10% chance of developing certain complications
e,g. encephalitis, pneumonia and hepatitis which can
be fatal.
Pregnant not - If baby is infected, he/she might be born with a
Vaccination and avoid pregnancy till the
patient IgG -, IgM - condition called congenital varicella syndrome
next period
(b-hCg -) where he might have a rash, similar to the chickenpox
rash, limb defects, IUGR, microcephaly, cataracts,
micropthalmia, MR (due to cortical atrophy), muscle
Mild illness: symptomatic control; she and bone defects
IgG -, IgM +
can be pregnant - If IgM+ Æ give Immunoglobulins that can prevent and
reduce severity of disease for mother. It is usually
given via IM injections preferably within 4 days of
Rubella Varicella exposure because efficacy is highest. If you develop
Vaccination No No sx, we will give acyclovir that reduces the severity and
Immunoglobulin No Yes duration of chickenpox.
Termination Yes if IgM (+) Never
26 
 
- Please don’t worry too much because the risk of History
infection and complications is low. I will write the order - Are you in a lot of pain? Would you like some
for you. Please come back tomorrow for review. painkillers? When did it start? How bad is it from 1-
- If IgM is positive then you have to proceed to the 10? What kind of pain? Burning? Stabbing? Shooting?
hospital for review by an obstetrician. Is this the first time? Do you have any problems with
your waterworks? Have you noticed any fever,
GBS In Pregnancy headache, any lumps and bumps in the body
especially in the groin area? Any vaginal discharge?
Case: Mrs. Mary Jones had her last antenatal visit at 37th week What about the ulcers? Did they come with the pain or
and vaginal swab for GBS was done which is positive. She is afterwards?
concerned to hear and wants to know the risks for the baby and - I understand you’re 20 weeks pregnant. How is it so
herself. far? Have you had regular antenatal checkup? Have
you had all the blood tests? Can you feel the baby
Task: kicking? Any bleeding or spotting down below? Any
a. Explain nature of disease and its appropriate tummy pain?
management - Are you in a stable relationship? May I ask how many
b. Answer patient’s questions partners have you had previously? Have you or any of
- Congratulate pregnancy your partners every suffered from an STI? What
- From the notes I understand that you are here to contraceptives were you using before this pregnancy?
discuss your results. Your vaginal swab shows the - How is your general health? When was your last
presence of bacteria called GBS. This is a bug. Before episode of genital ulcers? Did you have fever and
we go ahead, I would like to ask some question. Is it lumps in the body at the time? What treatment was
alright? given by your doctor? Did it help? Any problems?
- Ask for burning sensation in urine, smelly, increased SADMA?
frequency, lower burning pains, change in color of
urine Physical examination
- How is your pregnancy going so far? Any concerns? - General appearance
- Are you allergic to any medications? - Vital signs
- GBS are normal vaginal bacteria in healthy women - Abdomen: Fundal height, tenderness, rash in the
and is found in 18-27% of pregnant women. It will abdomen, blisters/ulcers (dermatomal distribution), lie
cause no harm to you, but we are concerned that if of baby and fetal heart sounds
present during labor, it can harm your baby. 40-50% - Pelvic:
of babies are colonized but only 1% develops o Inspection: redness, discharge, ulcers
neonatal sepsis. Although only 1% gets it, it is a (unilateral – syphilis or bilateral – herpes;
serious infection and carries serious mortality for the weeping/wet –herpes; pus or discharge –
infected baby. superimposed bacterial infection), vesicles,
- There are some risk factors which can exaggerate the warts
risk of infecting your baby: o Groin for tenderness and evidence of
o Preterm delivery lymphadenopathy
o Prolonged rupture of membrane - Urine dipstick and BSL
o Maternal fever >38C during labor
o Previous GBS infection ASHM (Australasian sexual health medicine) and RCOG
- Reassurance Æ don’t worry, it’s good we have picked Differential Diagnosis for Genital Ulcer Disease
up at this stage and we will do our best - Herpes – until proven otherwise
- We will give IV antibiotics (Penicillin 3G initially as LD - Syphilis – single, painless, wet ulcer
then 1.5 gms or erythromycin q4) during labor which is - Allergy/scabies/vulvar (squamous cell carcinoma)
started at least 4 hours before delivery - Varicella – painful vesicles – ulcer with dermatomal
- Baby will be assessed by pediatrician. If completely radiation
healthy, and no risk factors, no antibiotics will be - Donovanosis – not common in Australia;
given. Calymmatobacterium granulomatis)
- Give reading material and red flags arrange for review - Trauma
after 1 week - Lymphogranuloma venereum – Chlamydia
- Chancroid (Haemophilus ducreyi) – painful
Critical Errors:
- Failure to advise mother that it can be serious for the Investigation
baby - FBE, MSU (if indicated), swabs from ulcer to send for
- If you tell neonatal sepsis can be handled easily PCR, antibody testing in the blood specific for herpes,
- Offering antibiotics now oral swabs
- Offer full STD screening, and preferably the partner as
Recurrent Herpes in Pregnancy well

Case: You are a GP and 28-year-old primigravida who is 20 Management


weeks GA comes in complaining of pain and ulcers over the - Most likely you are having recurrent genital herpes. As
vulva over the last 2 days. you know, it is a viral infection that is usually acquired
by sexual contact. This virus stays within the body
Task lifelong even after treatment of the first attack. It lives
a. History within the nerve root. Whenever there is a period of
b. Physical examination stress, like for example, a febrile illness, pregnancy,
c. Management at present and during labor and in females, during periods, this virus becomes
27 
 
activated and causes symptoms (e.g. pain, blisters, ‐ Is my patient hemodynamically stable? How do you
ulcers, fever, swollen glands, and vaginal discharge). feel at the moment? Do you feel dizzy or do you feel
- Unfortunately, this virus cannot be eliminated from the like lying down. I can see from the notes that you’ve
body, but treatment reduces the severity of symptoms, had some bleeding and tummy pain and that you’ve
duration of illness, and prevents spread of infection. noticed some grape-like material passing out from the
- When you had the first infection, you were given oral vagina. From the investigation, you have a condition
antivirals. This time, we will give you a cream called molar pregnancy. This condition can be a
containing acyclovir that you can apply locally. I will serious problem. Unfortunately, there is no fetal tissue
also write for you some topical lignocaine gel to that we can see. I understand that it might be
reduce the pain. You can use warm salt water baths to shocking for you. Is it alright for me to continue?
relieve the pain. It is important to avoid sexual contact ‐ Molar pregnancy occurs when the fetus is not able to
until the ulcers heal completely. form completely. As you know, in a normal pregnancy,
- Wash your hands immediately after touching or the sperm and the egg fuse to form the fetus. This
scratching the area. Take oral analgesics as well. fetus carries equal genetic material from mother and
- During pregnancy, herpes infection can come again. If father. Sometimes, the egg is empty or it is fertilized
it happens within 6 weeks before EDD, we will give by 2 sperms at the same time. The resulting tissue
you oral antivirals. We will refer you to the specialist lacks maternal genes, therefore only the placenta is
obstetrician who might offer you cesarean section formed. This placenta grows and invades/erodes the
because there is a risk that with normal vaginal lining of the womb which causes bleeding. The
delivery, 5% of babies might get the infection and placenta is also responsible for the production of a
develop neonatal herpes. Neonatal herpes can be a hormone called beta-hcg that gives the usual
serious infection for the baby. The baby might be born symptoms of pregnancy such as nausea, vomiting,
prematurely, develop herpes of the eye, meningitis, and breast tenderness. In a molar pregnancy, the
and rarely, can be fatal within 1st 7 days. Some placenta is abnormal and grows massively and it
specialists recommend using continuous acyclovir contains fluid-filled sacs or cysts. The grape-like
starting from 36 weeks until delivery. material that you have noticed is the same sac.
- If insistent on NSVD: Rarely, the placenta starts to grow and invade the
o Avoid fetal scalp pH monitoring, artificial uterus. It travels within the circulation sometimes
rupture of membranes and other invasive reaching the lungs, brain, bones. We then label it as
procedures. invasive mole or choriocarcinoma that can carry
o Acyclovir will be started as soon as labor is serious consequences for you.
established ‐ At the moment, I need to send you to the hospital
o Treat baby with acyclovir after delivery urgently. They will admit you and call the obstetrician.
- If with 1st infection of herpes: risk to the baby is 50%. This pregnancy needs to be removed either by
dilatation and evacuation or by suction curettage. It
DISEASES OF PLACENTA AND MEMBRANES will be done under general anesthesia so you will not
feel any pain.
Molar Pregnancy Counseling ‐ After the procedure, they might decide to give you a
form of chemotherapy (Methotrexate) as some cells
Case: You are a GP and a 30-year-old female comes to from the mole can reach the circulation. We will need
complaining of tummy pain, vaginal bleeding and passing grape- to do serial hcg monitoring every week until it touches
like material with the bleeding. The ultrasound was done that normal level and stays normal for the next 3 samples,
has confirmed molar pregnancy. do monitoring hCg monthly x6 months then annually.
We will also do serial USD every 2 weeks.
Features: ‐ If it remains elevated or persistently highly, we will
‐ Bleeding + passage of grape-like debris need to check for the spread of the disease by doing
‐ May be exagerrage symptoms of pregnancy CT scans of the chest and abdomen. If anything is
(hyperemesis) detected, you will be referred to the cancer specialist.
‐ Uterus large for dates ‐ Once the treatment is completed, you need to avoid
pregnancy for 1 year because the pregnancy
Investigations: hormones can induce recurrence of the cancer.
‐ FBC, blood group and cross-match, beta-hcG, ‐ OCP: Yes.
ultrasound (pelvic: snow-storm appearance), CXR ‐ Regarding your next pregnancy, there is still a very
‐ Suction curette with oxytocin drip high chance that you might have a normal pregnancy,
‐ Hysterectomy if patient has completed family planning but the recurrence rate is higher (1:80 compared to
‐ Register in trophoblastic registry 1:15,000 for general population).
‐ I will refer you to the counselor because you need a
Followup lot of emotional support at this time. It is normal to be
‐ CXR upset after losing a pregnancy.
‐ Weekly serum beta-hcG until zero (8-12 weeks) then
monthly for 12months Oligohydramnios
‐ Avoid pregnancy for 12 months after hcG levels are
normal Case: Your next patient in your GP practice is a 28-year-old
‐ OCP is appropriate primi who works as a nurse in the Renal transplant unit. You
have looked after her pregnancy so far, and all appeared normal
Task up to her last visit 4 weeks ago. When she was 30 weeks AOG
a. Counsel patient regarding current and future she had a SFH of 28cm. Today her SFH is 29 cm and there
management appears to be less amount of liquor.
28 
 
Task: ‐ Do you have any tummy pain? Is your tummy tender?
a. Further relevant history Any bleeding or discharge per vagina? How are your
b. Relevant Physical examination findings and periods? Are they regular? Heavy or normal? Are you
investigation regular with your pap smear? Do you know what your
c. Diagnosis and subsequent management plan blood group is? Any other symptom symptoms like
headache, blurring of vision, polyuria or polyphagia?
Problem list Any chance of exposure to cats or dogs? Any past
a. Recognize and managem oligohydramnios history of uterine fibroids? Did you eat raw meat
b. Management plan and appropriate investigations recently?
c. Relieve patient concern regarding baby safety ‐ SADMA?

Differential diagnosis: Physical examination


Maternal factors: ‐ General appearance
- Wrong dates ‐ Vital signs
- Constitutional: small mother (hx, weight, parity, ethnic ‐ Chest and heart
group) ‐ Abdomen: FH (40cm), lie and presentation, head if
- Medical: HTN, DM, Immunological (SLE) engaged or mobile, FHT/FHR
- Socioeconomic: nutritional factors – anemia ‐ Pelvic exam: inspection and speculum
- Medication usage – steroids, warfarin, anti-epileptic ‐ Signs of edema
- Previous pregnancy with IUGR; FHx: IUGR
- Tobacco and substance misuse Management
‐ I have noticed that your pregnancy is larger than the
Fetal factors: date. There are a number of causes for it. It could be
- Genetic: chromosomal fetal defects wrong date, multiple pregnancy but it is not your case.
- Multiple pregnancy (each child IUGR) It could also be diabetes or abnormalities in the baby
- Fetal infections (TORCH) or certain infections. It may also be due to uterine
- Placental insufficiency – placenta previa, abruption, fibroids. At this stage, I would refer you to the
immunological obstetrician and organize blood tests such as FBE,
blood group, TORCH, BSL, urine microscopy and
Polyhydramnios culture, ultrasound, biophysical profile (AFI >25cm is
diagnostic of polyhydramnios).
Case: Your next patient in a small country town is a 26-year-old ‐ We will also need to do CTG and GTT (even with
Mrs. Jones who is 30-weeks-pregnant. She has recently moved normal GCT).
to this area. Her antenatal care up to now has been taken cared ‐ Can it be risky? Don’t worry, we have picked it up
of by one of your colleage. Midgestation USD is normal. 4 early. You will be seen by the specialist. Having said
weeks ago her fundal height was 26cm, but today, it is 40cm. that, there are some complications like premature
She feels a bit tired and uncomfortable with a large tummy and labor, premature rupture of membrane,
wants you to take over her antenatal care. She recently traveled malpresentation, placental abruption, cord prolapse,
overseas. pregnancy-induced hypertension, and postpartum
hemorrhage. That is why we will monitor you and your
Task baby very closely.
a. Brief history (traveled to NZ, no fever or jaundice, ‐ How will they treat it? If the polyhydramnios is mild
single baby, 18 weeks USD, B+) and asymptomatic, we just do observation. However,
b. Physical Examination (FHT normal, FH 40, cephalic, if it is moderate to severe, and you get SOB, you
head freely mobile, FHT, no tenderness, cervix cannot sit or lie down comfortably, and you are <35
closed) weeks, the specialist might do amnioreduction up to
c. Investigations 500 ml. if the pregnancy is >35, the specialist might do
d. Diagnosis and management induction of labor by artificial rupture of membrane. At
this stage, the specialist might consider giving
Causes indomethacin to reduce urine production.
‐ Wrong dating ‐ Prophylactic steroid decided by specialist.
‐ Multifetal gestation ‐ Red flags: bleeding, discharge, blurring of vision,
‐ GDM SOB, tummy pain, kindly go to the ED of the nearest
‐ Chorioangioma hospital immediately.
‐ Fetal abnormalities (NTD, UGI atresia) ‐ Reading material. Review,
‐ TORCH (CMV and toxoplasmosis)
‐ Fibroids VARIATIONS IN DURATION OF PREGNANCY (PRETERM
AND POSTDATISM)
History
‐ Congratulations on your pregnancy. Is it planned? Preterm Labor
How was it confirmed? Are you regular with your
antenatal checkup? Did you visit your GP before Case: Sarah is a 27 year old female and presents to a district
leaving? Did you receive appropriate vaccinations hospital where you work as HMO in ED. She is 32 weeks
before leaving? Did you have any problems there or pregnant and noticed some pain in the lower abdomen since
on flights? How was your midpregnancy USD? yesterday. She didn't break her water and the baby is kicking
Placenta? Single baby? How were your blood tests? well.
Any FHx of congenital anomalies? How is your baby?
Is he kicking well? Did you maintain a kick chart? Is
this distended tummy giving you any problems like
SOB or day-day lifestyle? Any fever or signs of
infection in the last 3 months.
29 
 
o Nitrazine test or amnisure
Task o Fetal fibronectin (may be FP if had sexual
a. History (spasms that are getting worse, all over, 8/10, intercourse within 24 hours, bimanual
no bleeding, good antenatal checkups, pap smear a examination done)
year ago) o Ultrasound (Abdomen):
b. Physical examination (distressed and in pain, FH o FBE: signs of infections
consistent with GA, cephalic presentation, FHT (+), o ESR/CRP
nontender on palpation, pelvic examination: no o Urine MCS
discharge, bleeding, bulging of BOW, normal cervix, - Management
3cm dilated, 70% effacement) o Pain-relief Panadeine, Pethidine IM 25-
c. Advise on management 100mg, diazepam
o Tocolytic
Preterm labor: o Betamethasone 11._ mg 2 doses 24 hours
- Gestational age <36 weeks, UC q5-10mins x 30 secs apart
in 60 mins, cervix >2.5cm dilated and 75% effaced - Refer to tertiary hospital

Physical examination Premature Rupture of Membranes


- Abdomen: lie, presentation, FH, head is engaged or
floating Case: 32 weeks GA pregnant female presented complaining of
- Pelvic: passage of fluid 2 hours ago. She is 24 years old. She has cone
o Inspection: discharge, bleeding biopsy done for abnormal pap smear and cervical suture is in
o Speculum: discharge, bleeding, cervical os, place. You are a GP in a rural area.
effacement, nitrazine test
o IE: consistency, position, station Task
a. Relevant history
Management b. Examination findings
- CTG, fibronectin and nitrazine test c. Management
- Start tocolytics (nifedipine or salbutamol) -- nifedipine
10mg orally q20 then 20 q4 Focused history:
- Betamethasone 11.4mg IV 24 hours apart - How much (how many pads? Is it soaked)? What is
- Refer to metropolitan hospital. the color (is it clear? associated mucus? Blood?
- Contraindications for tocolytics: Chorioamnionitis, Greenish material? How long? Is there any tummy
cervix >5cm, IUFD, abruptio placenta pain? Contractions? Any other associated waterwork
problems (e.g. increased frequency of urination?), any
Preterm Labor vaginal secretions? Fever? Hot flushes? Dizziness?
Vomiting? Heart-racing? Do you still feel the baby
Case: Linda aged 34 years presents to a country hospital where kicking?
you are working as year 1 HMO. Linda is 33 weeks pregnant - Pregnancy: is this the first pregnancy? Previous
and since this morning she had noticed few contractions and antenatal checkup? Any abnormalities on usd ? What
cramps in the lower abdomen. There is no vaginal discharge is your blood group? Previous deliveries and previous
and baby is moving well. Up till now, pregnancy has progressed gynecological problems?
well and all investigations have been normal - Pap smear: what was the cause detected. For the
cone biopsy, when and where did you have this done?
Task - PMHx: hypertension? DM? meds? SADMA?
a. History (started contractions a few hours ago, - How far do you live from here? Who can care for you
occurring every 5 minutes) if we decide to transfer you to a tertiary hospital?
b. Physical examination (3cm open, 50% effaced)
c. Probable diagnosis and management Physical examination:
- General appearance: pallor, anxious, BMI
Features - Vital signs: temperature, BP (postural drop), pulse, RR
‐ Gestational period is less than 36 completed weeks - Urine dipstick
‐ Uterine contractions preferably recorded on tocograph - Rapidly I’d like to check chest and heart
occur every 5-10 minutes, last for at least 30 seconds - Focus on the abdomen: general look abdomen. I’d like
and persist for at least 60 minutes to start with superficial palpation (tenderness means
‐ Cervix is more than 2.5cm dilated and more than 50- chorioamnionitis), feel fetal parts, check fetal position,
75% effaced gestational age, fetal heart sounds
‐ Contraindications to tocolytics: APH, effacement - With the consent of the page, I’d like to go for pelvic
>75%, cervical dilatation >5cm examination and ask consent for swabs. I’d like to
inspect for fluids. Can I have a description of the fluid?
Physical Examination Is it clear? Does it smell? Blood/mucus or other
‐ General appearance discharge. With complete aseptic condition, I’d like to
‐ Vital signs perform speculum examination (fluid at fornices). I’d
‐ Abdomen: FH, presentation, lie, contractions, like to collect cervovaginal swabs for MCS and collect
engagement low vaginal and anorectal swabs for GBS. I’d like to
‐ Pelvic examination: inspection and speculum confirm the diagnosis of PRM by nitrozine/lithmus test.
examination; No IE done Remove cervical suture and send for MCS!

Diagnosis and Management Investigations and Management


- Investigations - I’d like to arrange some investigations as soon as
o Cervical swab looking for pathogens possible: FBE, U/E, LFTs, CRP, CTG, USG, and refer
patient to hospital.
30 
 
- Give her erythromycin 250 mg QD for 7 days and - Chest and heart
betamethasone 2 injections 24 hours apart. - Abdomen: FH, lie of the baby, presenting part,
- If there is no evidence of infection or no engagement, FHT, tenderness over the abdomen,
contraindication for tocolysis: nifedipine/salbutamol contractions
- Organize admission at local hospital and tertiary - Pelvic examination:
hospital by nets (neonatal emergency transfer o Inspection and speculum: discharge,
service). bleeding, presence of show,
nitrazine/lithmus test to detect amniotic fluid
Postdated Pregnancy in the vagina,
o Bimanual: position, size, and effacement of
Case: You are a GP and a 41-weeks primigravida comes to cervix
your clinic because she is worried when she will deliver. - Urine dipstick and BSL

Task Management
a. History - It seems like your pregnancy is advancing towards
b. Physical examination (FH 39, head just, engaged, lie postdatism. 5-10% of normal pregnancies can go
is longitudinal, FHT normal, speculum: no discharge, beyond 42 weeks something we call as postdated
bleeding or show; closed, long, posterior, no bulging pregnancy. You are still within the normal range so
of membranes) please don’t worry. However, I want you to be aware
c. Management of certain risks associated with postdated pregnancy,
for example, placental insufficiency, meconium
Definition: aspiration, fetal distress, difficult delivery with higher
- RWH: >41 weeks + 6 days risk of undergoing cesarean section.
- LJ: 40 + 2 completed weeks - What we need to do is monitor you very closely to
prevent postdated pregnancy. Starting from now, we
Risks: will do CTGs 2x a week to assess fetal distress. We
- Placental insufficiency will also do ultrasound once a week to check the
- Meconium aspiration baby’s growth (BPS). We will also check the AFI.
- Fetal asphyxia Also, I will recommend a Doppler study of the
- Difficult deliveries (problems with molding) umbilical cord to check the flow of blood to the baby.
- Increased risk of operative deliveries At the end of all these tests, you will need to see the
- Increased risk of labor induction specialist obstetrician. They might give you options
- Dystocia which include elective induction of labor with the help
- 4x increased risk of stillbirths of prostaglandin tablets that are inserted within the
- 3x increased risk of neonatal death vagina to initiate contractions. The second option
- 10x increased risk of neonatal seizures (within 1st 48 would be to continue the pregnancy but with regular
hours of life) CTGs, USD and Doppler studies. The third option is
elective CS that carries minimal risk in safe hands.
History The decision is yours. Please bring your partner for
- Is this a planned pregnancy? Congratulations! How is the next consultation so we can discuss it together.
the pregnancy so far? Can you tell me how your - Meanwhile please look out for signs of labor which
pregnancy was confirmed? Have you had regular includes bleeding, discharge, leaking of fluid,
antenatal visits? All blood tests? What were the continuous/intermittent back or tummy pain.
results? Any problems? Did you have the sweet drink
test? Was it alright? When was your last ultrasound? ABNORMAL PRESENTATIONS
What was the result? Is it a single baby? Weight? Breech Presentation
Placenta? What was the expected date of delivery on
that ultrasound? Case: You are a GP and a 25 year-old primagravida with breech
- How’s your general health? Any past history of presentation at 32 weeks’ GA came in for consultation.
diabetes, high blood pressure? Currently do you have
any symptoms of headache, blurred vision, or swelling Task
of the legs? Any bleeding or discharge from down a. History
below? Any tummy pain? backache? Is the baby b. Physical Examination (lower pole of the uterus is a
kicking alright? Have you counted how many times in soft, smooth and with a rounded mass that bounces
how many hours? Do you have kick chart with you (10 between the fingers, position of heart sound is above
in 12 hours)? the umbilicus)
- FHx of postdated deliveries? Big babies? c. Diagnosis and management
- Have you had any gynecological surgeries or
procedures (adhesions)? What is your blood group? Causes of Breech
When was your last pap smear? Have you been - Maternal
vaccinated against gardasil? SADMA? o Polyhydramnios
- Do you have enough support at home? Any o Uterine abnormalities (bicornuate, septate)
problems? How far do you live from the hospital? Is o Placental abnormalities (previa)
there anyone who can drive you in case of an o Multiparity
emergency? o Contracted maternal pelvis
o SOLs (fibroids)
Physical examination - Fetal
- General appearance o Prematurity
- Vital signs: BMI and height o Fetal anomalies (neurological,
hydrocephalus, anencephaly)
31 
 
o Multiple pregnancy of labor the specialist will do cesarean delivery. If
o Fetal death footling, then do Cesarean delivery. We can reduce
o Short umbilical cord the risk of complications by 50% if we choose elective
cesarean section at 39th week. If you do decide to go
History on a trial with vaginal delivery, we will still do our best
- Is it a planned pregnancy? Congratulations on your to monitor you and your baby by doing regular CTG
pregnancy. How is the pregnancy so far? Are you and USD. It will be done in a tertiary hospital in the
regular with your antenatal checkups? How were your care of an experienced obstetrician.
tests? Ultrasound? Was it a single baby? What was - Is it a serious condition? Not really, but it makes
the position of the placenta? Amniotic fluid? Sweet NSVD difficult but not impossible. However, you are
drink test? Blood group? Have you taken folic acid? still at 32 weeks and there is a high chance that your
- Is the baby kicking normally? Are you maintaining a baby will still change its presentation
kick chart? Do you have any headache, dizziness, - Reading material. Referral.
BOV or leg swelling? Do you go to washroom quite - Red Flags: bleeding, tummy pain, blurring of vision
often? Do you drink a lot of water? Does your tummy
feel more distended than usual? Any vaginal bleeding, Transverse lie in Multigravida
discharge, tummy pain?
- How are your periods before? Were they very heavy? Case: You are an HMO working at a district hospital and a 38-
Were the cycles regular? Were you ever been weeks multigravida who lives 80 km from the tertiary hospital
diagnosed with fibroids or any other abnormality? was found that the baby had a transverse lie.

Physical examination Task


- General appearance a. Relevant history
- Vital signs especially BP b. Physical Examination (FH does not correspond to
- Abdominal: FH, bell shaped, lower pole of uterus is gestational, uterus is ovoid, fundus is empty and head
occupied by a soft, smooth rounded mass that lies in one of the flanks, no tenderness, FHT normal)
bounces between the fingers, FHS usually loudest c. Management
above the umbilicus, tenderness, FHT
- Pelvic exam/Speculum: discharge, os Case 2: Julia aged 35 years presents to your surgery for routine
- Leg edema antenatal checkup as advised by you last week. She is 38
- Urine dipstick and BSL weeks pregnant and till now her pregnancy has been
progressing well. On routine questioning she tells you that today
Diagnosis and Management she had uncomfortable feeling in her flanks and tense feeling
- Your baby’s position is breech. Normally, the baby’s but no other associated symptoms. She had normal USD at 18
head is down and the bottom is up. In your case, the weeks and other blood tests performed during pregnancy. This
baby’s butt/bottom is presenting down. is Julia’s 2nd pregnancy. She had one abortion when she was 32
- There are three kinds of breech years old. Julia lives with her partner in an apartment close to
o Frank: hips flexed and knees extended your surgery. She stopped smoking when she became pregnant
o Complete: hips and knees flexed but is still having a glass of wine here and there.
o Single or Double Footling: one of both legs
are completely extended. Differential Diagnosis
- First of all we need to do USD to confirm the diagnosis - Labor
and exclude the causes of breech and to make sure - Placental abruption
that the baby is fine. In most of the cases of breech - UTI/Pyelonephritis
near term or at the time of delivery, baby takes the
normal cephalic presentation. If not, with your Task
consent, the specialist obstetrician will try to turn your a. Focused history (tense on both sides, no radiation,
baby in the normal position by gently pressing the baby kicking well)
tummy. Do not worry. It is a painless procedure and it b. Examination and investigation findings from examiner
is done in a tertiary hospital. (FH 36cm, transverse lie, FHT+, no tenderness, no
- There are some complications which can be possible: discharge and os is closed on pelvic exam; urine
o Failure Æbaby can come back to breech dipstick +1, BSL 4.6mmol/L)
presentation c. Probable diagnosis and management advise
o Premature labor
o Bleeding Risk factors
o Fetal distress if umbilical cord goes to the - Multiparity
neck (0.5%) - Lax uterus Æ most common cause
- Contraindications of ECV - Previous cesarean section
o Oligohydramnios - Polyhydramnios
o Antepartum hemorrhage (placenta previa) - Placenta previa
o Multiple pregnancy - Uterine malformation
o Uterine structural abnormality - Small pelvis
o Fetal abnormalities
- What about the delivery? If the breech is complete or History
frank, the specialist can offer a trial vaginal delivery - I found out from the notes that your baby’s position is
but there are some risk to vaginal delivery which different from the expected. Can I ask a few more
includes fetal distress because of cord prolapse, hip or questions? How is your pregnancy so far? Any
shoulder dislocation, fracture of humerus, femur or abdominal pain/contractions or water leakage?
clavicle and asphyxia. If these develop during the trial Any vaginal bleeding? Do you feel the baby is
32 
 
kicking? Are you maintaining a kick chart? Did you
have regular antenatal checkups? How were the blood If patient not at term:
tests? What about the midpregnancy USD? Do you - Gentle cephalic version maybe attempted at 36-38
remember what the doctor said about the baby and weeks if patient consents. If successful, may induce
placenta (Single baby and position of placenta)? labor and go to vaginal delivery.
Sweet drink test? Did you have a low vaginal swab - Advise to report to hospital immediately when labor
done (GBS)? Do you know your blood group? Do you starts or if social conditions are unfavorable
feel your tummy is more distended than it should be? - Elective cesarean section
Did you have any infection during pregnancy?
- How many children did you have? What type of LABOUR AND DELIVERY
delivery (2 normal and 1 CS)? Were they big babies?
Complications? Reduced Fetal Movements
- How is your general health? Ever been diagnosed
with fibroids or any uterine problems? FHx of Case: You next patient in a country clinic is a 38 weeks
malpresentations? SADMA? gestational age lady with no fetal movements in the past 12
hours.
Physical Examination
- General appearance Task
- Vital signs a. History
- Chest and Lungs b. Physical examination
- Abdomen: FH, FHT, broad transverse uterus with a c. Diagnosis and management
firm ballotable round head in one iliac fossa and a
softer mass in the other, assess AFI (very subjective
Æ abdomen tense and hard to palpate fetal parts)
- Pelvic: Inspection and Speculum: discharge, blood, No fetal movement
cervical os, nitrazine test, NO PV

Management IUFD Hypoxia Normal


- Your baby has a transverse lie which is different from
the normal or expected position during term. It is
uncommon. It occurs in 0.5 to 1% of women. There
are several reasons for that: placenta previa (placenta Doppler
lying in the way of the baby and prevents the baby
from turning to normal position). We will need to do an
ultrasound to rule out this condition and
polyhydramnios (or increased amniotic fluid in the (-) FHT (+) FHT CTG
baby) which is also another cause of this abnormal
position. The commonest reason is a relatively large
and lax uterus after previous pregnancies. For now I
will organize an ultrasound and CTG for you and History
arrange for an obstetric assessment. - I understand you have come to see me because you
- There are two options to manage your pregnancy. haven’t felt your baby’s movement for the past 12
Whichever you choose, you will need to stay here until hours. Is it the first time? Have you noticed that your
delivery (Do we have a cesarean section unit in this baby is moving less in the last few days? Is it your first
hospital? If not, then transfer to tertiary hospital pregnancy? Did you have regular antenatal
because labor may commence soon). checkups? How was your USD? Lab tests? Sweet
- Why do I have to this stay in this hospital? The reason drink test? Did you have a low vaginal swab? How
for that is if labor starts and the baby has transverse was your BP throughout pregnancy? Do you know
lie, it can quickly progress to obstructed labor which your blood group? Did you have any infections or
can lead to uterine rupture. Another risk is cord febrile episodes? Do you have a fever? Headache or
prolapse (cord can slip into vagina) after membranes abdominal pain? How’s your waterworks? Have you
rupture and it is a life-threatening condition for the noticed any unusual vaginal discharge or bleeding?
baby. - General medical health? SADMA?
- If you agree, after ultrasound, an obstetrician can - Where do you live and who you do you live with? Do
rotate the baby to normal position. We call this you have any family members or close friends with
external cephalic version. If it is successful and your you today?
cervix is favorable, OB will rupture the membrane and
you will go to normal vaginal delivery. External Physical examination
cephalic version is quite a safe procedure. However, - General appearance
approximately 0.5% requires immediate cesarean - Vital signs
section due to fetal distress or vaginal bleeding - Abdomen:
(abruption). Your second option is elective cesarean o FH: 20-36 weeks = 32 +2, 36-40 = +3cm,
delivery. Regardless of your decision, we are here to >40 weeks = 4cm,
help you. o uterus (soft, tender, contractions)
- Let me reassure you that you and your baby will be o Speculum: nitrazine
closely monitored by the specialist. I will call the o Per vagina: check the cervix (4cm, posterior,
ambulance for transfer. closed, station -3)
33 
 
Diagnosis Any miscarriages? Do you have any history of medical
- I can hear your baby’s heart sound and it is within conditions like diabetes, or high blood pressure? Any
normal range. There are two possible explanations for surgical conditions for example gynecological
your presentation. The baby’s activity could be surgeries like cervical biopsies? How are you feeling
different throughout the day and absence of baby’s at the moment? Are you sleeping well? Appetite?
movement could be due to rest or sleep. However, we Bowel habits? Waterworks? Blood group? SADMA?
need to exclude the other possible cause which is FHx of operative/difficult deliveries or fetal
fetal distress due to hypoxia or lack of oxygen to the abnormalities? Do you know your height and weight?
fetus which makes your baby quiet. For this reason, I Do you have any family or friends to support you in
need to send you to the hospital where CTG will be your husband’s absence? How far do you live from the
performed. It’s a simple and safe procedure. Two hospital? Is there anyone who can drive you in case of
sensors will be placed on your abdomen to record emergency?
baby’s heart rate, uterine contractions and fetal - As you know, elective induction of labor is a big
movements. You will also be assessed by an decision by itself. As a medical health practitioner,
obstetrician. If the CTG pattern is normal, you might there are certain indications where induction is
have an ultrasound to assess the amount of amniotic necessary. This includes pregnancy extending beyond
fluid around the baby. If everything is fine, you might 42 weeks, chronic kidney or liver disease in the mom,
go home after that and the doctor will explain a kick very small baby, and problems with placenta. There
chart for you. If the CTG pattern is suspicious, the also some contraindications which will make it
doctor will most likely discuss induction of labor with impossible for us to induce labor like if the baby is too
you. If the CTG is abnormal, you might need to have big to pass through mother’s pelvis, or if the baby
an urgent cesarean section. You made a right develops any kind of stress because of insufficient
decision to come and see me today. oxygen supply. Sometimes, we avoid induction in
those who have had a previous surgical procedure to
Elective Induction of Labor the tummy.
- There are some risks associated with elective
Case: You are a GP and a 34-weeks pregnant female comes to induction which includes high chance of bleeding from
you asking for early induction of labor at around 37 weeks the womb, risk of rupture of the womb, and because
because she wants her husband to attend the delivery and he’s the baby is delivered before term, he might suffer from
going on a business trip in about 4 weeks’ time. consequences of prematurity. You need to know that
not all inductions end up having NSVD. Sometimes,
Task we need to use instrumental deliveries such as
a. Counsel patient and answer her questions forceps or vacuum, or ultimately cesarean section to
deliver the baby.
Indications for Induction of Labor - Induction of labor is usually done in the hospital where
- Maternal cesarean section facilities are available. You will be
o Postdated pregnancy seen by the obstetrician and they will assess the
o PROM baby’s position and size. If there are no
o IUGR/Oligohydramnios contraindications, they might go through it. Please
o Maternal Diabetes understand that we prefer to leave the baby inside the
o Pulmonary Embolism (shunting of oxygen) womb until Mother Nature decides for delivery. It is
- Fetal important for the baby’s growth and maturity. If you
o Fetal abnormalities wish, I can arrange a meeting with the obstetrician. It
o Placental insufficiency would be preferable if you could bring your partner to
o IUGR the meeting. If you like I can give you a certificate for
you husband which he can use to delay his trip.
Contraindications - Reading material. Review. Referral.
- Fetal distress - In the end, the decision is up to you. You can decide
- Placenta Previa with your partner after you discuss with the
- Malpresentation obstetrician.
- CPD
- Previous CS Pain Relief During Labor

Risk of Induction of labor Case: You are a GP and a 30-weeks pregnant primigravida
- Fetal Distress came to you asking about pain relief in labor.
- Postpartum hemorrhage
- High risk of operative delivery Task
- Uterine rupture a. Counsel accordingly

Counseling ‐ Congratulations! How is your pregnancy so far? I


- Congratulations. Please tell me why you want to have understand from the notes that you want to know
an early induction. Is there any way that your husband about pain relief. Do you know why there is labor
can delay his trip? pain? Actually, there is contraction of the uterus,
- How is your pregnancy so far? Any bleeding, dilatation of the cervix, and distention of pelvic tissues
discharge, tummy pain, headache, visual problems, as well as pressure in certain organs.
swelling? Did you have regular antenatal checks? Did ‐ There are many methods of pain relief including non-
you have all the blood tests especially the sweet test? pharmacological and pharmacological methods.
When was your last USD? What was the result? Do ‐ Regarding non-pharmacological methods:
you feel the baby kicking? Is this your first pregnancy?
34 
 
o Adjusting the position brings comfort to the ‐ Passenger:
mom (e.g. kneeling down or standing o Malposition especially occipitoposterior
upright) position
o Hot or cold bags applied to the tummy, back o Fetal macrosomia (>95% percentile)
and perineum o Coil abnormalties (short coil/cord coil)
o Relaxation or breathing techniques with or
without a massage to encourage the mother History
to tolerate the pain and reduce the anxiety. ‐ How are you doing at the moment? I understand you
o Transcutaneous electrode nerve stimulation are 40 weeks pregnant. Any problems like bleeding,
(TENS)Æ we place 2 electrodes parallel to discharge, or leaking of fluid? Tummy pain?
your backbone. It is helpful by interrupting contractions? Headaches or visual problems? Have
the transmission of pain to the nerves. The you noticed swelling anywhere in the body? How has
frequency of the current can be modified your pregnancy been so far? Have you had regular
accordingly. antenatal checks, blood tests, and ultrasound? When
o Hydrotherapy departments within the was your last USG? What was the result? Do you
hospital Æ hot showers to feel better remember the baby’s weight from the USG? What
o Hypnotherapy about the placenta? Was it normally located? How
o Injection of sterile water in lower back about the amount of fluid around the baby? Did you
‐ Pharmacological have a sweet test (16-28)? What was the result? Is
o Inhalation of nitrous oxide gas which gives this your first pregnancy? Any previous miscarriages?
pain relief for 20-30 secs. Patient can ‐ If not previous pregnancy: what was previous weight?
administer the gas by herself through a Any complications? Mode of delivery?
mask during a contraction. The machine is ‐ Any medical or surgical history? How is your general
called entonox which contains a mixture of health? Do you know your weight or BMI? May I ask
NO2 and oxygen in a 50:50. This procedure how tall you are? Have you ever been diagnosed with
is very safe for both mom and the baby. any problems like fibroids or growth within the
o IM Pethidine injections Æ gives pain relief ovaries?
for at least 2-3 hours. Effect comes rapidly ‐ SADMA? Gardasil? Blood group? Trauma to pelvic
within 15 minutes. It is commonly used but it bones? FHx of difficult deliveries?
does have side effects (e.g. acidity and
reflux symptoms, drowsiness, and Physical examination
respiratory depression in the baby) Æ we ‐ General appearance
will adjust the dose according to side effects ‐ Vital signs: BMI
o Epidural analgesia Æ gives complete pain ‐ Abdomen: FH, lie, presentation, FHT, engagement
relief in around 95% of patients; usually (>36 weeks/floating), amount of liquor
given by anesthetist into lower back ‐ Pelvic exam:
protecting the spine. Sometimes, patient- o Inspection: bleeding, discharge, show,
controlled epidural analgesia is given. There leaking of fluid
are some side effects like headache, o Speculum: cervical os, discharge, bleeding,
dizziness, and shivering. Rarely, leakage of bulging of membranes, nitrazine test
spinal fluid (dural tap). It has been noticed (leakage of amniotic fluid)
that the use of epidural analgesia during the o Bimanual examination: tenderness, position
2nd stage of labor leads to higher risk of of cervix, length, mass
operative delivery for the patient
o IV pethidine analgesia Æ reserved for Investigations
patients after cesarean section ‐ Ultrasound: look for fetal wellbeing, AFI, additional
masses or abnormalities
High Mobile Head At Term Management
‐ From history and examination, it seems like there is
Case: You are a GP and a 40 weeks primigravida is referred to nothing that might be preventing the baby’s head from
you by a nurse because the baby’s head is still 5cm above the engaging within the pelvic bone. It could be a normal
pubic bone. phenomenon where the head may go down within the
next few days. You will then have the signs of labor
Task which are abdominal contractions, leakage of water, +
a. History small amounts of bleeding
b. Physical examination ‐ What I want you to do is:
c. Discuss possible causes and management o Keep a daily kick chart for the baby’s
movements (at least 10 in 12 hours)
Differential Diagnosis o I will write a request for ultrasound with
‐ Passage: Doppler
o CPD – depends on age, nutritional status, o Come for CTGs 2x/week until 42 weeks Æ
type of pelvis; more common in if during the 42nd week, condition remains
underdeveloped countries; risks involve the same, the specialist might decide to
obstructed labor, shoulder dystocia, admit you for possible induction of labor with
increased risk of CS in primigravida, higher the help of artificial rupture of membranes
maternal morbidity and mortality, difficulty and the use of a vaginal gel (prostacyclin).
with subsequent pregnancies
o Fibroids/Ovarian tumors
o Placenta Previa
35 
 
‐ During your visit with the specialist, she might decide o If mild moulding not concerned
to do a pelvic examination to assess for possible CPD o If severe moulding think of obstructive
where the mother’s pelvis is not suitable for the baby’s labour.
head to engage into. It is done with the help of a
bimanual examination. Sometimes, the specialist Management
might do an Xray or a CT scan in doubtful cases. It ‐ Margaret I know you have been in labour for the past
also depends on the mother’s height. If during 8 hours. Unfortunately your last 4 hours are not very
pelvimetry the specialist thinks that your pelvis is efficient. You are at active phase of 1st stage labour.
insufficient for NSVD, they may offer you a trial of Cervix should dilate more rapidly. Expected rate is
labor, possibly followed by cesarean section. 1cm per hour. The most likely cause for failure for
‐ Pelvimetry: AP <13, Oblique < 12, and tranverse cervix to dilate is inefficient uterine contractions.
diameter <11 Æ contracted pelvis ‐ What could be done? In this case management is
‐ Referral and review artificial rupture of the membranes. With your consent
we will break your water. (Empty the uterus easy for
Prolonged First Stage of Labour uterine to contract). +/- Oxytocin or 1- 2hours of
observation.
Case: A 25-year-old primigravida has been admitted for labour 4 ‐ We will monitor progress of labour closely.
hours ago. At that time vaginal examination showed cervix was Contractions will be assessed every 15-20min. Rate of
effaced 4cm dilated. 4 hours later cervix is 5 cm dilated oxytocin infusion will be slowly increased and
adjusted. You will be on continuous CTG monitoring
Task once oxytocin is given. If in 4 hours the cervix fails to
a. History dilate by >4cm we will consider C-section. Abnormal
b. Physical examination CTG pattern or signs of obstruction will also be an
c. Management indication for C-section. Otherwise normal vaginal
delivery is possible once contractions are efficient. I
Features: Expected speed will also organize pain relief for you.
‐ Latent phase: 6hours up to 4cm
‐ Active phase: 1cm per hour. Up to 6 hours. Meconium Stained Liquor
‐ In this case now the expected speed: At least 3
contraction in 10 mins lasting 30-90sec Case: You are in ED in a district hospital where facilities for C-
section and new born resuscitation are available. Your next
History patient Mrs. Brown, is a 32-year-old G2P2L1 10days postdated.
‐ Hello Margaret. How are you? How long have you She presents with a history of leaking water which is green in
been having contractions (8 hours ago)? How often do colour. The pregnancy was uneventful, except for 36 weeks
they come (had 2 contractions in 10 min)? How long group B streptococcus positive.
do they last (30seconds)? Are they painful? 0-10? Did
you have something for the pain? Did it work? We can Task
adjust your pain relief in a few minutes. Have you a. Take history
notice water leakage or bleeding (No)? b. Physical examination (Per abdomen: cephalic
‐ Just a few questions regarding your pregnancy: Did position, engaged , FHS: normal OS dilated 2 cm, and
you have regular check-ups? Any problems with your well effaced; CTG: normal)
blood test? Mid pregnancy ultrasound? Sweet drink c. Management
test (big baby)? GBS swab? Result? Do you know
your blood group? History
‐ What is your and your partner’s height (big husband ‐ I understand it’s your second pregnancy. Could you
and small wife)? Are you generally healthy? PMHx: tell me more about it? Any pain? Any bloody
DM? Surgeries? Pelvis bone fracture? Ever been discharge? Is the baby kicking as usual? From your
diagnosed with ovarian cyst or fibroid (occlude the notes your 36 weeks bug test was positive. The doctor
pelvis)? must have informed you. We will take care of that. Are
‐ If multi: How big was the baby? What was before? you allergic to any med? What’s your blood group? If
the previous pregnancy was normal?
Physical Examination: ‐
‐ General Appearance: Is it just a delayed of 1st stage Physical Examination:
labour or obstructive labour ‐ General Appearance
‐ Vitals: tachycardia in obstructive labour ‐ Vital signs
‐ Abdomen: palpate the uterus and assess frequency ‐ Chest and heart
and length of contractions, lie and presentation. How ‐ Abdomen: Size of the uterus, Lie of the fetus
much of fetal head palpable per abdomen (5 fingers (longitudinal or transvers), presentation, head if mobile
palpable head above the pelvic brim. 0 fingers means or engaged. FHS
the head is already in the pelvis)? FHR? ‐ Pelvic: Inspection for discharge, blood; per speculum:
‐ Pelvic: Per Vaginal examination: effacement dilated? any discharge? Dilated? Effacement? Membrane
How much? Is membranes intact or not (NO)? The ruptured? Cord prolapse?
position of the head: Try to find the fontanelle: Anterior
Fontanelle: diamond shape. Occipital bone: posterior Diagnosis and Management
fontanelle. Occipital anterior/ occipital ‐ Mrs Brown you are postdated by 10 days, and you
posterior/transverse position/oblique. Stations: relation have rupture of membranes. We have to admit you.
to Ischial spine -5 to +5. Moulding and caput? Overlap We will do an ultrasound and CTG (continuous). Baby
of the suture line. looks fine now, if these two tests are good labor will be
induced. You will be taken cared of by the OB
36 
 
registrar. We will monitor the progress of the labour distress due to lack of oxygen. That is why we need to
and baby with CTG. If it progresses normally, we will monitor your baby closely. I will organize
allow you to have normal labour, but if not or if the cardiotocograph for you. CTG is a safe, non-invasive
baby isn’t well we will have to intervene, and use method commonly used during pregnancy and labor.
instruments to facilitate labour or you might need an We will place 2 sensors in your abdomen to record
emergency C-section if the baby goes into distress. baby’s movements, heart rate and uterine
Meconium stained liquor is quite common in postdated contractions. CTG will help us to assess your baby’s
pregnancy. Baby passes stool in the amniotic fluid and wellbeing.
it turns green. If CTG is normal we don’t need to worry ‐ If CTG is normal and progression of labor is good, you
about the meconium. A pediatrician will be present will still be able to have vaginal delivery. If CTG shows
during the labour. They will suction the nose and small abnormalities, we will perform fetal scalp blood
mouth and remove the meconium. Then they will sampling to assess acidosis. If present, we will need
cover the baby, check the APGAR score. If the baby is to perform emergency CS because this is a sign of
in distress they might consider giving your baby a fetal distress.
stomach wash. ‐ If your baby shows signs of distress in the 2nd stage of
‐ For the GBS infection: From the onset of labour IV labor (after full cervical dilatation). A pediatrician will
benzyl penicillin, 1.2g first dose at admission and then be present at your delivery. After birth, we will use
600mg 4-6hourly. suction to clear the baby’s mouth and nose to prevent
particles of meconium to descend to the lungs.
Case (Condition 125): Your patient is a 25yo primigravida who is o If CTG abnormal Æ oxygen to mom, stop
in early labour at 41 weeks of gestation. She is in the local oxytocin, left lateral position Æ check
district hospital where you are attending as a general monitor Æ if still abnormal Æ fetal scalp
practitioner. The hospital has good facilities but a consultant blood sampling to check pH (<7.2) and
obstetrician is not available. Pelvic examination 30 minutes ago lactate (>4.2) Æ emergency cesarean
showed the cervix was 3cm dilated, well effaced, and well section
applied to the presenting part. The cephalic presentation was ‐ If hospital doesn’t have facilities for emergency CS,
position left occiptotransverse (LOT), at zero station, with no then transfer patient to tertiary hospital.
caput or moulding evident. The membranes were still intact and
allowed to remain so. Spontaneous rupture of the membranes POSTPARTUM
then occurred and revealed profuse, thick meconium-stained
liquor. The pregnancy had been uneventful to date, and blood Postpartum Hemorrhage due to Endometritis
pressure and urine testing have been normal in labour. The fetal Case: A 30-year-old lady comes to GP clinic had her second
heart rate, as defined using auscultation, has been between 130 baby 10 days ago. Now she comes of complaining increase
and 140/min. bleeding per vagina

Task Task
a. Take any further relevant history you require. a. History (6pads fully soaked with clots. Slight lower
b. Ask the examiner about relevant findings likely to be abdominal pain. Full term vaginal delivery no
evident on general and obstetric examination complications. No fibroids. Skin delicate and easily
c. Advise the patient of the diagnosis and subsequent bruised. No breast tenderness, no swelling or legs or
management during and after delivery. discoloration.)
b. Physical Examination findings (GA: Well. A bit pale,
History V/S: BP: low but within normal range. Tachycardia,
‐ When your water break? Was it green in color? How Neck is normal., Abdominal examination: No rigidity
long have you been in labor/When did the contraction but mild tenderness in the lower abdomen, Uterus 14
start? How often is your contraction? How long does it weeks in size, Pelvic examination: No laceration, no
last? Do you feel movements of the baby? Is your due hematoma, Per speculum: Bleeding (+), OS: 2cm,
date a week ago? Is that correct? I know your Bimanual examination: No cervical excitation, 2
pregnancy has been uneventful, any problems with fingers above the public symphysis)
blood tests, midpregnancy USD, or GBS swab? Do c. Investigations
you know your blood group? I know you’re a bit d. Diagnosis and Differential Diagnosis
overdue, have you had an USD and CTG last week? e. Management
‐ Are you generally healthy?
Differential Diagnosis
Physical Examination - Retained placenta
‐ General appearance - Bleeding disorder
‐ Vital signs every 2 hours - Endometritis
‐ Abdomen: FH, fetal lie and presentation, uterine - DIC
contractions, - Trauma
‐ Pelvic examination: cervix, effacement, dilatation,
presence of membranes, presence of cord loop,
station, position of fetal head, signs of caput/moulding History:
- Is my patient is hemodynamically stable? If no:
Diagnosis and Management DRABC
‐ The baby has passed meconium which is the baby’s - Bleeding questions: When did it start? How much?
first stool. That is why your water looks green. It is How many pads are you changing? Are they
common and often normal in post-term labor. completely soaked? Are there clots? Any smell? Is it
However, it can also be an indirect sign of fetal bright red or dark bleeding? Any bleeding from
anywhere else in the body? (DIC) Do you have any
37 
 
fever? N/V? tummy pain? Any dizziness? SOB? Chest Postpartum Pyrexia
pain? Any vaginal discharge? How about your water
work? Dysuria? Frequency? How’s the baby? How are Case: You are a GP and a 29-year-old female had a normal
you coping? vaginal delivery 3 weeks ago. She had a baby boy who is
- Pregnancy Questions: Was the pregnancy normal? healthy and doing well. Patient is complaining of fever and
Any complications during pregnancy? Is this your first shivering.
baby? Was it a normal full term vaginal delivery? How
long was the labour? Was it a normal or complicated Task
labour? Any PROM? Did you have episiotomy? Were a. History (fever since 2 days ago, decreased appetite,
there any instrumental or other assistant method used breastfeeding, NSVD, abdominal pain (+),
during delivery? Was the 3rd stage of labour b. PE: pulse: 106, T: 38.6; mild tenderness over lower
complete? Was the placenta completely removed? umbilicus, uterus involuted, no mass or tenderness;
Was there any complication after delivery? Were you dipstick and BSL N; pelvic: no clots, discharge,
discharged from the hospital early? Any bleeding or episiotomy scar healing, no mass and tenderness;
clotting problem? Were you on any medications? Do fissured nipple, cracked and inflamed
you have any chronic condition? SADMA? c. Management

Physical Examination Mnemonic: After Uni, Every Woman Should Marry


- GA: any signs of dehydration?
- V/S: BP: 115/65 Temp: febrile RR:20 sat: normal Atelectasis (0-1)
- Breast examination UTI (1-2)
- Heart and lung Endometritis (2-3)
- Abdomen: Not distended, no signs of peritonitis, bowel Wound infection (3-5)
sound heard. Slight tenderness in the suprapubic Septic thrombophlebitis (5-7)
area. Renal angle tenderness (-) Mastitis (7-21)
- Pelvic examination:
o Inspection: Any laceration, Any hematoma, Breast abscess: can happen anytime
Any episiotomy cut infected bleeding any
discharge. Organism:
o Speculum (OBD): OS, Bleeding, Discharge, ‐ Staphylococcus aureus (from baby’s mouth or over
laceration? Trauma to the vaginal wall? Any skin)
blood discharge or tissue coming out from ‐ E. coli
the OS ‐ Candida
o Bimanual Examination (TAC): Tenderness,
Adnexal, Cervical excitation, Uterine size: History
after delivery: 20weeks ‐ Congratulations on your pregnancy and delivery. How
ƒ 1day: 16weeks is your baby doing? Please tell me more about this
ƒ 10days: 10 weeks or less fever? Since when? Chills? Severity? Is it continuous
ƒ 2weeks: inside the pelvis or come and goes? Medicine? Did it help?
- Calf pain/tenderness? ‐ Associated symptoms: cough? SOB? Chest pain?
- Urine dip stick: blood +++ How are your waterworks? N/V? tummy pain?
Discharge from down below? How was the delivery
Diagnosis and Management (Normal or CS)? Did you have an episiotomy? Does
- Based on physical examination and history the most the wound hurt?
probable diagnosis is Endometritis. Do you know what ‐ Any leg pain? Redness over calf? Temperature?
it is? The inner lining of the uterus is infected. But you Swelling?
can also have retained POC or some bleeding ‐ Are you breastfeeding the child? Tenderness/pain
problems. I’d like to do some investigations to rule out over the breasts? Redness over skin? Bleeding
other conditions: FBC,CRP, ESR, Blood group and around nipples? Is it painful while feeding?
hold, If the temperature is about 38.5 we do blood ‐ PMHx: DM? Hypertension?
culture, urine micro/cult and sensitivity, coagulation ‐ SADMA?
profile, LFTs, U/S, Swabs(high vaginal swabs, wound
swabs, Endo cervical swab) Physical examination
- If it’s mild Endometritis: Outpatient treatment: Panadol ‐ General: Dehydration? Pallor? Jaundice?
or oral Amoxycillin plus Metronidazole. ‐ Vitals
- This patient is allergic to penicillin: use Cephalexin or ‐ Chest: auscultation for air entry or added sounds
Erythromycin. ‐ Heart: visible localized swelling
- If moderate or severe (Temp: 39, Dehydrated, Toxic ‐ Breast: redness? Fissure? Cracked nipples? Palpate
and tachycardia): hospitalize and IV antibiotics: for tenderness, engorgement of breast, change in
cephalosporins and Metronidazole 5~7days temperature over that area, local lymph nodes that are
- Review: in 2 weeks’ time enlarged
- Red Flags: Increase temperature, chills and rigors, ‐ Abdomen: tenderness especially suprapubic, size of
and increase in abdominal pain uterus, loin tenderness (pyelonephritis)
- Retained POC: Antibiotics for 24 hours and D&C ‐ Pelvic: lochia, discharge, odor, color; episiotomy:
- Mastitis: Flucoloxacillin. Continue breast feeding with redness, edema, tenderness over wound; discharge
complete empty. Compresses(hot and cold) Follow up Æ swab for culture and sensitivity
in 2 days. ‐ Leg: swelling, redness and tenderness
‐ Urine dipstick and BSL
38 
 
Investigations ‐ I understand you have come to see me because
‐ FBE, CRP you’re feeling unwell. Did you measure your
‐ Urine MCS temperature? Is it up and down or constantly high?
‐ Swabs if appropriate How’s your appetite? Do you have N/V? Do you feel
‐ USG: retained POC tired? Do you have loin or back pain? How’s your
‐ USG of breasts if in doubt of abscess; mastitis: clinical waterworks? Do you have burning sensation when
diagnosis passing urine? Has the color of your urine change?
Do you have abdominal pain? How is the wound
Management looking? Any discharge or redness? Do you notice
‐ Postpartum pyrexia where patients develop fever any unusual vaginal discharge or bleeding? Tummy
within 6 weeks of giving birth is due to a number of pain? Do you have cough, runny nose, earache or
reasons including infection of womb, UTI, or breast sore throat? Are you breastfeeding? Any problems
infections. On exam, I could see that your left breast is with breastfeeding? Do you have breast pain? Have
red and tender. Most likely you have mastitis. It is very you noticed any changes in the color of breast skin?
common during initial breastfeeding period. Any lumps or swelling? Any swelling under the
Sometimes, because of poor feeding technique, the armpits? Do you have calf pain or swelling? Did you
baby suckles on the nipple only. The nipple cracks have a planned or emergency CS? Were you
and bleeds and bugs from the baby’s mouth or mom’s discharge from the hospital as expected?
skin enter the breast tissue and cause infection. At the ‐ Are you generally healthy? Medications? allergies?
moment, I would give some antibiotics (flucloxacillin or
cephalexin for the next 7-10 days). I would give some Physical examination
analgesics (PCT). It is important to keep the breast ‐ General appearance
drained by feeding or manual evacuation/expression. ‐ Vital signs
‐ For soothing effects, refrigerated cabbage leaf. While ‐ Lymphadenopathy: neck and axilla
feeding, try to massage the breasts towards the nipple ‐ Breast:
to help evacuation. There are ointments available to o Erythema, swelling, cracked nipples?
help with cracked nipples. I will give you reading o Palpation: tenderness and lump (site, size,
materials regarding proper feeding techniques. You shape, surface, border, mobility, tenderness,
need to rest and have ample fluids. We will perform fluctuation), temperature
USG of the breast and repeat 3 days later to see if ‐ Abdomen: scar, palpable uterus (no longer palpable
abscess has developed. by 2 weeks); renal angle tenderness
‐ Red flags: If the redness/pain does not go away, if you ‐ Pelvic: abnormal discharge; PV: size of uterus,
feel a swelling on the breast, fever persists, please tenderness (bulky and tender if endometritis), adnexal
come back or if very ill or high spikes of fever Æ masses and tenderness
admit for IV antibiotics ‐ Calf tenderness
‐ Endometritis: admit! Rehydrate (aminoglycoside + ‐ Urine dipstick
clindamycin + metronidazole)
Diagnosis and Management
Mastitis ‐ You have a condition called lactational mastitis. Do
you know what it means? It is a common condition.
Case 2: You are a GP and a 29-year-old woman came in About 20% of breastfeeding women in Australia will
because of fever. She had cesarean section 3 weeks ago and face similar problems.
was recovering well until 2 days ago when she becomes shivery ‐ Mastitis is an inflammation of the breast which is
and cold. commonly associated with bacterial infection
Case 3: A 25-year-old lady 4 days postpartum forceps delivery (staphylococcus). Commonly, breastfeeding is
used, Temp 39, Redness and tenderness in right breast: challenging and can cause cracked nipples, which is a
Task painful condition that leads to milk stasis. It also
a. History creates an access for bacteria which live on the skin
b. Physical examination (flushed, T: 39, PR: 90, to enter the breast tissue and cause inflammation. I
Bp:125/75, axillary lymphadenopathy on the right, will order some blood tests (FBE, CRP and blood
obvious erythema of the right breast, tenderness on culture).
the RUOQ, well-healed CS wound,. BSL 6.6, urine ‐ You need to continue breastfeeding. Do you need the
dipstick normal; pelvic examination: normal discharge, help of a breastfeeding specialist? You need to keep
no tenderness ) the affected breast well drained. You can have a hot
c. Diagnosis and management shower or put a face towel before breastfeeding. Cool
the breast after breastfeeding. Massage any breast
Causes of Pyrexia lump gently towards the nipple while feeding. Empty
‐ Genital the breast completely after breastfeeding (manual or
o Endometritis pump).
o NSVD: infected episiotomy scar; infected ‐ I will give you antibiotics (Flucloxacillin x 5
laceration days/Cephalexin/Clindamycin) and painkillers.
o CS: wound infection ‐ If your condition does not improve in 24-48 hours or if
‐ Non-genital you have any concerns, please come back to see me.
o UTI We might need to do breast USD to rule out breast
o Mastitis abscess. Otherwise, I will see you in 2-3 days to
o DVT discuss results of your test.
- Initial: continue with breast feeding. If no response in
History 12-14 hours treat with Flucoloxacillin 500 QID
‐ Congratulations on the birth of your baby. Is it your - If allergic: Cephalexin oral 500
first baby? Is everything going well at home?
39 
 
Postpartum Checkup Diagnosis and Management
- On examination, we found that you have a dry vagina.
Case: Your next patient in GP practice is a 22-year-old lady for It can happen normally after childbirth. At this moment
checkup after her first baby was born 6 weeks. Her pregnancy you are breastfeeding that is why a hormone called
was uneventful and the baby was delivered normally. prolactin is high which inhibits estrogen, the lack of
which leads to a dry vagina and painful intercourse. It
Task should be fine within a few months. In the meantime,
a. Relevant history (dyspareunia) you can use lubricants. If it becomes persistent, I
b. Physical examination might need to refer you to the obstetrician.
c. Diagnosis and management - In terms of contraception, breastfeeding can be an
effective form of contraception but you have to fulfill
Postpartum Issues: several criteria: exclusive breastfeeding in infant, child
- Fever <6months, you have not had your periods yet. Failure
- Depression rate is 1-2%. There are other methods of
- Psychosis contraception which are safe in lactation. Minipills or
- Mastalgia progesterone only pills. It is better to start after 3-6
- Dyspareunia weeks and their efficacy is better. Another option is
- Contraception depo-provera. It is injected intramuscularly and needs
7 days before becoming effective. The efficacy is
History 98%. Another option is implanon and it is better to
- Congratulations! How does it feel to be a mother? start 3 weeks after delivery and can be used up to 3
How is the baby? Does the nurse visit you? years. The last one is IUCD. These are devices
- Was it a planned pregnancy? How was the antenatal inserted into the uterus 6 weeks after vaginal delivery
care? Anything significant? How was the delivery? and 12 weeks after cesarean section. IUCDs are
Long labor? Any complications? Are there any pain effectively immediate, last for 10-12 years. Efficacy is
down below, discharge? Are you breastfeeding or 99%. Higher chances of PID and ectopic pregnancy.
bottlefeeding? Any breast pain or sore nipples? Any Condoms.
masses or lumps in the breast? - It is your choice.
- Is there any social support available to you? - Reading material.
- Pap smear? Pills? Periods? - Review.
- Partner: Are you sexually active now? Any problems
with intercourse? Was there any bleeding after Primary Postpartum Hemorrhage
intercourse? STIs? Did you get your periods back?
What contraception were you using before? Have you Case: Your next patient is a 25-year-old primi who had a normal
started taking any contraception? vaginal delivery 20 minutes ago in one of the country district
- General health? hospital. You are an HMO on call. Pregnancy was normal. Labor
went for 14 hours and now the midwife calls you because the
Vital Signs patient has lost 1.5L of blood. She asks you to come and help
- General appearance her.
- Vital signs
Task
a. Ask the midwife appropriate questions (BP 85/5-, pale,
- Breast examination blood clotting, uterine lax, no lacerations)
- Pelvic examination: b. Advise her on what to do until you arrive
o Inspection: discharge, lochia c. Complete the management when you reach the
o Speculum: check for dryness of vagina, hospital
episiotomy scar
Types
Postpartum dyspareunia ‐ Primary: blood loss per vagina of more than 500ml in
the first 24 hour after delivery

HISTORY
o Atonic uterus (insufficient contraction Æ
shortening and kinking of the uterine blood
vessels and prevent further blood loss)
Pre-existing Acquired o Retained placental fragments Æ prevent
placental site retraction
o Laceration of genital tract
Refer to o Uterine rupture
Non-organic Organic
psycho-sexual ‐ Secondary: bleeding of more than 500 ml after 24
therapist hours
o Retained products of conception (placenta)
Episiotomy Vaginal o Birth trauma
Scar Issues dryness o Uterine infections (endometritis)

History
‐ What are the vitals (85/50, 130)? Is she
OB referral Lubricant hemodynamically stable? Can you please secure IV
lines, take blood for grouping and crossmatching, and
start IV fluids. Is she on a urinary catheter? If not, can
you please insert a catheter?
40 
 
‐ Is she conscious (Yes)? Is she having SOB (yes)? Physical Examination
Can you please give her oxygen. What was the mode ‐ General appearance
of delivery (instrumental delivery with forceps)? Was it ‐ Vital signs
a single baby or multiple? Any genital tear? Was ‐ Abdomen: distention, uterus, masses, organomegaly
episiotomy done? What is the weight of the baby and ‐ Pelvic: episiotomy wound, check site of bleeding
how is the condition? Is the uterus lax or contracted whether from wound, cervix or uterus, discharge;
(lax)? Have you checked the placenta? Do you think lacerations, od open; uterus; adnexal masses
there are retained products? Is the blood clotting? Is ‐ Urine dipstick and BSL
the patient bleeding from anywhere else (No)?
Diagnosis and Management
On arrival: ‐ From the history and examination, you are suffering
‐ Check vitals, IV lines and catheter from a condition called secondary postpartum
‐ Start syntometrin (Oxytocin + Ergometrine) hemorrhage which is bleeding after 24 hours of labor.
o Ergometrin contraindication: heart disease There could be a number of reasons for that but most
and hypertension likely, in your case, it is because of a small piece of
‐ Massage uterine fundus placenta which has been retained in your womb.
‐ Check placenta ‐ At this stage, I will admit you, secure IV lines take
‐ Do speculum examination to check for lacerations blood for FBE, coagulation profile, grouping and
‐ Call registrar crossmatching and start IV fluids. I will take some
swabs from your vaginal area for any infections.
Management ‐ I will call the OB registrar and arrange an USD. If the
‐ I have called the registrar and they will take you to the ultrasound confirms retained placenta, the specialist
theater to examine the uterus under anesthesia to will do curettage. I will also start you on antibiotics
check for any retained placental fragments. They can because the uterus might have been infected
do bimanual compression of the uterus. If it doesn’t (ampicillin + gentamycin + metronidazole).
work, they will give you intrauterine prostaglandins to
promote contraction. If unsuccessful, they will go for
internal iliac artery ligation.
‐ If all measures fail, the last resort would be
hysterectomy. However, we will do our best to prevent
this as this is only your first pregnancy.

Secondary Postpartum Hemorrhage

Case: You are an HMO in ED and your next patient is a 35-


year-old woman. She delivered her baby 5 days ago and she is
complaining of vaginal bleeding.

Task
a. History (started 10 hours ago, soaked 7-8 pads,
NSVD, BS 3.2 kg, epistiotomy +, pain in stomach +,
NSVD, full term, not a difficult labor)
b. Physical examination (pale, SOB, increased HR,
fever, tachypneic, postural drop, + tender uterus, +
bleeding, scar okay, no laceration)
c. Management

History
‐ Is my patient hemodynamically stable? When did it
start? How many pads have you used since then? Is
the pad fully soaked? Have you passed any clots or
tissues? Are you bleeding from anywhere else like
nose, gums, urine? Do you have SOB, palpitations or
dizziness? Do you think you have fever? Any other
vaginal discharge? Any tummy pains? How was the
delivery? Was the baby term? Was it a long or difficult
labor? Did they use forceps? Did they give you a cut
during delivery? What was the weight and size of the
baby? Have you established breastfeeding? Any
problems with breastfeeding? Any problems with
waterworks? Are you aware of your blood group and
your baby’s blood group? Any pain in your legs?
‐ Any other significant past medical history? do you
have any FHx of bleeding problems?
41 
 
Investigations:
GYNECOLOGY ‐ FSH, LH, prolactin, estradiol
‐ Chromosome analysis
DISORDERS OF MENSTRUATION ‐ Pelvic ultrasound

Primary amenorrhea(Constitutional Delay/Familial) Management


‐ You have a condition called primary amenorrhea. It is
Case: Your next patient is an 18-year-old girl with primary defined as failure to start menstruation by 16 years of
amenorrhea. All secondary sexual signs have been present for age. In your case, it is most likely physiological or
the past 3 years. She has not spoken about this to anyone, but constitutional or familial as your mom also had
is now in a relationship. delayed periods, but there could be other causes as
well. So I would like to organize some investigations:
Task FBE, USD (abdominal), and hormonal studies, TFTs.
a. Focused history ‐ Reassure. But if persistent, I can refer you to a
b. Examination specialist gynecologist who may consider hormone
c. Diagnosis and management challenge test.
‐ Review after reports are back.

Differential Diagnosis: Amenorrhea


‐ Imperforate hymen Case: 24-year-old female with a 1 year history of amenorrhea.
‐ Genital malformations/Mullerian duct agenesis
‐ Excessive exercise Task:
‐ Eating disorders a. Focused history
‐ Pituitary tumors/Hyperprolactinemia b. Physical examination
‐ Hypothalamic disorder (stress) c. Investigations
‐ Turner syndrome d. Management
‐ Gonadal dysgenesis (ovaries) e. Differential diagnosis
‐ Thyroid disorders
Secondary Amenorrhea
History - Natural: pregnancy/lactation/menopause
‐ Ensure confidentiality - Metabolic: unstable DM, renal failure, hepatic
‐ HPI: how about your growth spurt (recently 2-3 failure, thyroid disorders
years)? Do you think your breasts have developed? - Hypothalamic: eating disorders (anorexia),
Axillary and pubic hair? How is your height when you emotional stress, excessive exercise (competitive
compare it to your friends? Are you on a special diet? sports), drugs (GnRh
Do you do excessive exercise? Do you take laxatives agonist/danazol/contraceptives)
or induce vomiting? Do you consider yourself - Pituitary: tumors, micro and macro-adenoma,
overweight? How do you feel when you look at pituitary infarction (Sheehan syndrome)
yourself in the mirror? Any change in your weight? Do - Ovaries: POF, PCOS
you have any weather preferences? Any lump in your - Uterus: asherman syndrome
neck or change in your voice? How about your bowel
motion? Have you noticed any milk secretions from History:
your breast? Any problems with vision or headache? - Can you please tell me when was your last menstrual
Do you experience cyclical abdominal pain every period? Did your period stop suddenly or gradually?
month? Any breast tenderness or early morning - Period: Menarche? Were they regular? How long is
sickness? the cycle? How long is the bleeding time? Did you
‐ Partner, Pills, Pregnancy, Pap have excessive bleeding or pain during periods?
‐ Social: how are things at home? How’s the uni? Any Spotting in between periods?
stressors (boyfriend, home)? - Partner: are you sexually active? Are you in a stable
‐ FHx: do you know when your mom had her periods? relationship?
Do you have a sister? When did she have her - Pills: do you use any form of contraception? Which
periods? type?
‐ SADMA - Pap smear? Offer if >2 years. Was it normal?
‐ Are you stressed or worried about this? - Pregnancy: Have you ever been pregnant?
- Are you on a special diet? Have you been stressed for
Examination the past year? Do you exercise regularly? And how
‐ General appearance: normal for age; BMI - normal; often?
hirsutism - Do you suffer from headaches? Visual disturbance?
o Puberty: 8-13 (F), 9-14 (M) Have you noticed milky discharge from your nipple?
‐ VS: BP; - Can you tolerate cold weather? Any change in weight
‐ ENT: thyroid swelling in the past year? Do you suffer from constipation?
‐ Breast: breast development; axillary hair - Hot flushes? Dryness of vagina? Is intercourse
‐ Chest and heart: normal painful?
‐ Abdomen: suprapubic mass (imperforate hymen) - Do you notice any excessive hair growth? Acne?
‐ Pelvic exam: inspect external genitalia (tanner staging Deepening of the voice?
– pubic hair development); speculum: hymen - Are you generally healthy? Any gynecological
‐ Urine dipstick, PT and BSL procedures in the past? SADMA?
- FHx: POF, thyroid disorders

Physical examination
- General appearance: BMI, hirsutism, acne
- VS
42 
 
- ENT: visual fields; palpate thyroid; do breast - Sometimes it’s not possible to see inside the uterus
examination including nipple discharge immediately, so contrast and xray study might be used
- Abdomen: to find the uterine cavity and define all the scar
- Pelvic exam: tissues.
o inspection and speculum: atrophic vaginitis - After procedure, gynecologist will prescribe estrogen
o size of uterus; palpate adnexa for masses to increase the repair of the inner lining.
and tenderness - As any surgical procedure, hysteroscopy carries some
- DO pregnancy test: I understand that the likelihood of risks. Complications are uncommon but it includes
my patient being pregnant is low but pregnancy must anesthesia risk, infection, bleeding. Rarely,
always be included. perforation.
- It is effective treatment and most likely you can get
Investigations: pregnant again. Success depends on the extent of the
- FBE, U&E, LFTs disease and how difficult is the treatment.
- FSH, LH, estradiol, prolactin, TSH
- Pelvic USD Secondary Amenorrhea (Premature Ovarian Failure)
- BSL
- Bone density scan Case: 30-year-old presenting because she hasn’t had a period
- CT/MRI if suspecting pituitary tumors for about 1 year.

Management: Task:
- Kathy according to you history and PE, you most likely a. History
have secondary amenorrhea due to excessive Physical examination
exercise? We will still run investigations to exclude b. Investigations
other causes like problems with the thyroid gland, c. Diagnosis
ovaries, and pituitary glands. d. Management
- Decrease amount of exercise to moderate and your
periods will come back to normal History: same as 1st 2 cases. Ask for differentials!!
- Other options include OCP or HRT to prevent - PMHx: autoimmune disease (SLE, DM, RA etc..)
osteoporosis thinning of your bones - Chemoradiotherapy or previous surgery
- Increase calcium in diet or we can consider ca/vit D - Drugs (especially cytotoxic)
supplementation - FHx: 10% of POF run in family

Asherman syndrome after miscarriage Physical examination:


- General appearance: hirsutism, acne, BMI
Case: 30-year-old lady who had a miscarriage 5 months ago - Vitals
and has come to see you because she hasn’t had a period yet. - Visual fields, Thyroid, breast
OTHER: after manual removal of placenta - Abdomen
Task - Pelvic: Inspection – atrophic changes
a. History - Bimanual examination – size of uterus, adnexal
b. Manage case masses and tenderness
- Pregnancy test
History:
- I understand that you have come to see me due to Investigations:
absence of your period since 5 months. - FSH (high), LH, estradiol, prolactin, TSH
- Did you experience any spotting? Sorry to ask but at - Pelvic USD (thin endometrium, few follicles growing)
which age of pregnancy did you miscarriage? Did you - BSL
have curettage? Any problems after miscarriage - Bone density scan
(fever? Bleeding? Discharge?)
- 5Ps Management:
- Ho w many pregnancies have you had? Could you be - According to your history and PE, you have secondary
pregnant? Any gynecological procedures done in the amenorrhea most likely due to premature ovarian
past? failure. POF is a condition in which ovaries stop
- Ask differential diagnosis questions functioning normally in women who are younger than
40. In women with POF, the ovaries stop releasing
Management: eggs or release them only intermittently, and stop
- Sophie first we would do PT to exclude pregnancy. producing the hormones estrogen, progesterone and
We also would check your hormones, FSH, LH, testosterone or produce them only intermittently.
estradiol, prolactin, TSH to exclude different causes - In the vast majority of cases, POF has no known
- We will do pelvic usd. According to your history you cause. Some cases of the condition can be explained
most likely have secondary amenorrhea due to by genetic abnormalities (Turner or fragile x
asherman syndrome. It is the formation of adhesion or syndrome), exposure to toxins and autoimmune
scar tissues inside the uterus. It is a well-recognized disorder.
complication of curettage. - That’s why I would like to refer you to a gynecologist
- The ability of the inner lining of the uterus to recover is for further assessment and management.
reduced during the pregnancy. Together with surgical - You also need to know that POF often interferes with
procedure, it contributes to scarring inside the uterus. a woman’s ability to get pregnant. Even so, between 1
- There is a treatment for this condition. I will refer you and 10% of women with the condition are able to
to a gynecologist. Most likely the doctor will perform conceive normally as there is a chance of intermittent
hysteroscopy. Under anesthesia,a small flexible optic improvement of ovarian function.
tube is placed thru the cervix into the uterine cavity. It - However, if infertility is a problem, there is also
helps to see intrauterine adhesions and cut them. effective an treatment for that, IVF with donor eggs.
43 
 
- One of the main goals for POF is replacement of Investigations
estrogen that the ovaries stopped producing. OCP or ‐ LH: FSH ratio = 3:1; estradiol, testosterone,
HRT are usually used. These hormonal pills will androstenedione, prolactin, TFTs, FBS, pelvic USD
reduce your symptoms, prevent osteoporosis and
possibly, cardiovascular disease. Most experts agree Management
that young women with POF should use hormonal ‐ According to history and PE, you most likely have
therapy at least until they turn 50. secondary amenorrhea due to Polycystic ovarian
- HRT Æ if patient wants to get pregnant because of syndrome. We will run investigations to confirm it and
lower levels of hormones exclude other causes.
‐ PCOS is a condition that causes irregular menstrual
Secondary Amenorrhea secondary to Polycystic Ovarian cycles because monthly ovulation is not occurring and
Syndrome levels of androgens or male hormones are elevated. It
is a very common condition. About 5-10% of women
Case: A 21-years-old lady has come to see you in your GP are diagnosed with PCOS. The cause of PCOS is not
clinic. She hasn’t had a period for a few months. completely understood. It’s believed that abnormal
levels of the pituitary hormone LH and high level of
Task androgen interfere with normal function of the ovary.
a. History (sexually active, stable partner, not on OCP, ‐ Classic PCOS symptoms include absent or irregular
amenorrhea x 6 months; menarche at 16, teacher, not periods, abnormal hair growth, scalp hair loss, acne,
so stressed; no headache, weakness, n/v, s/p weight gain, and difficulty becoming pregnant.
appendectomy, goes to gym 2x a week, no weather Although PCOS is not completely reversible, there are
preference, a number of treatments that can reduce symptoms.
b. Physical examination (BMI 27, increased hair on chin ‐ What is your main concern? Periods and Hirsutism
and upper lip) o Lifestyle modification like healthy diet and
c. Investigations regular exercise often help to normalize
d. Management menstrual function.
o We can also use oral contraceptives for 6
Features: months plus hair treatment like laser therapy
‐ Roterdam criteria (2/3) or electrolysis
o Anovulatory cycles (prolonged >40 days and o If it’s not effective I will prescribe OCP plus
irregular) anti-androgen (spironolactone or
o Clinical or biochemical of androgen excess cyproterone acetate) – Yasmin and Diane
o PCO on USD (>12): necklace appearance ‐ Pregnancy
‐ Increased LH Æ increased androgen o Lifestyle modification and try to have regular
(androstenedione) Æ converted to testosterone sexual life for 6 months. If you cannot
‐ Decreased FSH Æ increased insulin secretion conceive, I will refer to gynecologist for
specific treatment.
History o Metformin (improves insulin resistance and
‐ I understand you came to see me because you are weight loss):
worried about your periods? When was your LMP? o Clomiphene citrate – 50-70%
Did your periods stop suddenly or gradually? o FSH injection
‐ Periods: When was your first period? How many days o Surgery: drilling
of bleeding? How many days apart? Any bleeding in o IVF
between? ‐ Are there any complications of PCOS?
‐ Stable relationship? Sexually active? o Type II Diabetes
‐ Are you on any contraceptives? When was your last o Endometrial Hyperplasia and Cancer
pap smear? Was it normal? o Sleep apnea
‐ Have you ever been pregnant? Are you trying to o Metabolic syndrome
conceive at the moment? o Depression
‐ Are you on a special diet? Have you been stressed ‐ Review once labs are in. Reading material.
recently? Do you exercise regularly? Have you
suffered from headaches or any visual disturbance? Pubertal Menorrhagia (Metrostaxis)
Have you noticed milky discharge from your nipples?
Can you tolerate cold weather? Any recent change in Case: Your next patient in GP practice is a 12-year-old who is
your weight? Any recent change in your weight? Do having heavy periods for the last 10 days
you suffer from constipation? Do you have hot Task
flushes? Is intercourse painful? Have you noticed a. History (vaginal bleeding 5-6 pads/day, soaked with
excessive hair growth, acne, thinning of your hair or big clots; 1st period)
deepening of your voice? b. Physical examination (distressed, pale, tenderness in
‐ Are you generally healthy? Have you ever had any lower abdomen; postural hypotension; tachycardia)
surgeries or gynecological procedures? SADMA? c. Diagnosis and management
‐ FHx
History
Physical Examination - Is my patient hemodynamically stable?
‐ General appearance, hirsutism, acne and BMI - When did the bleeding start? How many pads per
‐ Vital signs day? Are they soaked? Is it getting worse or better?
‐ Visual field, thryoid and breast exam What is the color of the blood? Any clots? Is it smelly?
‐ Abdomen Is she bleeding from anywhere else? Is this her first
‐ Pelvic exam: size of uterus, adnexal masses and period? Does she bruise easily from minor trauma?
tenderness Any dizziness, SOB, fainting or palpitations? Is there
‐ Urine dipstick and urine PT any tummy pain? Any possibility of trauma or foreign
44 
 
body down below? Do you know if your daughter is History:
sexually active? Any chance she could be pregnant? - HPI: Since when? Are your periods regular? How
Any bleeding disorder running in the family? How many days do you bleed for? How many pads do you
about the development of breast and pubic hair? Is in a day? Are they soaked? Any clots? Any associated
she on any kind of medications? Do you know her discharge with the bleeding? When was your last
blood group? SADMA? Is her immunization up to period? Any spotting or bleeding in between your
date? Have you considered vaccination against HPV? periods? Any pain during periods? What was your age
at menarche? Do you have symptoms like n/v
Physical Examination headache, irritability, swelling of your body before
- General appearance: distressed, pallor, dehydration, periods? Are you sexually active? Stable relationship?
jaundice May I ask do you have any problems related to
- Vital signs: postural hypotension, tachycardia, RR, intercourse, e.g. pain/bleeding? What contraception
Temperature and oxygen saturation normal do you use? What type and since when? Have you
- Neck swelling (Thyroid) and LAD used IUCD? Have you ever been diagnosed with STIs
- Tanner Staging or other pelvic infections?
- Signs of skin bruising - I understand you have 3 kids, age of last child? Mode
- Chest and heart of delivery?
- Abdomen: organomegaly, tenderness - Have you ever suffered from a bleeding disorder, DM,
- Pelvic exam: inspection for blood clots, signs of thyroid? Any previous gynecologic sx? Have you lost
trauma, sexual abuse; development of genitalia; weight recently or change in appetite? Any night
sweats or prolonged fever?
Diagnosis and Management - SADMA? Pap-smear?
- Your daughter has a condition called pubertal - FHx: bleeding, thyroid
menorrhagia which is not uncommon. Because Jane
is not stable, and her BP is falling, I would like to Physical examination:
organize an ambulance, and start IV lines. They will - General appearance: pallor, jaundice, dehydration,
take blood for investigations: FBE, coagulation profile, BMI
PT, APTT, vWF screen (factor VIII assay, vWF - VS: BP (postural)
antigen, ristocetin cofactorm PFA-100), urine - Skin: bruises or purpura
Chlamydia, PCOS screen, 17-OH-P, platelet-function - Stigmata of hyperandrogenism: acne, hirsutism,
assay, iron studies, pregnancy test, TFTs, blood group central obesity, pigmentation, change of voice, male-
and crossmatching. pattern baldness
- At the hospital she will be seen by a specialist and - Palpate thyroid for enlargement and LN
they will start her on IV premarin for sometime plus - Auscultate chest/heart
tranexamic acid to control the bleeding. Depending on - Palpate abdomen and check for tenderness especially
the results, they might do blood transfusion. Once she the R/L iliac fossa; palpable mass;
is stabilized, they would put her in - With patient’s consent, I would like to ask for the
uninterrupted/continuous OCPs plus iron tablets until pelvic exam. On inspection, I would like to quantify
her hemoglobin is normal for at least 3 months. bleeding (soaked pads), any clots or associated
- Reassure. discharge? With a sterile speculum I would like to look
- If patient is hemodynamically stable: give oral at the cervix for any signs of trauma, ulcerations,
estrogen, do workup, continuous and iron tablets. lesions, and polyp. Take swabs of vagina and culture
to check for infections. Do bimanual examination
Abnormal/Dysfunctional Uterine Bleeding looking for any signs of cervical excitation, size and
shape of uterus, and any pelvic mass I can feel.
Case: 43/F comes in with painless heavy periods for the last
four months. She is mother to 3 kids. Previously, some Investigations:
investigations have been done including FBE, hormonal profile, - FBE, U,C/E, coagulation profile, blood grouping, LFTs,
pap smear, diagnostic d&c and an endometrial biopsy. All TFTs, iron studies, complete hormonal assay
results are normal except for her hemoglobin which is 70. including serum b-hcg, pap smear, TVS (fibroids and
check thickness of endometrium).
Task: No further history taking allowed. - Hysteroscopy w/ or w/o endometrial sampling –
a. Talk to the patient regarding diagnosis and future visualization of uterus
management - CT/MRI may needed

Differential Diagnosis: Management


a. Fibroids - Most likely from your history and PE, you have a
b. Endometrial cancers condition called DUB where you have bleeding without
c. DUB an apparent cause in spite of complete investigations.
d. Bleeding disorders It is a very common condition, the cause of which is
e. Hypothyroidism usually not known. It is suggested that disturbances of
f. Cystic hyperplasia the normal brain axis leads to hormonal changes.
g. PREGNANCY Sometimes there is a problem within the vasculature
h. Drugs – anticoagulants, estrogen-containing of the endometrium, which is the lining of your womb,
preparations, anti-psychotics (there is reduced vasoconstriction of endometrial
i. IUCD vessels and increased prostaglandin E1 and
j. Trauma prostacyclin)
k. PID (Chronic) - It is a diagnosis of exclusion. The therapy is a step-
l. AVM ladder therapy. We start with medical intervention,
reserving the surgical intervention for resistant cases.
45 
 
- Therapy: ‐ Take iron supplements
o Medical (controls up to 70% of cases): ‐ Eat well-balanced diet
Mefenamic acid (500mg TID) 4 days before ‐ Avoid aspirin
next period Æ tranexamic acid (1g OD on
D1 of menses) Æ OCP Æ Danazol (anti- Postcoital Bleeding (Cervical Cancer)
estrogen)Æ GnRh agonist (zoladex and
synarel) Case: You are a GP and a 40-year-old female comes to you
o Surgical: D&C (high recurrence)Æ complaining of vaginal bleeding after intercourse for the last 7
endometrial ablation (laser/cautery)Æ days. She is a mother of 4 kids. Her husband passed away 2
uterine artery ligation/embolization Æ years ago and she has a new partner recently.
hysterectomy
- Give iron supplements: ferrous sulfate 325mg TID 3 cases about cervical cancer:
- Prognosis is good with medical therapy. Up to 70%
success rate. Task
a. Relevant history (2 pads/day x 2days; no clots; last
Dysfunctional Uterine Bleeding pap smear x 14 years ago)
b. Physical examination findings: abdomen: non-tender
Case: Reena, aged 41 years presents to your clinic. She tells mass palpable and uterus enlarged to 12 weeks;
you that she had heavy periods for the last few months and inspection: (+) bright red blood on speculum, os
describes them as a nightmare. Previously she had regular closed and (+) mass probably originating from the
periods but for sometime, they have becomes excessively cervix; speculum: small ulcer on the cervix
heavy. She feels tired and has to put herself to bed every month c. Management
for at least one day. She works as an accountant in a busy
company and has to take a few offs every month due to which Presentation: postcoital bleeding in a female smoker who does
her boss is not happy and she is at risk of losing her job. She not have regular pap smears
lives at home with her partner and two children.
Differential Diagnosis
Task ‐ Atrophic vaginitis
a. Further history ‐ Cervical ectropion
b. Physical examination ‐ Cervical polyp
c. Differential diagnosis and management ‐ Cervical cancer
‐ Endometrial cancer
Differential diagnosis ‐ Bleeding disorder
‐ Fibroids
‐ Contraceptives (depo-provera) History
‐ Endometriosis ‐ Is my patient hemodynamically stable?
‐ IUD ‐ Please tell me more about the bleeding? Was it
‐ Bleeding disorders and warfarin related to intercourse the first time you had bleeding?
‐ Miscarriage and ectopic pregnancy How many pads are you using for the bleeding at the
‐ Endometrial cancer moment? Any clots? Any discharge apart from the
‐ Thyroid disorders bleeding? Any itchiness? Any problems passing
water? Any bleeding from anywhere else in the body
History: (nose, gums)? When was your LMP? Any possibility
‐ 5Ps. Signs and symptoms of thyroid disorders, you might be pregnant at the moment?
bleeding disorders, weight loss Contraception?
‐ At the moment do you have any dizziness, N/V,
Physical examination lightheadedness? How are your cycles? Are they
‐ General appearance regular? Any bleeding in between cycles?
‐ Vital signs ‐ I understand you have 4 kids. When was your last
‐ Chest and Lung delivery? Any complications during any of the
‐ Abdomen: masses pregnancies or deliveries?
‐ Pelvic examination ‐ How is your general health? Any history of high blood
‐ DO PREGNANCY TEST! pressure, DM, bleeding disorders, thyroid problems?
‐ When was your last Pap smear? What was the result?
Investigations May I ask why you didn’t have any pap smears?
‐ FBE, TFTs, LFTS, UEC ‐ SADMA?
‐ Abdominal and vaginal ultrasound ‐ FHx: gynecological cancers, bleeding
‐ D&C ‐ Do you have any weight loss? night sweats?
‐ Endometrial sampling Tiredness? Any pain anywhere in the body? Any
‐ D&C lumps you have noticed?
‐ Hysteroscopy
Physical examination
Treatment ‐ General appearance
‐ Aim is to reduce the amount of blood loss ‐ Vital signs: postural BP
‐ Give hormone replacement (progesterone), anti- ‐ Abdomen: obvious abdominal distention, tenderness
prostaglandin medication (NSAID) or blood clotting on palpation, mass (can I find out if it is uterine or
and reduce bleeding (tranexamic acid) ovarian in origin) is it tender? Mobile? Percussion
‐ Options: OCP, progesterone tablets, progesterone note? Any other viscera that is enlarged? Ausculate
releasing IUD, tranexamic acid (most effective bowel sounds?
therapy; reduce bleeding by 50% 4x a day for 4 days) ‐ Lymph nodes especially inguinal lymph nodes
‐ Keep a menstrual diary ‐ Heart and lungs
‐ Rest as much as possible
46 
 
‐ Pelvic examination Risk factors:
o Inspection: discharge, bleeding, clots - Nulliparity
o Pelvic examination: evidence of ectropion - Early menarche, late menopause
(seen as very red patch over cervix which - Unopposed estrogen therapy (OCP/HRT)
bleeds upon touching), cervical os - DM
o Bimanual examination: palpate mass, - Obesity
tenderness, whether os is open or close,
cervical excitation, adnexal mass that I can Task:
feel? a. Take a further history required
‐ Urine dipstick and pregnancy test b. Ask the examiner relevant examination findings
c. Discuss further management plan with the patient
Diagnosis and Management
‐ From the history and examination, my concern is a Physical examination
mass we have noticed to be arising from the pelvis. - General appearance: BMI,
Unfortunately, it seems like this mass is probably a - Vitals: BP, RR, PR, Temperature
nasty growth of the cervix. - Abdomen: scars, masses, striae, masses,
‐ Do you understand what I’m saying? Do you want me organomegaly, FHT, lateral grip, pelvic grip, FHT
to call someone for you? Are you alright to continue? - Gynecological exam:
‐ The first step would be to confirm the diagnosis with o informed consent
the help of a procedure called colposcopy and biopsy. o inspection: discharge, ulcers, lesions, warts,
It will be done by the specialist gynecologist. We will scratch, atrophic changes; speculum Æ
also do some blood tests before the biopsy including cervix, atrophic changes; do PAP and
FBE, LFTs, UEC, TFTs, MSU for MCS. Once the endometrial sampling where possible
diagnosis is confirmed, they will do CT scans of the
chest, abdomen and pelvis to find out at which stage Atrophic vaginitis – dx usually done using speculum examination
the disease is at. Depending upon the staging, the seen as thin, friable vaginal wall which may bleed to touch.
treatment options are cone biopsy (reserved for stage Typically, history will be a 10-year-postmenopausal lady
1), total hysterectomy, chemotherapy and complaining of a yellowish-brown vaginal discharge or just
radiotherapy. mucus. (+) dyspareunia; tx: topical estrogen cream initially and
‐ Staging systemic estrogen/progesterone (if uterus intact)
o I – confined to cervix
o II – involves the uterus but not the lower 3rd Cervical polyps – diagnosis usually made in speculum
of vagina examination – seen as red protrusion from the cervical os; tx:
o III – extends to the pelvic walls including the attempt removal by grasping it with sponge forceps and twisting
lower 1/3 of vagina the pedicle
o IV – distant metastasis
‐ Refer to gynecologist for further management Endometrial polyp – usually identified on hysteroscopy where
‐ Please don’t worry. There is still a lot of hope for you. the polyp is directly visualized; typical history of a 50-year-old
‐ Support groups. Counselor. female with bleeding PV; estrogen dependent (incidence
‐ Review decreases after menopause. History of tamoxifen use; tx:
‐ Reading material. removal during hysteroscopy and send to pathology
‐ If pregnant:
o <20 weeks: offer termination Endometrial hyperplasia – diagnosed on hysteroscopy; TVS is
o >20 weeks: may still offer termination but more accurate. Two types: simple (proliferative endometrium)
usually induction of labor done after 35 or complex (proliferative endometrium with atypical changes and
weeks gestation if left untreated will progress to cancer in 2 years); tx: high-dose
progesterone with frequent reassessment; definitive tx:
Postmenopausal bleeding hysterectomy with oophorectomy

Case: Your next patient in your GP practice is a 52-year-old lady Endometrial CA – 5th most common cancer in women in
who complains of bleeding PV. She initially noticed brownish Australia. Px complains of vaginal bleeding or irregular
staining of her underpants a week ago and came to get a postmenopausal bleeding; (+) hx of anovulatory cycles or
checkup. abnormal endometrial cells on pap; tx: surgical removal (Total
hysterectomy Æ bilateral salpingo-oophorectomy Æ bilateral
Investigations ordered: pelvic and para-aortic LAD Æ peritoneal cytology) and staging
- bHcG – normal during surgery; good prognosis if diagnosis is made early;
- FBE – Hb 12m/L, wbc 8500 consider RT for deeply invasive tumor
- Abdominal USG
o Normal uterus, tubes and ovaries DYSMENORRHEA AND ABDOMINAL PAIN
o Endometrium 12 mm thick (4-8mm)
Dysmenorrhea
Problem list:
- Hemodynamic stability Case: Mary is 14 years old presents to your GP clinic while her
- History to r/o differential diagnosis mother is outside in the waiting room. You know her for 6 years.
o Hormones – estrogen content of HRT She complained of severe central lower abdominal pain with her
o Vaginal /uterine atrophy periods for the past few menstrual cycles. The pain gradually
o Uterine cervical polyps begins on day one of her menses and becomes very severe
o Endometrial hyperplasia within a few hours. She gests nauseated and sometimes vomits
o Cancer (uterus, cervix, vagina) and sometimes she feels a nagging ache at the top of her thighs
47 
 
when she gets the abdominal pain. Paracetamol does not - Medication
relieve the pain. Yesterday her mom gave her strong analgesia o ASA or PCM
with codeine (endone) which relieved the pain but she slept for o Prostaglandin inhibitors (Mefenamic acid)
the remainder of the day. Mary’s menarche was at 13 years of o NSAIDS (Naproxen or ibuprofen) Æ start 1
age. Her cycles were irregular for the first 6 months but now are day before the period then continue for the
regular every 28 days lasting about 7 days. She is otherwise next couple of days
well. Her mother suggested Mary to see you because she is o Thiamine 100mg
concerned that the severity of pain might indicate that there is o Low-dose OCP
something serious with Mary. - Initially during first 1 or 2 years of period, you don’t
produce eggs and therefore you don’t experience
Task pain. However, when eggs become produced,
a. History (menses started yesterday, 1 pad/day, every chemicals (prostaglandin) are released which increase
28 days, sexually active and uses condoms, not on the contraction of the uterus (womb) producing pain.
OCPs)
b. Physical examination Secondary dysmenorrhea: menstrual pain for which an organic
c. Investigation cause can be found; begins after menarche, after years of pain-
d. Diagnosis and management free menses; >30 years of age; begins 3-4 days before menses
and becomes more severe during menstruation. May have
- Ensure CONFIDENTIALITY at all times!!!!!! intermenstrual pain, dyspareunia, etc.
- Consent: - Causes:
o legal age: 18 y.o o PID
o sexual activity: 16 y.o. o Endometriosis
o mature minor: >12 y.o. o IUCD
- Scale PAIN! o Submucous myoma
- Sexual history: are you sexually active? How long o Intrauterine polyp
have you been active? Are you in a stable o Pelvic adhesions
relationship? How long? How many sexual partners
have you had? Do you practice safe sex? Investigations
- SADMA? - FBE
- Other bleeding problems - MSU
- Pregnancy test
Dysmenorrhea Differential diagnosis: - USD
- Hysteroscopy, D&C, HSSG
Primary dysmenorrhea
- Menstrual pain associated with ovular cycles without Mittelschmerz
any pathologic findings; usually commences within 1-2
years after menarche and becomes more severe with Case: You are an HMO in ED and a 14-year-old girl comes
time up to about 20 years. complaining of severe lower abdominal pain.
- 50% of women and up to 95% of adolescents
- Features: Task
o Low midline abdominal pain a. History (severe right lower quadrant pain, 7/10, for 2
o Pain radiates to back or thighs hours, 3rd time for 2 months relieved by panadeine
o Varies from a dull dragging to a severe forte, 2nd episode went to hospital, workup done was
cramping pain normal, can’t remember what doctor said, and
o Maximum pain at beginning of the period discharged after being pain-free, periods regular 28-
o May commence up to 12 hours before the 30 days, not sexually active, FHx of DM and MI; LMP
menses appear 2weeks
o Usually lasts 24 hours but may persist for 2- b. Physical examination (BMI 17, mild tenderness of
3 days deep palpation on RIF, hymen intact)
o May be associated with nausea and c. Investigation if relevant
vomiting, headache, syncope or flushing d. Diagnosis and management
o No abnormal findings on examination
- Investigations: Features
o MSU - Rupture of Graafian follicle Æ small amount of blood
- Risk factors: mixed with follicular fluid released into pouch of
o Obesity Douglas Æ peritonism
o Smoking - Features: onset of pain in mid-cycle, deep pain in one
o Early age at menarche or other iliac foosa (RIF>LIF), often described as
o Longer periods “horse-kick pain”; tends to move centrally; heavy
o Alcohol feeling in pelvis; relieved by sitting or supporting lower
o Lack of exercise abdomen; lasts for fe minutes to hours
o Anxiety, stress, depression - Patient otherwise well
- Management: - Sometimes can mimic acute appendicitis
o Lifestyle modification - Management
o Avoid smoking/alcohol o Explain and reassurance
o Relaxation techniques (yoga) o Simple analgesics
o Avoid exposure to extreme cold o Hot water bottle
o Place a water bottle over the painful area
48 
 
Differential Diagnosis - Usually pain appears suddenly in the middle of the
- Ectopic pregnancy cycle and subsides within hours. It is not harmful and
- PID doesn’t signify presence of disease.
- Ruptured ovarian cyst - You need to have rest. Drink plenty of fluids. Take
- Ovarian torsion panadol or ibuprofen or Panadeine or Panadeine
- Mittelschmerz forte. You can use local heat applications or warm
- UTI baths.
- Acute appendicitis - If pain is severe and doesn’t respond to simple
painkillers, your doctor might consider OCP to block
History ovulation.
- Is my patient hemodynamically stable? I understand
you came to the ED because of abdominal pain. PREMENSTRUAL SYNDROME
When did it start? Can you show me with one finger
where is the pain? Has it always been there or did it Premenstrual Syndrome (PMS)
start somewhere else? Can you describe the type of
pain? Does the pain travel anywhere else? Can you Case: Nancy aged 32 years visit your surgery and tell you that
recall any precipitating factors? How bad is the pain she frequently feels irritable, tearful and bloated before her
on a scale of 1-10? Does anything make it better or periods every month. This has been going on for last couple of
worse? Is it the first episode? What happened last years. Her menses are regular lasting for about one week and
time? Do you remember any investigation result and symptoms completely resolve within 1-2 days of onset. Her
what doctor said? Are there associated symptoms like menses occur every month and she considers they are not
fever, nausea or vomiting? How’s your waterworks? painful and are not heavy. She has no bleeding in between her
Any stinging or burning sensation? Has the color of menses or after intercourse. She is a school teacher and lives
urine changed? How are your bowel movements? with her husband at home. She had two children aged four and
When was the last time you opened your bowels? six years of age.
- When was your LMP? How long is your cycle? How
long is the bleeding? When was your first period? Any Task
excessive pain or bleeding during the periods? Are a. History
you sexually active? b. Physical examination
- How’s your general health? Any surgeries in the past? c. Diagnosis and further advice
SADMA?
- Whom do you live with at home? Any problems at Predisposing factors:
home or in school? ‐ Mental illness
- FHx ‐ Alcoholism
‐ Sexual abuse
Physical Examination ‐ Family history
- General appearance ‐ Stress
- Vital signs and growth chart
- Abdomen: Precipitating factors
o Inspection ‐ Cessation of OCP
o Palpation: guarding, rigidity, rebound ‐ Tubal ligation
tenderness, tenderness at McBurney point, ‐ Hysterectomy
Rovsing sign, Psoas sign (pain on extension
of hip), obturator sign (pain on internal Sustaining factors
rotation of hip) ‐ Diet – containing caffeine, alcohol, sugar
o Auscultation ‐ Smoking
- Urine dipstick ‐ Stress
‐ Sedentary lifestyle
Investigation: Transbadominal USD + Doppler (helps exclude
torsion) Differential diagnosis
‐ Psychologic: Depression
Diagnosis and Management ‐ Thyroid disorders
- According to your history and PE, most likely you have ‐ PCOS
a condition called mitteschmerz syndrome. Have you ‐ Mastalgia
ever heard about it? The word means “middle pain” ‐ Menopause syndrome
because this pain is typically felt during the middle of
the menstrual cycle. This pain coincides with History
ovulation. It’s a very common condition. As many as 1 ‐ Rule out anxiety and depression question
in 5 women experience mittelschmerz pain. Some ‐ Home situation
every cycle, some intermittently. It is more common in ‐ Ask about psychologic symptoms: Insomnia,
young women under 30. There are a number of Moodiness, Irritability, Anxiety, Tension, Depression,
theories why women experience this pain. Confusion, Food cravings
o The ovaries have no opening. At ovulation, ‐ Physical symptoms: headache, dizziness, hot flushes,
the eggs break through the ovary wall and breast swelling and tenderness, abdominal
causes pain. bloatedness, constipation
o At time of ovulation, blood is released from
ruptured egg follicles and may cause Management: Diary/CBT/Lifestyle modification/Relaxation Æ
irritation of the abdominal lining. antidepressants
o There is also contraction of fallopian tubes
and some other contributory factors leading
to spasm and pain
49 
 
‐ Diary: write her symptoms for at least 2-3 months
period.
‐ CBT o You must note the temperature on a chart to
‐ Lifestyle modification (exercise, diet) compare changes from day-to-day. Avoid
‐ Relaxation sexual contact from the first day of period up
‐ Medication to 72 hours after rise in temperature
o Nil or negative: evening primrose oil, gingko o 99% effective if done correctly and
biloba, progesterone, OCP, bromocriptine consistently;
o Weak: magnesium, calcium, vitamin E, vitex o Benefits: no side effects
angus o Limitations: tedious and should be
o Moderate: pyridoxine vitamin b6 (mild- motivated; unsuitable if woman has fever or
moderate), st. john’s wort, spironolactone other health condition; period of abstinence
o Strong (for PMDD): SSRI and clomipramine, of longer
GnRH agonists, danazol ‐ Calendar/Rhythm Method:
‐ PMDD: fluoxetine 20mg mane for 10-14 days before o Monitor 6 (at least 12) cycles and select the
anticipated onset of menstruation or sertraline 50 mg shortest and longest cycle.
daily o Shortest Cycle (– 21) and Longest cycle
(10)
Pre-menstrual Dysphoric Disorder ƒ 14-6 = 8 (Sperm viable for 6 days)
ƒ 14+2 = 18 (Egg viable for 2 days)
PMDD Criteria: (A) Symptoms must occur during the week o Avoid sexual contact on the unsafe days.
before menses and remit a few days after onset of menses; five o 95% effective if used correctly
of the following symptoms must be present with at least one o Benefits: No side effects, no cost, and do
being 1-4 and should be symptom free for one week: not require any special device
‐ Depressed mood or dysphoria o Limitations: must monitor length of
‐ Anxiety or tension menstrual cycle for 6 months;
‐ Affective lability ‐ Billings Ovulation Method
‐ Irritability o Based on careful observation of the nature
‐ Decreased interest in usual activities of mucus so that ovulation can be
‐ Concentration difficulties recognized
‐ Marked lack of energy o Fertile mucus is wet, clear, stringy and
‐ Marked change in appetite, overeating or food increased in amount and feels lubricated
cravings due to estrogen
‐ Hypersomnia o Last day of this type of mucus is peak
‐ Feeling overwhelmed mucus day which is followed by abrupt
‐ Other physical symptoms change of thick mucus associated with
‐ B. Symptoms must interfere with work, school, usual secretion of progesterone
activities or relationship o Infertile phase: 4 days after peak mucus day
‐ C. Symptoms must not merely b an exacerbation of o Intercourse is avoided from the first
another disorder awareness of increased clearer wet mucus
‐ D. Criteria A, B and C must be confirmed by until 4 days after maximum mucus
prospective daily ratings for at least 2 cycles secretion.
o Most effective method if done correctly;
CONCEPTION CONTROL failure rate is 1-2/100 women-years
o Failure: women are only able to detect 3-
Natural methods of contraception 4days of wetness prior to the peak moisture
day and still have sex 4-6 days prior to
Case: A 19-year-old females comes to your GP clinic to consult ovulation when sperm survival is still
about contraception as she is now going to start sexual relations possible
with her boyfriend. She is not interested in barrier methods or ‐ Coitus interruptus: male withdrawal before ejaculation;
hormonal contraceptives and wants to know about natural least effective
contraception. ‐ NOT EFFECTIVE AGAINST SEXUALLY
TRANSMITTED INFECTIONS
Task
a. Focused history : 5Ps (gardasil vaccination) and Counseling about OCP
general health
b. Explain methods Case: 14-year-old girl who seems to be mature for her age
came to you for contraceptive advice. She is your regular
Natural: They require regular periods and high motivation. patient, is generally healthy. She has been with her 15-year-old
These methods will help determine when to avoid intercourse boyfriend 6 months now, and wants to discuss the OCP with
during your cycle, meaning your safe and unsafe periods. you.
‐ Bibasal temperature: relies on measurement of your
body temperature Task:
o Check temperature every morning before a. take a further history required
getting out of bed or any activities. b. ask for relevant findings
Temperature rises slightly during ovulation. c. discuss OCP with the patient
o 0.2-0.5C increase in temperature indicates
ovulation Æ avoid sexual contact for up to
72 hours after the change in temperature
50 
 
Problem list: - It also depends on the type of progesterone.
a. confidentiality and consent - At this stage, I would recommend for you to continue
b. social/ethical aspect (boyfriend’s age) for 4-6 months and if it does not stop after that, then
c. 14-year-old minor – Gillick’s test we might consider changing your OCP dose to a
d. Discuss OCP – r/o contraindications; usage higher estrogen-containing pill or different
instructions and adverse effect progesterone.
- Review and Reading materials.
HEADS (psych history) - Red flags: severe bleeding, nausea/vomting, etc…
Home situation
Education/employment Indications for high-dose estrogen OCPs
Activity/alcohol - Uncontrolled menorraghia
Depression/drugs - Taking other enzyme inducing (p450) drugs such as
Suicidality/smoking anti-epileptics
- Low dose pill failure
Management
- Partner's age: >2 years age difference is not On OCP wants to change to HRT
acceptable;
- Gillick's test: if you are able to show me that you're A 45-year-old lady came to your GP clinic and she is on OCP.
able to understand what you are saying, and at the She wants to change to HRT because she has heard about it
end of the conversion you are able to understand what from her friends.
I said, then I can give you the script. (how are you
going to use the OCP? What will you do if you missed *48-year-old px with irregular periods and husband had
the pill?) vasectomy;
- Will not protect against STIs.
- Advise on 7 days row. Use other contraceptive *53-year-old with amenorrhea for last 2 days (years)/with history
methods for the first 7 days. If you missed the pill or of breast cancer.
have had any nausea, vomiting, diarrhea then use
barrier method Task
- Reading materials a. History
- Review again for 3 months b. Management
c. Answer her questions
Breakthrough Bleeding with OCPs
History:
Case: Your next patient in GP practice is a 22-year-old female ‐ 5Ps:
who started using Microgynon 30 because she wants to start ‐ Vasomotor symptoms: hot flushes? Night sweats?
sexual relationship with her partner in the near future. She has Palpitations? Lightheadedness/dizziness? Migraine?
had some per vagina spotting over the last 4 weeks and is ‐ Urogenital: dyspareunia? UTI? Vaginal dryness?
concerned. Decline in libido? Bladder dysfunction (dysuria)?
Stress incontinence/prolapse?
Task: ‐ Psychogenic: irritability, depression, anxiety/tension,
a. History (spotting x 2-3 for 4 weeks) fearfulness, loss of concentration, tearfulness, loss of
b. Diagnosis concentration, poor short term memory, unloved
c. Management feelings, mood changes, loss of self-confidence
‐ Frequent headaches? Migraine? FHx: CVS, cancers,
History osteoporosis? Breast lumps? History of heart
- Could you talk more about it? Do you take the pills at disease? Hypertension? Unusual bleeding? Pills? Any
a regular time? Have you skipped or missed a pill? weight gain? Nausea/vomiting?
Smoking? STDs? Are you taking any other ‐ SADMA: smoking? Medications: steroids?
medications (anti-epileptics/antibiotics)? Recent ‐ FHx: Premature menopause
diarrhea or vomiting?
- Any chance you could be pregnant? Partner? Pap ‐ Contraindications of HRT:
smear? o Estrogen-dependent tumor (endometrial,
breast cancer)
Factors for breakthrough bleeding: o Recurrent thromboembolism
- Not taking pills at the same time (decreases efficacy) o Acute IHD (absolute)/history of CHD
- Missed pill (relative)
- Smoking o Uncontrolled hypertension
- Medications o Active liver disease
- AGE o Pregnancy
o Undiagnosed vaginal bleeding
Management o Otosclerosis? Intermittent porphyria
- What you have is a case of breakthrough bleeding
which occurs in between periods. It could be a light Investigations: FBE, LFTs, BSL, Lipid profile, U/C/E, TFTs,
spotting in your case or a heavy bleeding. It is a Estrogen/FSH/LH
common side effect of OCPs.
- There are several reasons why breakthrough bleeding Management:
can happen: if not taking pills at the same time (15 ‐ From the history, you are not a candidate for HRT.
minutes), should not skip pills, smoking, medication or However, I would like to request for some medications
STDs, or AGE. For some women, the low-dose pill to check if you’re already reaching menopause. HRT
does not contain enough estrogen to maintain the is not a contraceptive method. Both HRT and OCPs
stability of the endometrium (lining of the uterus) do not prevent STIs.
which causes breakthrough bleeding.
51 
 
‐ Menopause is a natural phenomenon. One of the Physical examination
things I am concerned about menopause is ‐ General appearance and BMI
osteoporosis and heart disease. It is advisable to ‐ Vital signs
change lifestyle: maintain healthy weight, adequate ‐ Dysmorphic features of cushing syndrome, PCOS
relaxation and exercise, do pelvic floor exercises ‐ Palpate thyroid
regularly, reduced smoking, caffeine, alcohol intake, ‐ Auscultate chest and heart
increased exposure to sunlight. ‐ Abdomen to palpate renal or suprarenal mass and
‐ Some other methods of contraception: barrier, IUCD, listen to bruit
implanon, injectables, etc… during next consultation ‐ Pelvic exam:
o Inspection: discharge, bleeding
Additional information: o Bimanaual exam: position and size of
‐ Ways to know: organize LH and FSH (30-40) Æ most uterus, tenderness, cervical excitation
likely menopausal; if FSH and LH are that high Æ ‐ Urine dipstick, pregnancy test and BSL
stop OCP and get symptoms Æ HRT; require regular
follow up. Management
‐ 45 Æ too early; but requires support; usually high ‐ From the history and examination, the most important
dose HRT given; finding is that of a high blood pressure. Have you ever
had your blood pressure checked before? Usually, at
OCP-Induced Hypertension your age, having a high blood pressure can be due to
a number of causes. Most likely, it can be related to
Case: You are a GP and a 26-year-old female comes to your the use of the pill as the headaches that you have
clinic asking about the chances of becoming pregnant within the started along with the use of the pill. I still need to rule
next 6 months out other causes of hypertension such as smoking,
any problems with the blood supply to the kidneys,
Case Before: Patient coming to you who is a heavy smoker and certain growths in the adrenal gland related to the
has hypertension. She is on OCP. kidney, cardiac problems, and the like. I would do
some investigations like FBE, U&E, Urine MCS, ECG,
Task uric acid level, lipid profile, LFTs, TFTs, blood group,
a. History (regular 2-3 days, 28 days, on the pill, pap rubella antibody, infection screening.
smear n, no previous pregnancies/miscarriages, non- ‐ We still need to check your BP during the next visit.
smoker, social drinker, mom with DM) However, I want you to please stop using the pill.
b. Physical examination: BP 155/95, Around 2% of females, especially those who have
c. Diagnosis and management family history of high BP, those who are overweight,
>35 years old, and smokers can develop high blood
History pressure due to OCPs. Some women get high BP
‐ I can see from the notes you wish to become pregnant from the progesterone component of the pill. Usually,
in the near future. Congratulations on your decision. this rise in blood pressure is only seen with the
‐ Please tell me more about your periods? Are they systolic component. The good news is that it is
regular? How many days of bleeding? How many completely reversible. However, you need to stop
days apart? Are your periods heavy? Are they painful? smoking and adopt a healthy lifestyle to reduce this
Any spotting in between? risk to minimum. Meanwhile, you may use another
‐ I understand you’re sexually active, since when? What form of contraception, probably condoms. Becoming
form of contraception do you use? What type of pill pregnant at this stage might further complicate your
are you on? Since when? Have you had any side condition, so my advice is once the results are back
effects from the pill (nausea, weight gain, and your BP is normalized, you can plan for the
intermenstrual spotting)? Have you or your partner pregnancy. I would like to see you in one week’s time
ever been diagnosed with a STI? At the moment, do with the results of the tests. Please come back if you
you suffer from any vaginal discharge? Any bleeding develop further headaches, visual problems, fainting
or itchiness down below? Have you ever had pelvic or dizziness.
infections before? Have you had any
pregnancy/miscarriages/gynecological surgeries Post-pill amenorrhea??
before?
‐ When was your last pap smear? What was the result? Case: Your next patient in GP practice is a 30-year-old woman.
Have you had gardasil? She did not have periods for the last 2 months. She is on
‐ PMHx: diabetes, hypertension, kidney disease, MIcrogynon 30.
infections, liver? History of clotting problems in you or
your family? Task
‐ SADMA? a. History (on the pill, periods stopped GRADUALLY,
‐ How’s your appetite/sleep? Any recent history of b. Physical examination
fever, cough, diarrhea, tummy pain? How do you c. Diagnosis
consider your weight to be? Do you know your blood d. Management
group?
‐ Any FHx of fertility problems? Pregnancy related Secondary Amenorrhea
problems? Diabetes? High blood pressure? ‐ Pregnancy (breast tenderness, spotting, early morning
‐ Headache: how frequent, since when? Have you N/V)
noticed any association with particular food or time of ‐ PCOS (weight gain, acne, hirsutism, irregular periods)
day? What do you take to relieve pain? Any ‐ Hypthyroidism (weather preference, puffy face,
associated N/V/abnormal sensations/visual edema, mood)
disturbance? ‐ Eating disorder/exercise induced
‐ Hyperprolactinemia (breast discharge, medications,
headache, nausea and vomiting
52 
 
‐ Asherman syndrome (gynecological procedures/D&C) History
‐ Stress - Can you please tell me, what is your main concern?
‐ Premature ovarian symptoms Do you think your daughter has started her periods?
‐ Post-Pill Amenorrhea Did you notice any breast development? Since when?
Have you noticed any hair growths in the armpits over
History the pubic area? Since when? I understand she is on
‐ I understand you have not had your menses for the phenytoin/carbamazepine? Any side effects? Who
last 2 months. Any chance you might be pregnant? takes care of her medications? Since when did she
What’s your LMP? Do you have symptoms like breast last see her neurologist.
pain, N/V, spotting? - Please tell me more about her mental retardation.
‐ Pills: any problems with that? Do you think you might Was she born this way? How would you describe her
have missed your pill anytime? Are you taking it mental age to be? Is she able to do daily life activities
regularly? Did you have any diarrhea or vomiting? Are like eating, dressing, and going to the toilet? Does she
you on any other medications? need partial or complete supervision? Who takes care
‐ Review of systems: hirsutism of her most of the time? Do you experience any
‐ Partner? Pap? Gardasil vaccination? difficulties while taking care of your daughter? How is
‐ Any previous pregnancies? her school performance? Any problems at school? Is it
‐ Any Family history of premature ovarian failure or a special school? Do you think she might already be
cancers? sexually active? Have you discussed anything with her
‐ SADMA? PMHX like Periods? Contraception? Previous medical or
surgical illnesses? Any concerns about her growth?
Physical Examination Do you have enough support at home, from family
‐ General appearance friends and partner? Financial problem?
‐ Vital signs and BMI
‐ Visible hirsutism, acne, puffy face or edema Counseling
‐ Vision: visual fields, funduscopy, visual acuity - I understand from the history that your daughter has
‐ Neck: thyroid enlargement not had her periods up till now. However, some
‐ Breast examination: nipple discharge degree of breast development has occurred so we
‐ Abdomen: masses, tenderness might expect that she will start menstruating soon. It is
‐ Pelvic exam: very good that you have come at this time to discuss
o Inspection: discharge, atrophic vagina contraception. However, no form of contraception is
o Speculum: cervical os, bleeding required until periods start. Usually, we recommend
o Bimanual: size of uterus, adnexal masses, oral contraceptives that might be most suitable for her.
CMT Because she is on antiepileptic medications, we might
‐ Urine dipstick, BSL, Pregnancy Test need to give her a pill with high dose of estrogen.
Please understand that the pill prevents pregnancy
Diagnosis and management only and not STDs. If you find that giving a pill
‐ There is no abnormality on physical examination. everyday is inconvenient, we can give her injections of
According to your history, the most likely cause of not depo-provera every 3 months. However, with
having the periods is endometrial atrophy secondary prolonged use, it will produce side effects including
to the pill. reduced density of bones as well as problems with
‐ However, we need to rule out pregnancy. The only periods. There are other options as well like implanon
possible reason is one of the hormones and IUCDs. However, the management is better
(progesterone) in the pill is causing thinning of the suitable for females who can look after themselves.
lining of the womb. - Can we remove her womb instead? The oral pill is the
‐ DIAGRAM best option for your child because you are already
‐ Do not worry. It is a reversible condition. At this stage, giving her some medications and you just need to add
we will stop the Microgynon 30 and you can use other one more. Regarding permanent sterilization, it is
forms of contraception at this time or I can shift you to usually not allowed for girls under the age of 18 years
Microgynon 50 or we can use the triphasic pills. Most without approval from the court. Please understand
likely your periods will return. In case you don’t or that being mentally disabled does not deprive your
you’re really concerned, I can refer you to the daughter from the right to be treated just like other
gynecologist for further investigation. people. We, as doctors, only prescribe something if it
‐ Reading material. Referral. Review. is in the best interest of your daughter. I understand
you are concerned; however, I am sure you would be
Request for sterilization for a disabled person/Contraceptive equally worried if your daughter suffered from any of
advise for disabled the complications of this surgery which includes
complications with anesthesia, bleeding, infections,
Case: You are a GP and a mother of 13-year-old child comes to and long-term effects on her bone growth and
you. She is intellectually disabled and epileptic. She is on hormonal imbalances.
carbamazepine. She wants your advice because the child goes - I gave the consent appendicectomy. Why can’t I do it
to school for both boys and girls. She is worried about now? Appendectomy is a medical emergency where
contraception and the risk of pregnancy. the decision is taken on medical grounds. If you like,
you can contact the family court or the guardianship
Task board. They have the legal authority to allow this kind
a. Relevant history of procedure.
b. Address mother’s concerns
53 
 
Depo-Provera Counseling History
‐ Why do you want to change? Who suggested
Case: A 25-years-old female is in your GP clinic and who wants implanon? Any side effects of OCP? Any chance you
to have depo-provera. are pregnant now? Did you have previous STIs? Pap
smear
Task ‐ Previous pregnancies/miscarriages? How are your
a. Advise about depo-provera cycles? When was your LMP? Any medical conditions
and FHx of hypertension, diabetes?
Counseling ‐ SADMA?
- It is the only injectable IM contraceptive available in
Australia and it has progesterone in it. The dose is Counseling
150mg by deep IM injection in the first 5 days of ‐ The implanon, as you know, is a small device that
menstrual cycle and same dose is given every 12 goes below the skin in the non-dominant upper arm
weeks. under local anesthesia. It contains a certain hormone
- Do you have any migraine? Stroke? Cancer? Any (etonogestrel) that will cause 2 things: inhibits
undiagnosed vaginal bleeding? Hypertension? Heart ovulation and increases the viscosity of the cervical
disease? Diabetes? Lipids? Liver disease? mucus. It is a very safe contraceptive method. The
- 5Ps: periods, pap smear, do you want to be pregnant failure rate is <1% and it lasts for 3 years.
in the next 12 months? ‐ Upon removal, most women will ovulate during the
- When the woman has depo-provera in the body her first month. The procedure needs to be done by a
own hormone production is switched off. Because of trained personnel.
this the ovaries will not release eggs thus pregnancy ‐ Advantages: convenience, rapid reversibility, available
is prevented. It is a highly effective method of at low cost through the PBS systems, suitable for
contraception more effective than the combined pill women with CI to estrogen
and failure rate is 1%. ‐ Absolute contraindications: pregnancy, undiagnosed
- The advantages of depo-provera are: It is highly vaginal bleeding, active thromboembolic disease,
effective and therefore has low failure rate. It can present or history of severe liver disease,
relieve pre-menstrual tension and period pain. It is progestogen-dependent tumors, breast cancer,
also likely to cause some reduction in risk of ovarian hypersensitivity to components of implanon
and endometrial cancer, and endometriosis. As it is ‐ Relative contraindications: long-term use of liver
given every 12 weeks, no other effort or remembering enzyme inducing drugs, past or family Hx of
is required. thromboembolic disease, obesity (>100kg Æ efficacy
- The disadvantage is that you have to take injection is less), women for whom regular periods are
every 3 months. Once the injection is given, the important
hormone cannot be removed and if you want to stop ‐ Side effect: Menstrual disturbance is the most
depo-provera you have to wait for the hormone to common reason for removal
wear off. In some women, it can take 6-12 months for o bleeding approximating normal (35%),
periods to return. There is a concern about the risk of infrequent bleeding (26%), amenorrhea
thinning of bones if woman is using depo-provera for a (21%), frequent or prolonged bleeding
long period of time. (18%)
- Side effects may include reduced periods due to low o breast tenderness, fluid retention, weight
level of hormones. After 2-3 injections, most women gain, skin disorders (improve), mood change
will have no periods at all because there is no lining ‐ Effective immediately if inserted during day 1-5 of the
building up to shed. Some have intermenstrual patient’s menstrual cycle; if not, then important to
bleeding which is usually light and irregular or have ascertain the patient is not pregnant and alternative
heavy bleeding which can be controlled by hormone contraception should be used for 7 days after
treatment. A small amount of weight gain can occur. insertion.
There can be headache, abdominal discomfort and
mood changes. Women who have increased Emergency Contraception after Rape
incidence of depression can have reduced interest in
sex. Case: You are a GP and 18-year-old Samantha came to your
- Contraindications clinic asking for emergency contraception and advice.
o Bleeding disorders or taking anticoagulant
medication Task
o Undiagnosed vaginal bleeding a. Explain methods of emergency contraception
o History of some forms of cancer b. Manage the case
o Serious medical conditions
o Already pregnant or those who want to Case: Rosie aged 24 years presents to the ED of the local
become pregnant within 12 months hospital where you are working as an intern. She tells you that
- Not recommended for greater than 2 years. she was sexually assaulted by a person to whom she met in a
pub. She is very distressed and teary. On further questioning
Implanon Counseling she discloses that she doesn’t know this person and had never
met him before. He offered her a lift home and then stopped the
Case: Your next patient is a 19-year-old female previously on car in a lonely place and assaulted her. Rosie is an overseas
OCP and now requests implanon. university student and lives in a shared accommodation and had
no other medical or any surgical problems.
Task
a. Relevant history (friend mentioned; no problems Task
except missed pill) a. Further relevant history
b. Advice patient and answer questions b. Physical examination
c. Management advice
54 
 
- I understand from the notes that you are here for Counseling
emergency contraception which is available OTC. Is ‐ REVEAL THE CONCERN: why have you decided
there some special reason to see me today? that?
- I am sorry to hear that, but don’t worry there is a lot of ‐ Are you in a stable relationship? Have you completed
support and you are not alone at this moment of crisis. your family? How’s your general medical health? Any
- Confidentiality statement. medical/surgical problems in the past.
- Would you like to take any legal action? (No doctor. ‐ WARNING: I would like to inform you that reversal
This man is known to my family and I don’t want to can be done, but has a very low successful rate, and
make a fuss about it.) I respect your decision but I tubal ligation is considered a permanent form of
would like to get samples and keep it in the hospital sterilization
just in case you will change your mind later. ‐ COUNSELING: I will tell you what the method is
- Were you injured anywhere else? about, advantages and disadvantages, and other
- Menstrual history: When was your LMP (3 weeks alternative methods. Tubal ligation can be done by 2
ago)? How are your periods? Are they regular? What methods done by specialist under general anesthesia.
is the cycle? Bleeding? How many days apart? The first method is either the specialist can cut the
- Sexual history: Do you know if the man suffered from tubes and tie them together so that the sperm and ova
any STIs (No)/Did you see any discharge on his do not meet, or can put clips. Success rate is more
private part? Are you sexually active? Are you in a with the first method, whereas with the second
stable relationship? Are you using any form of method, there are chances that clips may dislodge.
contraception? Have you or your partner ever been Failure rate is 1:300 which means one in every 300
diagnosed with STIs? Pap smear women who gets the procedure gets pregnant.
- Any history of clotting, hypertension, migraine, Disadvantages of tubal ligation: a. reversal rate is low,
undiagnosed vaginal bleeding, breast cancer? and reversal is not covered by medicare, b.
anesthesia risk/complication, c. ectopic pregnancy, d.
Examination will not protect from STIs.
- General appearance ‐ How long in hospital; 1-2 days.
- Vitals ‐ ALTERNATIVES: I would like to give you some
- Pelvic examination with consent information about the other methods you can use such
o Inspection: sign of injury, vaginal secretions, as IUDs, implants, depo-provera where compliance is
consent to take low and high vaginal swabs not a major issue. For men, there is also a procedure
for STD screening called vasectomy. The advantages are: a. simple, b.
- Chest, heart, abdomen to check signs of assault done under local anesthesia, c. less complications, d.
- Urine dipstick lower failure rate
‐ QUESTIONS:
Management o Will it affect my sexual life? No. It makes it
- We need to take blood samples for HIV, syphilis, better because you’re not scared about
Hepatitis B&C, HSV and take urine sample for PCR pregnancy.
and Chlamydia o When can I resume sexual life? Once
- I would give you antibiotic coverage: Azithromycin 1g effects of operation is over.
SD o Is there any effect on my periods? Not
- I would like to refer you to a psychologist or counselor really, but there are some studies which
for support (rape crisis team). have shown that if more of the fallopian tube
- Let’s talk about emergency contraception. The first is cut, it leads to heavier bleeding. Not yet
method is levonorgestrel (Postinor). This is a POP. 2 proven.
pills (0.75mg each) Æ 12 hours apart or 1 pill (1.5mg) o Will I gain weight? No.
given up to 5 days but most efficient if taken within 72 o What if I need kids later? The cut tubes may
hours. Efficacy is 85%. be rejoined by microsurgery, but there is no
- The next method is combined pills or Yuzpe method guarantee of reversal of fertility. Pregnancy
75% efficacy ([50mcg estrogen and 250 mcg rate after reversal varies from 30-80% and
progesterone] 2 tablets now then 2 tablets 12 hours that depends on the technique. The simple
apart) or copper IUDs with a failure rate of <1% if clip method gives better chance of reversal.
used within 72 hours. Regardless, it is considered a permanent
- There are chances of getting pregnant even after method and shouldn’t be entered lightly.
taking the emergency pills. Therefore, I would like to ‐ CLOSURE: I would recommend you to go home and
review you after 2-3 weeks to do pregnancy test discuss what we have talked about today and if you
especially if you miss you period. have decided, I will give you a referral letter.
- Most common side effect is nausea and vomiting. If
she develops it, she needs to take the drug again.’ Vasectomy
- Reading material
Case: A couple comes to your GP clinic. They have completed
Tubal ligation their family and want to discuss with you about vasectomy.

Case: 32-year-old lady comes to you in your GP practice. She Task


has 3 kids and would like to go ahead with tubal ligation. a. Relevant history (family complete, and wife doesn’t
want to take OCPs
Task: b. because of side effects)
a. Counsel the patient regarding ligation c. Explain the procedure
b. Answer patients questions d. Complications
e. Follow up
55 
 
Relevant history INFERTILITY
- How much do you know about vasectomy? Did you
make this decision after discussing with your wife? Primary Infertility
What is your age? Are you married? How many
children do you have? What are the ages of your Case: Your next patient in GP practice is a young couple who
children? comes to you because they have been trying to conceive for the
- How is your general health? Any previous medical last 15 months. They are happily married for 3 years and have
conditions (DM, breathing disorder, hypertension?? not sought any medical attention before.
Previous operations especially in your private parts?
SADMA? Task
- Any problems or issues with your personal life? a. History (26 year old, no medical problems, most of the
- If person is unmarried, <35 years of age, says he time in overseas, sexual contact 1x/week, menarche
doesn’t have a children, emotional crisis or spouse is 13 years old, on pills, STDs, surgeries or
not involved Æ be very careful gynecological problem)
b. Counsel patient about management
Procedure
- I would like to explain the procedure to you, its Causes
complications, how effective it is and important issues ‐ Male factor
regarding reversibility. ‐ Ovulatory (PCOS, HPA, POF)
- It is the most common method of sterilization in men. ‐ Tubal (PID, tubal damage, pelvic adhesion)
It is a simple operation that can be done under local or ‐ Maternal age (rate of fertility declines >30)
general anesthesia. It usually takes about 30 minutes. ‐ Endometriosis
Two small cuts are made on each side of the back of ‐ Coital problems (frequency, erection, problems,
the scrotum or one cut is made in the middle. The psychological factors)
tube that carries the sperm (vas deferens) which lies ‐ Cervical (mucus) or uterine factors (adhesions, polyps
just below the skin is picked up and cut. About 1 cm of or myomas)
it is removed. The ends are tied off and burned with a ‐ Unexplained
hot needle. This blocks the flow of sperm so when you
ejaculate, the semen will be free of sperms. Investigation
- What happens to the sperm? They are still produced ‐ Hormones: FSH, LH, midluteal progesterone, TFTs,
in the testicles and lie around the blocked tubes for estrogen,
around 3 weeks. After that, they become non- ‐ TVS for structural abnormalities
functional and absorbed. ‐ HSSG
- How effective is it? 1 in 500 vasectomies fail because
the tubes somehow manage to rejoin. History
- Complications: bruising, hematoma, bleeding, ‐ Confidentiality
infection Æ but usually settles very quickly. You will ‐ Have you ever been pregnant before? Any history of
be given pain killers. The sperm granulomas, which miscarriages? Pregnancy from any previous
are brought about by blockage of the semen usually relationships? How long have you been trying to
clear up by themselves. conceive? Does your husband have kids from
- It can take about 15-20 ejaculations to clear all the previous relationship?
sperms from the tubes above the cut. About 2-3 ‐ Periods: menarche, regular, how many bleeding, how
months after the operation, you will have 2 separate many days apart? How is the flow? Do you pass
sperm counts to make sure semen has no sperm. clots? Any bleeding in between period? When was
Until that time, it is important to use some form of your LMP? Do you get any severe pain when you
contraception. have your periods? Any pain on intercourse? Have
- When to start sexual activity? Normal sexual activity you noticed any abnormal hair growth on your body?
can be started 4-8 days after vasectomy. Have you gained weight recently? Have you noticed
- Can it be reversed? Consider it to be permanent and any milky discharge from the breast? Any problem
irreversible procedure. The cut tubes can be rejoined with your vision? Have you ever been diagnosed or
by microsurgery but there is no guarantee of regaining screened for STIs? Any history of pelvic infections?
fertility. Only 40% chance that it can lead to Any history of previous surgeries or gynecological
pregnancy. procedures? Have you ever used any method of
- Written permission of the wife is preferable. Discuss contraception? What was it? Any problems because of
other methods briefly. that? Any history of diabetes, thyroid or increased
- Is it going to affect my sexual function? NO. It will blood pressure? Any FHx of infertility from your side or
make no difference. Rather, it can be improved your partner’s side? Any issues with your married life?
because the worry of conception is removed. Before How often do you have intercourse? Are you aware of
you go for the procedure you can take your time to your fertile/infertile days? Any stress? Are you a
reconsider it and avoid strenuous activity for 4-7 days happy couple? Do you have problems with your
after the procedure. waterworks or bowel? Do you exercise a lot?
- There is no known association with prostate or SADMA?
testicular cancer. Vasectomy doesn’t help to prevent
STIs. Management
‐ I could not find anything positive in the history other
than the frequency of your intercourse which could be
the cause of not having a baby. Do you know which
days you are fertile? If your cycles are regular we can
56 
 
calculate it, or check your temperature, or observe the Examination
consistency of mucus (time of maximum vaginal ‐ Anxious; vital signs normal; focused examination on
wetness corresponds to day of ovulation) to know the abdomen: no visible/palpable mass or tenderness in
time of ovulation. At this time, it is recommended you abdomen; inspection and speculum normal;
have more frequent intercourse or at least 3x a week. uterosacral nodularities and tenderness on bimanual
‐ Still, I would like to organize some investigations to examination; may have fixed retroverted tender uterus
rule out the other causes of infertility. I would advise ‐ Pelvic examination:
for your partner to have semen analysis. For you, we o inspection: discharge, bleeding, redness,
will start with FBE, TFTs, hormonal assay like lacerations, mass, ulcers,
midluteal phase progesterone, FSH, LH, prolactin, o speculum: Vagina and cervix: cervical
estrogen, sperm antibody screen, TVS, and if motion tenderness (PID/ectopic pregnancy);
required, the specialist might consider doing cervical os (miscarriage)
hysterosalpingography (HSSG) or hysteroscopy. I o bimanual examination: site, size, shape,
would refer you to the gynecologist/infertility clinic for consistency, mobility, and adnexal
further evaluation and management. masses/tenderness
‐ Do not worry. I understand that it is a very difficult time
for you but I am here to help. Even if we find Investigation and management:
something, a lot of options can be done for you: ICSI, ‐ Most likely you have a condition called endometriosis.
IVF, surrogacy, or adoption. Did you ever hear it before?
‐ Reading material. Referral. Review. ‐ Explain retrograde menstruation and draw diagram.
The tissue lining your womb is deposited in unusual
ENDOMETRIOSIS AND ADENOMYOSIS locations by backing up of menstrual flow into
Fallopian tube, ovaries, abdominal cavity or other
Endometriosis organs of the body. These abnormally located tissues
form nodules and adhesions that respond to your
Case: A 30 years old lady comes to your GP clinic complaining hormones during periods causing pain.
of dysmenorrhea for the last 3 months. She tried using OCP but ‐ Start with painkillers
was not relieved. ‐ Refer to OB gyne for usd but gold standard is
laparoscopy. It is a tube with camera for direct
Task visualization of your tummy to see these nodules to
a. History make a definite diagnosis
b. Physical examination ‐ Treatment Options
c. Investigations and management o Medical: stop hormone production
(progesterone oral/IM); GnRh agonist x 6
Differential diagnosis months; danazol - treatment of choice
‐ Endometriosis according to JM;
‐ Chronic PID o Surgical: laparoscopy -- definitive
‐ Adhesions (previous surgery) investigation and laser surgery performed
‐ Fibroids (submucous myoma) when needed; laparotomy;
‐ Uterine polyps ‐ Pregnancy: helpful because it creates a state of
‐ Ovarian masses menopause
‐ IUCD ‐ Support groups
‐ PCOS ‐ Family meeting and refer to counselor
‐ Reading materials; review;
Location
‐ Ovaries: 60% INFECTIONS OF THE GENITAL TRACT
‐ Uterosacral ligaments: 60%
‐ Pouch of Douglas: 28% Vulvovaginitis
‐ Causes adhesions and fibrosis and during
menstruation would cause bleeding as well due to Case: A father came with her 4-year-old daughter who has had
hormonal stimulation; a 2nd episode of painful urination over the last year. During the
first episode, the daughter had some yellow vaginal discharge
as well. Now, on examination, everything is normal except there
‐ Symptoms: dysmenorrhea, dyschezia, dyrsuria, is mild erythema of vulva and vagina. Urine dipstick is clear.
dyspareunia, infertility
Task
History a. History
‐ Can you tell me more about the pain? (dull tummy b. Diagnosis and management
pain before menstruation and more severe during
menses for 6 mos; tried ocp prescribed by gp but not Differential Diagnosis
relieved severity 4/5; may radiate to thigh/back) - Vulvovaginitis
SORTSARA? Mass in tummy? Previous history of - Foreign body
PID? Previous surgery? Fever? Vaginal discharge? - Child abuse
5Ps; any painful intercourse, or defecation? Urine? Do - Allergy
you have any kids (no but been trying to conceive); - Infestation of pinworm
Pap smear? Any unprotected intercourse? Any - Type I diabetes Mellitus
previous STD? Past history of pelvic surgery? HOW - UTI
IS THE PAIN AFFECTINGYOUR LIFE? SADMA? FHx
57 
 
History - Associated features: fever, bleeding, tummy pain,
- Describe the discharge? Is it thick or thin? Is it foul- itching?
smelling? What about the amount? Any fever? - Relation to periods?
Frequency of urination? Any change in color of urine? - LMP? How many days? How many days apart?
Is it a smelly urine? Is she toilet-trained? Any change Bleeding inter-menstrually?
in toilet training (bed wetting)? Is she drinking more - Are you sexually active? Are you in a stable
water than usual? Who looks after her? Does she go relationship?
to childcare/kindergarten? Do you think she might - Is your partner suffering from any STI?
have put something in her private area? Any abnormal - Method of contraception? Number of previous sexual
behavior like sexual plays or playing with the partners?
genitalia? Is it possible that she is left unattended or - History of previous STIs?
unsupervised? Does she scratch her bottom at night? - How are your waterworks? Burning? Frequency on
Have you changed her soap recently? Does she take urination?
a bubble bath? Does she go for swimming? - Have you been pregnant before? Any chance you are
- BINDS pregnant now?
- FHx of asthma or allergies - PMHx: DM, HPN, previous gynecological
surgery/procedure
Examination: - Recent use of cream or pessaries (consider allergic
- General appearance rxn)
- Vital signs and BMI - Latest pap smear?
- Abdomen: distention, mass, scratch marks, sign of - Meds taken? Steroids?
abuse - Vaccination (gardasil)
- Genital inspection: redness, discharge, scratch marks - Smoke/drink/recreational drug use
- Urine dipstick and BSL
Physical Examination:
Management - General appearance: pallor, jaundice, dehydration,
- Your child has an inflammation of the private part BMI
called vulvovaginitis. It is a common condition in this - VS: temperature, PR, RR, BP (postural drop)
age group (2-8 years). In this age, there is lack of - Ausculation of chest/heart
estrogen so the mucosa (lining) of the vagina is thin - Abdominal examination: tenderness (posterior fornix
and irritable. When the child scratches, it becomes of the vagina), organ enlargements, mass, bowel
infected easily. It usually resolves by itself. I would sounds
also like to do urine microscopy and culture and take a - Inspection of pelvic area – bleeding, discharge (color,
swab of the discharge. quantity, and smell), scratch marks, warts
- There are risk factors: FHx of eczema, bubblebaths or - Bimanual palpation – adnexal mass, cervical
salt baths, nappies, irritating soaps, wet swimsuits, excitation, check size/position of uterus and cervix
sand from the sandpit, and overweight - Sterile speculum examination – check where the
- It is not a serious condition. Avoid bubblebaths, use discharge is coming from and position/condition of the
cotton underwears and loose clothing, general vulval cervix; take a swab and send for culture and wet
hygiene, wipe bottom from front to back to avoid mount
infection, warm shallow bath with a cup of vinegar, - Get urine dipstick/finger BSL/PT
advise zinc cream or castor oil to relieve redness
- When to refer: if foreign body is suspected and if Management
general measures fail and condition is persistent - Most likely from the history and PE, what you have is
- Complications: vulvar adhesions, UTI a vaginal infection called trichomonas vaginitis
- Reading material - Caused by a parasite called T. vaginalis, usually
- Review transmitted thru sexual contact
- Most common STI worldwide
Trichomonas Vaginitis - Common in females of child-bearing age
- Possible to carry organism without signs and
Case: You are a GP and young female came in with greenish symptoms
vaginal discharge. - Gives symptoms like itching, burning of urine, watery
greenish discharge with fishy smell
Task: - Infections in male are asymptomatic
a. Focused History - Diagnosis is by visualizing the organism within the
b. Physical Examination vaginal secretion under the microscope
c. Management - Treatment:
o Metronidazole 2g SD (+ antiemetic – due to
Vaginal discharges: SE of N/V, metallic taste) or 400 mg BD x 5
- Whitish, curd-like – candida albicans days
- Grayish – bacterial vaginosis o Alternative for pregnant women:
- Brownish – atrophic vaginitis Clotrimazole
- Greenish – trichomoniasis - Important to prevent complications: UTI, PID,
Recurrent trichomoniasis – infertility
Focused History: - Higher chances of developing other STIs especially
- When did the discharge start? Continuous? HIV Æ important to test for other STIs (consent)
- Describe the consistency (sticky or watery), color and - Practice good genital hygiene – wash vaginal area
smell before and after intercourse
- How many pads do you use per day? Are they - Do not share towels
soaked? - Remember to shower after swimming
- Practice safe sex with condoms
Advise to bring partner for consultation and treatment
58 
 
Vaginal Discharge - Long-term use of steroids and antibiotics
- Obesity
Trichomonas Candida Bacterial - Wearing tight clothing
vaginalis vaginitis vaginosis
Itchy + + + History
Dyspareuni + + - - I understand that you have recurrent white vaginal
a discharge. When did this episode start? What is the
Discharge fishy frothy curdy fishy+++ color of the discharge? Any blood stains? Is it smelly?
and green profuse thin Is it itchy? Is it sore down below? How many attacks?
and grey How was it diagnosed? Did you take anything for that?
Vaginal Organism Hyphae clue cells Which treatment were you on? Is the discharge
Swab whiff test related to your period or intercourse? Any tummy
Other Ph>4.5 pains?
investigation - Are you sexually active? Are you in a stable
In Very safe PROM relationship? Have you or your partner ever been
pregnancy dangerous diagnosed with STDs? What contraception are you
Cause: using? Do you have any problems with the OCPs?
preterm Does your partner have any symptoms?
labour, - Periods: LMP? Are they regular? How many days of
premature bleeding? how many days apart?
rupture of - Pregnancy: Any chance you can be pregnant now?
membrane Any previous pregnancies?
Treatment Tinidazole Local Metronidazol - Pap: Are you regular with your pap?
Pregnancy: Clotrimoxazol e - Any possibility of using local perfumes or local creams
Metronidazol e (pessary) Pregnant: down below? Some people use sexual toys, do you
e Clindamycin happen to use them?
Treat + - - - PMHx: Any medical history of long-term use of steroid,
partner diabetes or any long-term antibiotics?
- What is your profession? Do you wear tight jeans?
Gonorrhea Chlamydia - FHx: diabetes, cancers?
- SADMA?
Mucopurulent + +
Investigation 1st void urine PCR same
Physical Examination
or endocervical
- General appearance and BMI
swab
- Vital signs
In pregnancy PROM same - Chest and heart
Pneumonia - Abdomen: masses or RIF/LIF tenderness
Ophthalmia - Pelvic examination: nature of discharge, color, smell,
Treatment Ceftriazone + Azithromycin 1 thick, blood stain, vulvar erythema; per speculum
Azithromycin dose 1 g stat cervix is healthy with discharge; per vagina
Treat partner + + examination for any CMT, adnexal masses
- Urine dipstick, BSL, pregnancy test (optional)
Recurrent Moniliasis/Candidiasis
Diagnosis and Management
Case: You are a 25-year-old lady complaining of recurrent white - From the history and examination you have a
vaginal discharge. She was diagnosed with monilial infection condition called recurrent moniliasis or candidiasis. It
and was given treatment for that. She has now come to you for is a fungal infection caused by Candida albicans. It is
further advice. a common condition and there are some risk factors
leading to repeated attacks. The risk factors are long-
Task term use of OCPs, DM, pregnancy, obesity, long-term
a. History (on-and-off for the last 3 months, given vaginal use of steroids, antibiotics and wearing tight clothings.
tablets by GP without relief, on OCP, not pregnant, no - At this stage, I would like to check the BSL, FBE and
history of long-term use of steroids or antibiotics, or do swab. I would advise you to stop OCPs and I can
obesity) book another appointment to discuss the alternative
b. Examination (+ whitish curd-like discharge with vulvar methods of contraception. Until then, I would advise
erythema) you to use condoms. It is not STD but it is best to
c. Diagnosis and management abstain from intercourse until the condition resolves.
- I will shift you to oral antifungals with fluconazole
Differential Diagnosis 50mg or Itraconazole 100 mg OD for up to 2 weeks
- Candidiasis (up to 6 months depending on the severity) or vaginal
- Trichomonas vaginalis Nystatin if not comfortable with oral formulation. For
- Foreign body remission or long-term prophylaxis we can give
- Atrophic vaginosis weekly fluconazole for a few months.
- Cervical ectropion - Before taking any antibiotics, advise GP that you are
- Malignancy undergoing treatment.
- Wear loose undergarments. Keep the area dry and
Risks thoroughly dry after bathing. Don’t use any creams or
- Long-term OCP perfumes in the vagina/vulva. Advise vaginal douche
- Diabetes
- Pregnancy
59 
 
(1tsp bicarbonate in 1 L water). Bathe genital area Investigations
gently 2-3x a day for symptomatic relief. Thoroughly ‐ FBE, urine for MCS, U&E, USD of abomen (fluid in the
cleanse vagina including recesses between rugae and adnexa or in sac or normal), complete STD screening
fornices, and also the folds around vulva. Avoid (urine PCR for Chlamydia and Herpes, Pap smear,
wearing pantyhose, tight jeans, or using tampons. high vaginal swab for wet film preparation for
Avoid having intercourse or oral sex during infected Trichomonas, endocervical swab for Chlamydia and
period. Do not use vaginal douches, powders or Gonorrhea, syphilis with VDRL and RPR, Hepatitis B
deodorants or take bubble baths. serology, HIV, throat swab or anorectal swab if
indicated, urethral swab if indicated)
Pelvic Inflammatory Disease
Diagnosis and Management
Case: You are an HMO in the Emergency Department and a 24- ‐ My most likely diagnosis is PID. It is the infection of
year-old female comes in due to pain in the RIF for 1 day. pelvic organs caused by bugs that are usually
acquired through sexual contact. The most common
Task ones are Chlamydia and Gonorrhea. These infections
a. History (getting worse, not related to change in are very common in young sexually active females.
position, feverish, vaginal discharge, smoking 10 The usual symptoms are high-grade fever, severe
cigarettes per day, LMP 2 weeks ago) tummy pain, and tenderness of the cervix.
b. Physical examination (in pain, pale, feverish (39), BP ‐ It is important to treat this infection carefully because
110/70, PR 104, tenderness at RIF, yellowish there are a number of complications both short- and
discharge on undergarments and vagina, cervical long-term.
excitation and adnexal tenderness on right side, no ‐ Short-term complications are abscess formation and
mass and uterine size normal) peritonitis
c. Investigation ‐ Long-term complications include a 10% chance of
d. Provisional and Differential diagnosis damage and obstruction of the fallopian tube after first
e. Management episode of PID and 30% after second episode, and
75% after 3rd episode, infertility, ectopic pregnancy,
Differential Diagnosis chronic pelvic pain and infection.
‐ Pelvic inflammatory disease ‐ That is why, we need to admit you and start you on IV
‐ Ectopic Pregnancy antibiotics most likely ceftriaxone IV 250 mg SD along
‐ Acute appendicitis with Azithromycin 1gm oral and later one switch to
‐ Ruptured ovarian cyst oral medications that you will need to continue for the
‐ Torsion of ovary next 2 weeks (doxycycline + metronidazole).
‐ Please avoid sexual activity until you are completely
History free of symptoms. Practice safe sex. If not already
‐ Is my patient hemodynamically stable? I would like to done, get yourself vaccinated with gardasil. We might
interview the patient in the resuscitation table. Where need to trace the contacts if required. Do not worry.
is the pain? How severe is the pain? Character? With IV antibiotics, the recurrence is quite low, but you
Associated symptoms like discharge, bleeding, need to be careful and practice safe sex in the future.
waterworks, N/V/ fever? When was the last time you
opened your bowel? History of constipation or Recurrent Ulcers (Herpes)
diarrhea? Is this the first episode of pain? Have you
had surgeries done previously? Case: your next patient in GP practice is a 30-year-old lady
‐ When was your LMP? Are your cycles regular? Do complaining of recurrent vulvar ulcers.
you get pain or spotting in between your periods? Any
complaints of excessive pain on day 1? Task
‐ Are you sexually active? Are you in a stable a. History (4x in that last year, went to dr gave
relationship? What contraception do you use? How medications and occur again, painful, discharge
many sexual partners have you had previously? Have yellowish no fever, 6/10, sexually active and affects
you or your partners ever been diagnosed with an sexual activity, pap smear normal)
STI? Have you ever been pregnant before? Any b. Physical examination (irritated and distressed, vitals
possibility you might be pregnant now? Have you had normal, no mouth ulcers, pelvic: ulcers few 3-4,
gardasil vaccinations? Pap smear? discharge +, not offensive, vulvar region, tenderness)
‐ How is your general health? SADMA? c. Diagnosis and management

Physical Examination History


‐ General appearance ‐ To help you today, I need to ask you some questions
‐ Vital signs and orthostatic hypotension and some of them might be sensitive, is it alright with
‐ Abdominal: distention, tenderness, guarding, rigidity, you?
Rovsing, mass, bowel sounds, hernial orifices ‐ How long have you been suffering from this? Do you
‐ Genital: discharge (color, quantity, smell), bleeding, currently have ulcers? Have you ever had them
signs of itching, trauma; sterile speculum looking for checked? Did you notice any precipitating symptoms
discharge, bleeding, condition of the cervix like like periods, stress, pregnancy? At the moment, are
redness; bimanual for size and position of uterus, you having any pain? Any vaginal discharge? Fever?
cervical excitation, adnexal mass and tenderness Body aches? Body pain? Urinary symptoms? Are you
‐ Urine dipstick, pregnancy test, and BSL sexually active? Do you have a steady partner? Have
you had unprotected sexual intercourse? Any history
of sexually transmitted disease? Have you ever been
screened for STI? Does your partner have similar
symptoms? Are you regular with your pap smear? Did
you get the gardasil vaccination?
60 
 
‐ Period: menarche, LMP, regular? - What are the risks? There are more risk of sexually
‐ Pills? Do you use condoms? transmitted infection, physical or sexual abuse,
‐ Have you ever been pregnant? pregnancy, drug and alcohol abuse
‐ PMHx or FHx? Social history? SADMA? - I will give you a medical clearance once we have the
results.
Physical examination - Offer gardasil vaccination
‐ General appearance
‐ Vital signs and BMI Retained Tampon
‐ Rashes or ulcers anywhere else in the body
‐ Pelvic: Case: 35/F presenting with offensive vaginal discharge.
o Inspection: ulcers, bleeding, painful,
discharge, Task:
o Swab a. History
‐ Abdomen/chest/heart b. Physical examination
‐ Urine dipstick and BSL c. Diagnosis
d. Management
Diagnosis and management
‐ Most likely your ulcers are caused by a virus called DIFFERENTIAL DIAGNOSIS
herpes simplex virus. This virus stays in one of the - Bacterial vaginosis
nerve roots of your body and under certain conditions - Trichomonas infection
such as menstruation, pregnancy, or low immunity, it - Foreign body (tampon/condom)
reoccurs and forms ulcers. - Cervicitis
‐ Usually the first attack is most severe. This is a - Cervical ectropion
sexually transmitted disease and I am afraid that you - Neoplasm
have acquired the virus from unprotected sex. At this - Atrophic vaginitis
stage, I would like to organize some investigations. I
would like to take a swab and send it for culture and History
sensitivity and I would also like to screen you for other - I understand from your notes that you have offensive
STIs. I will give you some strong pain killer and local vaginal discharge. I might need to ask some sensitive
gels (lignocaine) to apply. questions. Is that okay?
‐ Acyclovir within 72 hours of onset of rash. - When? Describe the discharge. Color? consistency?
‐ Rest. Warm salt baths. Do not scratch ulcers because Continuous/on-and-off? Related to coitus or menstrual
you can spread it in other parts of the body. Please cycle? Itchy or painful down below? Pain in your
wash your hands if you scratch them. Wear loose tummy? Fever?
clothings and clean cotton underwear all the time. - Has it happened before?
Avoid tight jeans. - Periods: regular? Amount? Duration? Painful?
‐ Sexual abstinence until the active lesions clear and Menarche? LMP? Do you use tampons or pads? Have
please practice safe sex (condoms + washing of you recently lost/missed a tampon
genitals before and after intercourse). - Partner: are you sexually active? Do you have a
‐ I would also like to recommend for your partner to stable partner? Does your partner have symptoms?
come and see me or his GP to organize STD Have you or your partner been diagnosed with STIs in
screening as well. the past?
‐ Reading material. Review. - Pills: what contraceptives do you use? Any history of
unprotected sex?
STI Screening - Pregnancy: how many? Ask for details if required
- Pap smear: when? Any abnormal results?
Case: Suzie aged 20 years presents to your surgery for the first - General history: water works and bowel movement?
time. She has recently started working at a local brothel and her - PMHx/FHx/SADMA
employer has told her she needs to have a 3-monthly health
checkup and get a certificate. Suzie lives independently in a Examination
shared accommodation and had no other medical or surgical - General appearance: pallor, BMI,
problems. - Vitals: BP, Temperature, RR, PR, O2 saturation
- Quick chest/heart
Task - Abdomen: masses/tenderness
a. Further history - Focused pelvic exam: ask for informed consent
b. Examination o Ask about discharge – color, amount,
c. Management advise consistency, smell, blood, redness, scratch
marks
Further history o Bimanual examination: uterine size, shape,
- Since when? What type of sexual practices do you tenderness, adnexal masses, mass in fornix
do? Are customers practicing safe sex? Any vaginal (can I remove the mass) cervical motion
discharge? Are you on any contraception (OCP)? tenderness (+ in ectopic pregnancy, PID,
Have you had any STIs? Did you have your pap endometriosis)
smear? Have you had gardasil vaccination?
Management:
Management - From history and PE, you have retained tampon which
- STI Screening: Chlamydia, gonorrhea, syphilis, HIV, got infected which I already removed. Since you don’t
HBV, HCV (if with history of IV drug abuse); hepatitis have signs of infection such as fever, no need for
A (MSM) antibiotics at this stage.
- Doctor is it legal? If the place is licensed, then it is - I would like to give you advice to prevent further
legal. recurrence
61 
 
o Personal hygiene ‐ Can it be STD doctor? It is unlikely, but it can also be
o Change tampons 3-4x a day caused by gonorrhea. Therefore, with your
o Use external pads at night permission, I would like to ask you to undergo STD
o Red flags: fever, muscle aches, pains, screening and swab the discharge.
dizziness, ‐ I would advise you to have Hot Sitz bath 4x a day,
- Advise to review after 3 days to see if there is any give you pain relief, and refer you to a gynecologist
infection ASAP to drain the pus. This procedure is called
- Give reading material marsupialization where a cut is done at the center, roll
- If with fever or signs of TSS: Call ambulance and it and stitch it outside that leads to a permanent
admit. Start IV fluids and IV antiobiotics (flucloxacillin x opening. I would also give you Azithromycin and
5-7 days) and should not use tampons in the future at Ceftriaxone.
all. ‐ I would advise you to wear loose clothing, maintain
- If with fever only: clean with povidone iodine 3x/day good personal hygiene, and practice safe sex.
for 2 days and oral flucloxacillin; send blood and urine ‐ Can it happen again doctor? Yes, there is a 10%
for culture; vaginal and cervical swab for culture chance of recurrence but prognosis is good and
recovery is excellent.
Bartholin Abscess ‐ Reading material. Referral. Red flags.

Case: You are a GP and a 35-year-old lady comes in with a Cystitis


lump in the vulva which she finds uncomfortable.
Case: You are a GP and a 24-year-old female Melissa comes in
Task complaining of pain on micturition with frequency, urgency, and
a. History (very painful lump, (+) yellowish to reddish lower abdominal discomfort for 2 days. She is married, a non-
discharge, can’t sit or walk comfortably, periods smoker, and has had appendectomy 10 years ago.
regular, sexually active, in stable relationship x 1 year,
on pills) Task
b. Physical examination (irritable, BMI 28, VS normal, a. History (terminal dysuria, no fever/N/V)
vulva: left labia majora, pea size, no redness but with b. Physical Examination (mild suprapubic tenderness,
discharge, no ulcer, tender, hot to touch; urine urine dipstick: ++++ RBC, ++ leukocytes)
dipstick, BSL, normal) c. Investigation
c. Diagnosis and management d. Management

History Differential Diagnosis


‐ When did you notice the lump? Does it come and go - Cystitis
or is it there all the time? Is it increasing in size? Any - PID
discharge? What is the color? Is it painful? How is it - STDs
affecting your life? Have you noticed any lump in any
part of your body? Any rash or vesicle in your private History
part? Is it the first time? Do you feel feverish? Any - Pain questions: SORTSARA?
problem with your waterworks? Bowel habits? - 5Ps Æ especially Periods (LMP) and sexual history
‐ 5Ps - FHx of kidney problems
‐ Any past medical or surgical history? Are you on any - Allergies? Previous history of UTIs
medications? SADMA?
Physical Examination
Physical examination - General appearance
‐ General appearance and BMI - Vital signs
‐ Vital signs - Abdomen: tenderness, masses, CVA tenderness
‐ Chest and heart - Urine dipstick, urine PT
‐ Abdomen
‐ Pelvic: Management
o Inspection: site, size, shape, discharge and - Lifestyle modification
if smelly, color o Drink ample fluids
o Palpation: temperature, tenderness, o Voiding post-intercourse
consistency, fluctuant o Cranberry juice
o Speculum examination o Hygiene: wipe from front to back
o Bimanual examination - Medications: trimethoprim/amoxicillin/cefalexin
‐ Lymph nodes - Review after 3 days to check for sensitivity to drug
‐ Urine dipstick and BSL
Urinary Tract Infection/Prescription Writing
Diagnosis and Management
‐ You have a condition called bartholin abscess. Case: You are a GP and your next patient is a 23-year-old
There’s a gland called bartholin gland located on each female who complains of lower abdominal pain.
side of your vaginal opening and there is a collection
of pus in the small duct. Because it is blocked, the Task:
secretions and pus are trapped causing a painful a. History: dull constant, not related to menses; LMP: 6
swelling. It usually appears within 2-3 days and weeks
causes severe pain on walking and sitting. It is usually b. Physical examination: suprapubic tenderness;
caused by a bug E. coli, Streptococcus, c. Diagnosis
Staphylococcus. d. Management and write script
62 
 
History UTEROVAGINAL PROLAPSE AND INCONTINENCE
‐ Can you tell me where exactly is the pain? How bad is Urinary (Stress) Incontinence
it on a scale of 1-10 (4)? Does it go anywhere? Can
you describe the character? Is it a dull ache or sharp Case: A 50-year-old woman who had 3 kids aged 29, 25 and 22
pain? Any aggravating or relieving factors like came to your GP clinic complaining of leakage of urine.
movement? Any associated nausea, vomiting, fever or Task
vaginal discharge? Bleeding or spotting? Any history - History
of constipation or change in bowel habits? How are - Appropriate investigations
your waterworks? Any burning or frequent passing of - Diagnosis and management
urine? Have you noticed any blood in the urine? Is this
the first time to have it? When did you have it? What Causes of incontinence (DIAPPEERSS)
were the symptoms? What test? What treatment was D – elirium
given? Any problems? When was the last episode of I – infection of UT
UTI? A -trophic urethritis
‐ Partner/Pills: Are you sexually active? Are you in a P – harmacological (diuretics)
stable relationship? How many partners have you had P – sychological (acute distress)
previously? Do you always practice safe sex by the E – ndocrine (hypercalcemia)
use of condoms? Any history of STIs? What method E – nvironmental (unfamiliar surrounding)
of contraception do you use? R – estricted mobility
‐ Period: When was your LMP? Are they regular? How S – tool impaction
is the cycle like? Any chance that you might be S – hincter damage or weakness
pregnant?
‐ SADMA? PMHx
Drugs Causing Incontinence
Examination - Antihypertensives/vasodilators - (ACEI, prazosin,
‐ General appearance: pallor, jaundice, dehydration labetalol, phenoxybenzamine)
‐ Vital signs - Bladder relaxants – (anticholinergics, TCAs)
‐ Chest/heart - Bladder stimulants – (cholinergic, caffeine)
‐ Abdomen: distention? Tenderness on palpation - Sedatives – (antidepressants, antihistamines,
especially in the RIF/LIF. Any mass palpable? Organ antipsychotics, hypnotics, tranquilizers)
enlargement? Bowel sounds? Hernia? - Others: alcohol, loop diuretics, lithium
‐ Pelvic exam: Inspection (bleeding, discharge, scratch
marks, ulcers); speculum (bleeding, discharge, Incontinence:
position of cervix); bimanual (size and shape of - Stress: small amounts of urine; involuntary during
uterus, cervical excitation, adnexal mass/tenderness) coughing, straining, laughing, etc; due to weakening of
‐ Urine dipstick (leukocytes and nitrates), pregnancy muscles in the pelvis Æ increasing intra-abdominal
test and BSL pressure results in leakage of urine
- Urge: large amounts of urine; want to go to bathroom
Management but cannot control; problem with detrussor or nerves
‐ As previously you had another attack of UTI, do you
know what it is? At the moment, I am sending a Investigations
sample of your urine for culture and microscopy. You - MSU!!!
need to drink ample fluids especially cranberry juice. - Urodynamic studies (measure pressure in the bladder
After passing water and stools, please wipe from front and urethra)
to back. I will write some antibiotics for you. o Urge incontinence: pressure in bladder
Trimethoprim or Cephalexin (500mg 2x a day for 5 increases very fast reducing bladder
days or amoxicillin + clavulanic acid (500mg/125mg capacity;
2x a day for 5 days (especially if pregnant) o Stress incontinence: intravesical pressure
does not increase when urine fills; bladder
capacity is normal
Name of patient Doctor’s name
Risk factors:
DOB/age Address - UTI
- Obesity
Address Telephone number - Smoking
- Caffeine
Prescriber no. - Constipation
- Chronic cough
PBS/Private - Multiparity
- Menopause

History
What do you mean by losing urine? Is it small or
large? Do you lose urine when you laugh, cough,
Tab Trimethoprim exercising or just normal? Do you lose a lot of urine
when you try to reach the toilet? Any feeling of
300 mg x daily for 3 days masses down below?
- Any burning in urination? Frequency? Frothy urine?
Change in color of urine? Polyuria? Polydypsia or
polyphagia?
63 
 
- Was it difficult labor? Assisted labor? Assisted you have associated tummy pain or heavy/dragging
delivery? Symptoms of menopause? Hot flushes? kind of sensation in the lower tummy? Any urinary
Dyspareunia? Mood swings? Pap smear? Partner? complaints like frequency, burning or leaking of urine?
Stable relationship? Any previous history of STDs? Any loin pain? Any history of prolonged cough,
Have you started with mammography? constipation, asthma or respiratory problems? Do you
- Any medical history? Chronic cough or constipation? have any problems emptying the bowels? Any
Joint problems? complaints of discharge or bleeding from down below?
- SADMA? BMI? Any fever? Itching? When was your LMP? Any
problems during or after menopause?
Examination ‐ When did you have the hysterectomy? Why did you
- General appearance: BMI, dehydration, pallor, have it? Any complications afterwards? Was it done at
jaundice a tertiary care center? After the surgery, did you do
- Vitals pelvic floor exercises? Any other surgeries that I
- Chest/heart: chronic infections should be aware of? Did you take any HRT
- Abdomen afterwards?
- Pelvic examination: rule out prolapse (cystocele) ‐ May I know are you sexually active at the moment?
- Ask examiner for any demonstrable stress Any complaints of pain or discomfort during sex? How
incontinence (ask patient to cough and check for many kids have you had? Any history of big babies?
leakage of urine) Difficult or instrumental deliveries?
- BSL and Dipstick ‐ SADMA?
‐ Have you recently noticed weight loss? Change in
Diagnosis and Management appetite? Night sweats? Lumps and bumps in the
- You most likely have a condition called stress body? Pap smear? Mammogram?
incontinence. When the urethra is no longer in the
pelvis, there is an increase in intra-abdominal Physical examination
pressure, which affects both bladder and urethra ‐ General appearance
increasing the bladder pressure more than the urethral ‐ Vital signs
pressure, resulting to involuntary loss of urine. ‐ Chest and Lungs
- I completely understand that it is a very frustrating ‐ Abdomen: for tenderness
condition for you, but let me assure you that we can ‐ Pelvic exam
manage it. o inspection: Obvious lump, discharge, ulcer,
- Stress incontinence is highly associated with UTI so I redness, discharge
would like to order urine microscopy and culture (ask o Sterile speculum examination asking the
examiner for results). patient to strain looking for any visible lump
- I would advise you to maintain a bladder diary. Avoid while straining; sims left lateral position
too much physical stress, lifestyle modification (weight (knee-chest position) Æ gradually withdraw
reduction, smoking cessation, decrease caffeine while asking a patient to strain Æ
intake), avoid constipation and coughing lump/bulge in the vagina (best way to detect
- Start pelvic floor exercises (contract pelvic muscles as cystocele and rectocele)
if your lifting your pelvis or holding urine 40-50x daily ‐ Urine dipstick and BSL
at 3 months)
- Refer to gynecologist regarding vaginal pessaries. Diagnosis and Management
They may consider giving you HRT and urodynamic ‐ Most likely what you have is prolapse of the vaginal
studies but will be decided upon by the specialist. wall after hysterectomy. Once the uterus is removed,
- Surgery will only be indicated if conservative the upper part of the vagina loses its anatomical
measures fail. Bladder neck suspension, suburethral support. Usually, during hysterectomy, the surgeon
rings, and local injection of collagen. will secure the upper part of vagina with the help of
- For urge incontinence: bladder training and anti- ligaments attached to the backbone and pelvic wall.
cholinergic medications (oxybutynin, propantheline, Some of these ligaments become loose because of: a.
imipramine, tolterodine)Æ refer to physiotherapist loss of estrogen b. prolonged straining/coughing c.
- Review and Reading Material putting on weight.
- Stress incontinence: MSU for urine and culture ‐ This phenomenon is quite common after
- Postmenopausal bleeding: Transvaginal hysterectomy. Up to 30% of patients might develop
Ultrasound this. It can affect the urinary system leading to
frequent recurrent UTIs. It can also affect the wall of
Post-hysterectomy Prolapse the bowel causing constipation. Sexual functioning
may be affected and might cause pain and discomfort
Case: You are a GP and a 52-year-old female comes to your during intercourse.
clinic complaining of something coming out from her vagina ‐ The treatment will be tailored according to your
especially after straining wishes, but you will need to see a specialist
gynecologist. The first option is conservative
Task management which includes pessaries along with
a. History pelvic floor exercises. Usually, this suitable for old,
b. Physical examination females who are not fit for surgeries. The second
c. Management option is the surgical approach. It is called vaginal wall
suspension surgery (sacrocolpopexy). The surgeon
History will attach the upper part of the vagina to the strong
‐ Please tell me more about your problem? Since when tissues within the pelvis usually to the lower backbone
have you noticed this lump? Is it present all the time or or sacrum. There are 2 options regarding the
does it come and go? Any changes with change in approach: laparoscopic or keyhole surgery OR
position like prolonged standing or lying down? Do abdominal approach best decided by the surgeon.
64 
 
‐ The recurrence rate after the surgery is very low - PR: differentiate between cystocele and rectocele
therefore the surgery is mostly curative. - BSL and Urine dipstick
‐ Review. Reading material.
‐ Pelvic floor exercise (kegel): done to strengthen the Diagnosis and Management
muscles of the pelvic floor. The exercise can be done - You have a condition called uterovaginal prolapse with
either sitting or lying down. The patient needs to stress incontinence and candidiasis,
empty the bladder before exercise. Contract the pelvic - Menopause resulting to lack of estrogen, difficult
muscles, hold contraction for at least 5 seconds, labor, big babies and constipation leads to the laxity of
release it slowly and repeat 3-4x and gradually build the pelvic floor ligaments. It is a common condition
up duration for up to 10 seconds. She must not among females in your age group.
contract the abdominal, thigh or buttock muscles. - At this stage, I would like to refer you to the
Exercises must be repeated 3x a day as many times gynecologist. I would advise you to start with pelvic
as possible. Results are usually apparent within 8-10 floor exercises (contract pelvic floor muscles as if
weeks. Safe to be done during pregnancy trying to hold urine).
- The specialist might insert a pessary which is a device
Uterine Prolapse inserted into the vagina to support the uterus. They
need to be changed every 3-6 months. They also
Case: An a 80-year-old lady comes to your GP clinic advise topical estrogen to improve the discomfort.
complaining of mass protruding down below and rash around - Will it affect intercourse? Pessaries will not interfere
the private area for several months. with your sexual performance.
- If conservative measures do not work, the specialist
Task might consider doing surgery to fix the ligaments.
a. Relevant history - How long will I be in the hospital? Usually 3-5 days.
b. Physical examination (BMI 29, maculopapular rash You can go home once you’re feeling well and once
around introitus and inside of thigh, urine dipstick + you have started urinating without problems.
sugar, BSL 11.3mmol/L - Postop advice: For the first two weeks, restrict your
c. Diagnosis and management activities. Rest. Avoid heavy lifting. Avoid sports and
swimming. For 1st 6 weeks abstain from sexual
History intercourse.
- I read from your notes that you have something - Driving: It is not advisable to drive for the first 2
bulging from your private area. Since when? Can you weeks.
tell how it happened? Is it increasing? Do you feel any - Complications: Pain, bleeding, injury to nearby
abdominal discomfort? What is the effect of this structures, anesthesia complications
bulging on your life? Is this swelling affecting your - For the candida, I will prescribe you antifungals. It
waterworks? Do you leak urine while you strain, might be related to high blood sugar. I will give you
cough, etc? Do you have a strong urge to void on the referral to physician to investigate further
way to the toilet or do you leak a large amount of urine - Lifestyle modification: normal BMI, stop smoking,
on the way to the toilet? Any discharge down below? high-fiber diet
Constipation? Waterworks? - Referral to specialist obstetrician. Reading material.
- Rash? Since when? Is it itchy? Can you describe the Review.
rash for me? - Advise OGTT.
- Period: When was your last period? Any irregular
bleeding after that? Hot flushes? Mood swings? Prolapse:
Breast pain? Irritable? - I: cervix remains within vagina
- Pregnancy: how many pregnancies? Were they big - II: cervix comes up to introitus
babies? Did you have any difficult labor or prolonged - III: most of uterus lie outside vagina
labor?
- Partner: are you sexually active? Do you have a Uterine Prolapse
stable partner? Do you have painful intercourse?
Have you or your partner ever been diagnosed with Case: A 58-year-old lady comes in your GP clinic complaining of
STDs? lump from the vagina.
- Pap smear: When was your last pap smear? Result?
- Mammography? Task
- Past medical history: chronic cough, diabetes, asthma a. History
- FHx: Osteoporosis, MI b. Physical examination
- SADMA c. Explain management

Physical Examination History


- General appearance - I understand you have come because you are worried
- Vital signs about a lump that is coming out from your vagina.
- Abdomen When did you first notice it? Does it come and go or is
- Pelvic examination: it present all the time? Did you notice that it appears
o Inspection for morphology of the rash when you’re straining? Do you have a dragging
(maculopapular rash around the introitus sensation or heaviness in the tummy? Any unusual
and groin area), scratch marks, discharge, vaginal discharge or bleeding?
obvious bulge - Do you have chronic cough? How’s your waterworks?
o Speculum: wall of vagina, rash, discharge, Have you noticed increased frequency or feeling that
blood, ask patient to cough (cervix comes your bladder is emptying incompletely? Do you have
up to the introitus), leakage of urine, cervix urine leakage during coughing, straining or laughing?
o PV: adnexal masses, CMT, Do you have regular bowel movements?
65 
 
- When was your LMP (menopause risk factor)? Any BENIGN TUMORS
spotting or bleeding after that? Are you sexually
active? Does this problem cause difficulty or pain with Ovarian Torsion/Ruptured Ovarian Cyst
intercourse? When was your last pap smear? Was it
normal? How many pregnancies have you had? How Case: Julia aged 35 years presents to ED of local hospital
many children do you have? Do you remember the where you are working as year 1 RMO. She had severe right
birth weight (>4kg)? Did you have NSVD? Did you sided abdominal pain for the last 1-2 hours associated with
have instrumental delivery? nausea and vomiting. She had similar pain a few months ago
- Are you generally healthy? Surgeries? Medications? but lasted only for a few minutes and was relived with panadol
Smoking? What are you doing for a living? and neurofen. She had no other significant medical or surgical
problems. She had known allergies and is not on any regular
Physical examination medications. Julia works as a business consultant in a local firm
- General appearance and lives with her partner. She smokes about 10 cigarettes per
- Vital signs and BMI day and is a social drinker.
- Abdomen: masses and tenderness
- Pelvic: Task
o Inspection: evidence of prolapse and a. Further history (10/10 in severity, tried panadol and
atrophic changes; can you please strain or neurofen but did not work; RLQ, no fever, no rash, no
cough (for 2nd degree prolapse)? problems with bowel motions or waterworks; periods
o Speculum (left lateral position): using sims are regular, LMP 3 weeks ago)
speculum prolapse, check for cystocele or b. Physical examination (uncomfortable but fully
rectocele, degree of prolapse, atrophic conscious and oriented, PR 84, BP: 100/70, T and RR
changes, discharge, appearance of cervix, normal; no LAD, no lumps and bumps, chest and
o Bimanual examination: any pelvic masses heart normal; inspection normal; no distention; palpate
palpable, size of uterus, and adnexa; ask tender at RIF but no rebound or guarding/rigidity, no
patient to squeeze to fingers to assess of palpable mass; no organomegaly; pelvic examination:
pelvic muscle strength normal; PR normal; urine PT negative, urine dipstick;
- Urine dipstick and BSL FBE normal U&E normal; USD pending)
c. Diagnosis and management
Degree of prolapse
- I – cervix protrudes/sits into lower 1/3 of vagina Features
- II – cervix protrudes on straining outside of vagina ‐ Ovarian Cysts:
- III – cervix/uterus lies outside of the vagina o Common in women under 50 years of age
o Best defined by TVS
Diagnosis and Management o Symptoms: pain, pressure symptoms,
- You have a condition called uterine prolapse. Have menstrual irregularities
you ever heard about it? The uterus, bladder and ‐ Ruptured ovarian cyst:
bowel are supported by a tight hammock of muscles o 15-25 years
slung between the tail and pubic bone. These muscles o Symptoms: Sudden onset of pain in one or
are known as pelvic floor muscles. Ligaments also other iliac fossa; No systemic signs; Pain
anchor uterus in place. If these tissues are weakened usually settles within a few hours
or damaged, the uterus can slip down into the vagina. o Signs: tenderness and guarding in iliac
We call it uterine prolapse. fossa, PR: tenderness in rectovaginal pouch
- Common causes of uterine prolapse include vaginal o Investigation: USD + color Doppler
childbirth especially if baby was large or delivered o Management
quickly or if there was a prolonged pushing phase or ƒ Explanation and reassurance
instrumental delivery. Another group of risk factors is ƒ Conservative: simple cyst <4cm,
being overweight, having chronic cough, constipation, internal hemorrhage, minimal pain
and heavy lifting which are factors that increase intra- ƒ Needle vaginal drainage by USD
abdominal pressure. The last predisposing factor is a for simple larger cyst
low level of estrogen after menopause. ƒ Laparoscopy: complex cysts, large
- I will refer you to a gynecologist for further cysts, or external bleeding
assessment and to discuss treatment options. ‐ Ovarian torsion
- Treatment depends on age, degree of prolapse, and o Mainly from dermoid cysts
patient preference. o Symptoms: severe cramping lower
- Meanwhile, I will arrange a meeting with a abdominal pain, diffuse, pain may radiate to
physiotherapist who will teach you pelvic floor
exercises (effective for 1st and 2nd degree). I also
recommend for you to have lifestyle modification. Try
to keep your weight within the ideal range, have a flank, back or thigh; repeated vomiting,
balanced diet, regular exercise, and smoking exquisite pelvic tenderness, patient looks ill
cessation. o Signs: smooth, rounded mobile mass
- The most effective treatment is surgery which is palpable in abdomen; may be tenderness
vaginal hysterectomy. Sometimes before surgery or if and guarding over the mass
woman is not fit for surgery, or if woman does not o Investigation: USD + color Doppler
want surgery, a vaginal pessary can be used which is o Management: Laparotomy
a donut-shaped device inserted into the vagina and
positioned to prop the cervix and uterus. It should be Differential Diagnosis
changed every 6 months. Side effects include irritating ‐ Ectopic Pregnancy
discharge and increased risk of ulceration as well. ‐ Ruptured ovarian cyst/torsion
Local estrogen can be used to decrease the side ‐ PID
effect.
66 
 
‐ Acute appendicitis Physical Examination
‐ Acute mesenteric ischemia ‐ General appearance
‐ Renal colic ‐ Vital signs
‐ Abdomen: Visible masses, organomegaly, tenderness
History ‐ Pelvic Exam: discharge, bleeding, cervical os
‐ Since when? SORTSARA? Associated features like close/open, enlarged irregular uterus about the size of
fever? Nausea? Vomiting? Any discharge from below? 12 weeks’ GA. No palpable adnexal masses. CMT
Any problems with waterworks or bowel movements? negative.
‐ 5Ps: ‐ Urine dipstick, BSL, urine PT
o Period: LMP? How many days of bleeding?
How many days apart? Any bleeding in Diagnosis and Management
between? Any chance you might be ‐ From the history and examination, the most likely
pregnant? cause of your heavy periods is uterine fibroid or
o Pills myoma. However, I need to do some investigations to
o Partner: stable relationship? Ever been confirm the diagnosis. The investigations are beta-
diagnosed with STIs? Partner? hcg, iron studies, coagulation profile, urine MCS, and
o Pap smear TVS. On TVS, there is a large 10 cm hypoechogenic
o Pregnancy region in the fundus of the uterus.
‐ PMHx: Surgery or any medical illnesses? ‐ A fibroid is a benign tumor which is formed inside the
‐ FHx: uterus. It is very common in the reproductive age
group. Let me reassure you that it is not a cancer. The
Physical Examination exact cause is unknown, but it is suspected that the
‐ General appearance sex hormones, estrogen and progesterone, play a
‐ Vital signs significant role. That is the reason why fibroids rarely
‐ Abdomen and inguinal orifice grow in pre-pubescent girls and postmenopausal
‐ PR women. Pre-existing fibroids stop growing and even
‐ Pelvic examination shrink once a woman passes menopause. Fibroids
‐ Urine dipstick, urine PT, BSL often cause no problems but occasionally, it can be
associated with:
Diagnosis and management o Anemia - because of excessive menstrual
‐ Refer to OB registrar blood loss and cause fatigue, pallor and
‐ Start IV line and take bloods breathlessness
o Urinary problems - because large fibroids
Uterine Fibroid can bulge the uterus against the bladder
causing a sensation of fullness or discomfort
Case: A 35-year-old female comes to your GP clinic and the need to urinate often
complaining of heavy menstrual flow for several months. she o Infertility - presence of fibroids can interfere
has 2 children 6 and 8 years. She still wants to have a baby in with the implantation of fertilized egg in a
the future. Her FBE showed low hemoglobin. number of ways making successful
implantation difficult
Task o Miscarriage and premature delivery – can
a. History (x6 months, change pads every 3 hours, 4-5 reduce blood flow to placenta or may
pads/day, 9-10 days, feel pressure and fullness in compete for space with the developing baby
lower tummy and sometimes has difficulty passing ‐ I will need to refer you to the gynecologist for further
urine, urinary frequency) assessment and management. Treatment depends on
b. Physical examination (Specific findings will be given the location, size, and number of fibroids. If fibroids
only when asked) – uterus 12 weeks size and irregular are small and not causing symptoms, we may just
c. Investigation observe. For the large fibroids, we can use drugs to
d. Management shrink prior to surgery and these drugs are danazol
(GnRh agonist) or medroxyprogesterone. The surgical
History options are uterine artery embolization. It is done
‐ Are you bleeding now? Since when did you have the under local anesthesia. A fine tube is passed via an
heavy periods? How many times do you have to artery in the arm of leg into the main artery supplying
change a pad in a day? Are the pads fully soaked? Do the fibroids. Fine particles like sand are then injected
you pass any clots as well? What is the duration of into the artery to block its blood supply. The fibroids
your periods? Do you feel dizzy, palpitations, fainting? slowly die and symptoms should settle over a few
Any pain during your periods? Do you have bleeding months. The whole process is monitored by xray.
between periods? Do you have any bleeding Another option is hysteroscopic myomectomy. The
disorders? LMP? Any chance you could be pregnant? gynecologist can pass a tube via the cervix and
‐ Are you sexually active? Are you in a stable remove the fibroid. They can also choose to do
laparoscopic myomectomy which is a key-hole
surgery through abdomen. Very less likely, they will go
relationship? Are you on any contraceptive? Have you for open surgeries especially if the fibroids are very
or your partner ever been diagnosed with STDs? big. The last option is hysterectomy. This is done
‐ Previous pregnancies? Pap smear? especially if the woman has completed her family. The
‐ Any problem with your waterworks? Do you have any one disadvantage of having open surgery is that
burning while urinating? cesarean section is more likely done in succeeding
‐ FHx of bleeding disorders or gynecological pregnancies because of the weakening of the
tumors/cancers? abdominal and myometrial wall.
‐ Iron therapy.
‐ Reading material. Referral.
67 
 
Acute Urinary Retention secondary to Fibroids Diagnosis and Management
‐ You have an acute retention of urine and on PE, I
Case: A 45-year-old woman comes to you in your GP clinic found a pelvic mass which can arise from the uterus
complaining of difficulty to pass urine. or the ovaries. You need to have a pelvic USD to
establish the diagnosis. I need to refer you the
Task hospital where you will be assessed by the
a. History (3children, NSVD, pap smear >2 years gynecologist.
normal, USD GB normal, + discomfort during sexual ‐ The gynecologist will arrange further investigations
intercourse, regular bowel movement) including FBE, U&E, CA-125, and TVS/TAS.
b. Physical examination (BMI 25, vital signs normal, soft, ‐ If this problem is due to fibroid which is a benign tumor
no distention or masses, no discharge or bleeding, of the uterus, the treatment will depend on site, size
bimanual normal, pelvic mass which is hard to and desire for pregnancy. You have an acute
distinguish if it arises from uterus or adnexa, urine presentation and most likely it will require surgery,
dipstick and BSL) laparoscopic or open.
c. Management ‐ If it is benign ovarian cyst or tumor, cystectomy can be
performed. However, in women above 40, bilateral
Causes salpingo-oophorectomy plus total hysterectomy is
‐ Pelvic Mass (Fibroids or ovaries) preferred. A gynecologist will discuss diagnosis and
‐ Pregnancy all available options.
‐ Herpes simplex ‐ A catheter should stay in the bladder until a cause for
‐ Prolapse your presentation has been identified and treated.
‐ Neurologic problems
‐ Renal stones BREAST
‐ Constipation (elderly)
‐ Medications (antidepressants/antipsychotics) Cyclical Mastalgia
‐ Males: Prostate enlargement
Case: A 40-year-old woman comes to see you in your GP
History practice. She complains of cyclic pain in both breasts. On
‐ I understand you came to see me because you can’t examination, there are some lumps in her breasts on the upper
pass urine? For how long? Are you comfortable outer quadrant. She was not able to tolerate OCPs because of
enough for me to ask you a few questions or you want vomiting and her mother was diagnosed with breast cancer
me to address this problem first? when she was 60 and was treated with radical mastectomy.
‐ Is it the first time? Can you recall any precipitating
factors like trauma to the back or pelvis? Have you Task
noticed change in urination or frequency before? Do a. History
you have any bowel problems? Do you have difficulty b. Diagnosis
or discomfort when you try to urinate? Did you have c. Management
leakage of urine while laughing, coughing or
sneezing? Have you noticed any rash in your private Risk factors:
area? Have you noticed any unusual vaginal ‐ Caffeine intake
discharge? Have you noticed any lump coming out of ‐ Inappropriate brassieres
your vagina? ‐ Obesity
‐ When was your LMP? Is it regular? Any excessive
pain or bleeding? Have they always been heavy or is History
it something new? Are you in a stable relationship? ‐ Let me acknowledge your pain and your concern
Have you ever been diagnosed with STD? Do you about the breast lumps. I know you’re worried about
have pain or discomfort during sexual intercourse? your mom’s condition, but before we go ahead I would
What type of contraception do you use? How many like to ask you some questions.
pregnancies have you had? How many children do ‐ Some of the questions might be sensitive, is that
you have? Type of delivery? BW? When was your last okay?
pap smear? Was it normal? ‐ When did you start having pain in your breast?
‐ Are you generally healthy? Ever been diagnosed with Severity (1-10), site (both breasts/single)? radiation?
renal stone? Any medications? Allergies? Aggravating factor (periods)? Associated factors?
How is it affecting your life? Any previous history of
Physical examination similar problems? Any previous breast problems in
‐ General appearance general? Nipple discharge? Changes in breast?
‐ Vital signs and BMI Swelling and erythema? Any lumps and bumps in the
‐ Abdomen: palpate distended bladder (smooth, firm, body? Back/bone pains? Any cough or other chest
oval dull suprapubic mass) symptoms? Headache, N/V, or visual changes?
‐ Pelvic exam ‐ Do you drink too much coffee? Do you have bra
o Inspection: any evidence of prolapse problems?
o Speculum in left lateral position with sims ‐ 5Ps: pregnancy: any chance you could be pregnant at
speculum the moment? Partner, pills, periods (regular?
Bleeding? Clots duration of cycle? Menarche); pap
smear: any abnormal pap so far?
o Bimanual examination: size of uterus, ‐ FHx: other cancers? PMHx
contour, consistency, adnexal mass
‐ Urinary catheter and take urinary sample for
microscopy and culture. After emptying bladder, can I
palpate any abdominal masses?
‐ Urine dipstick and PT
68 
 
Management ‐ Paget disease of nipple
‐ With respect to your worries about the cancer, let me ‐ Breast eczema
reassure you that the pain and lump sensation is due
to a benign condition called cyclical mastalgia. Most Features
likely, it is because of hormonal changes during ‐ Benign hyperplastic lesions within large mammary
menstruation. It usually starts a couple of days before ducts and not premalignant
menstruation and relieved during the commencement ‐ Present with nipple bleeding or blood-stained
of menstruation, but let me reassure you that it is not discharge and must be differentiated from infiltrating
cancer. It is very common in women aged 30-40 years carcinoma
‐ Advise weight reduction ‐ Involved duct and affected breast segment should be
‐ Reduce caffeine intake (not >1-2 cups/day) and low excised Æ ductectomy
fat
‐ Stop smoking Triple Test for Lumps
‐ Wear good quality comfortable brassiere ‐ History and physical examination
‐ Prescribe analgesics ‐ Imaging (USD or mammography)
‐ If not responsive, then add mefenamic acid, vitamin ‐ Biopsy (Fine needle or core biopsy)
b1 and b6. Æ evening primrose oil Æ danazol
‐ Because of your concern about your mom’s condition, MENOPAUSE
which increases your risk of having a breast cancer
(1:14 to 1:10), I will refer you to a specialist who will Menopause Investigations
order further investigations like mammography (every
2 years from now) and annual examination by GP and Case: Your next patient is a 54-year-old female who had her last
monthly self-breast examination. period 18 months ago. Now, she has mood swings, sweating,
‐ Exercise (aerobic upper exercises) and dyspareunia. She also has FHx of osteoporosis. She has 3
‐ See his sister children and her mother has osteoporosis.
‐ Cause: Estrogen
Task
Nipple discharge (Intraductal Papilloma) a. History
b. Physical Examination: dry vagina
Case: Marion aged 51 years presents to your GP clinic in a busy c. Investigation
afternoon and tells you that she is quite worried about her nipple d. Management
discharge. The discharge is from right nipple describing it as
pinkish. The discharge is spontaneous and she had also noticed Features
discoloration on her nightie. It happened last night and also last ‐ Cessation of menses for >12 mos.
week. It is of small amount, leaving a stain about the size of 20 ‐ Pre-menopausal – 5 years before the onset of last
cent piece on her clothing. She never had any breast problems menstrual period
before and is very concerned. Marion is a mother of 3 who she ‐ Perimenopause – the time when menses become
bottlefed. She had paternal grandmother who had mastectomy irregular (2 years before)
although she doesn’t know any more details. She had attended ‐ Postmenopause – women who have not experienced
a breast screen clinic about six months ago and was all OK. She menstrual bleeding from a minimum of 12 months and
is still menstruating but her cycles have become quite irregular up to 5 years after menopause
and scanty over the last year. ‐ FSH/LH increase and estradiol decrease
‐ Symptoms:
Task o Bleeding: oligmenorrhea/menorrhagia
a. Further history o Hot flushes: heat centered on the face and
b. Physical examination spreads to neck and chest; accompanied by
c. Differential diagnosis and management advise vasodilation and sweating; episodes last 2-4
minutes happening several times a day;
DISCHARGE DIAGNOSIS should be fine after 70 years
Blood Intraductal papilloma o Sleep disturbance
Cancer o Vaginal dryness (estrogen deficiency which
Green Duct ectasia can lead to vaginal atrophy and
Mammary dysplasia dyspareunia; pale vagina; pH which is
Yellow Mammary duct ectasia usually <4.5 in reproductive years increases
Abscess to 6-7 and hence more prone to infections)
Carcinoma o Sexual dysfunction: low libido and
White Lactation cyst decreased vaginal lubrication; elasticity of
Hyperprolactinemia wall decreases and vagina may become
Drugs: chlorpromazine shorter; continuing sexual activity may
Straw-color Fibroadenoma prevent changes
Serous Carcinoma o Incontinence
o Breast pain and tenderness in early
Abnormal: menopause
‐ Color of discharge (serous, blood) o Skin changes
‐ Spontaneous discharge o Osteoporosis
‐ Discharge coming from nipple o Cardiovascular problems
o Dementia
Differential Diagnosis o Anxiety, tearfulness, blues, loss of
‐ Ductal papilloma concentration are NOT menopausal
‐ Infiltrating ductal carcinoma symptoms
‐ Medications (metoclopramide, SSRIs, OCP, cocaine)
69 
 
‐ Management Atrophic Vaginitis
o Education
o Investigations Case: You are a GP and a 60-year-old female comes in
o Healthy lifestyle (diet, exercise, pelvic floor complaining of vaginal discharge.
exercise)
o Consider HRT Task
ƒ Relieve flushes and vaginal a. History (vaginal discharge x 5 days, brownish,
symptoms menopause 5-6 years ago, no HRT)
ƒ Induce feeling of wellbeing b. Physical examination (thin and dry; pale, discharge)
ƒ Prevent osteoporosis c. Diagnosis and management
ƒ Improve skin
ƒ Efficacy regarding History
cardioprotection is controversial - When did you notice it? What’s the color? How much?
‐ Itchy, bitchy, sweaty, sleepy, bloated, forgetful and Is it smelly? Is it itchy? Does your partner have similar
psycho complaints?
- Periods: menarche, LMP, menopause? Are you
History sexually active? Are you in a stable relationship? Do
‐ Ask menopausal symptoms: problems with bleeding? you have problems with sex? Have you or your
Hot flushes? Sleep disturbance? Dyspareunia? partner ever been diagnosed with STIs?
Sexual dysfunction? Incontinence? Breast pain and - Pregnancies? Pap smear? When was the last one?
tenderness? Skin changes? Osteoporosis (bone pain, Mammography?
backaches)? CV problems? Signs of dementia? - 4B and 2Ps in a postmenopausal woman:
Differentiate mood swing from depression? Any o Bladder, bowel, breast, bone
change in weight or appetite? o Prolapse and Postmenopausal symptoms
‐ 5Ps: pills, pregnancy, partner (history of STD), Pap (mood swings, hot flushes, irritability,
smear, mammography? Periods (postmenopausal dyspareunia, bleeding)
bleeding)? - FHx: cancers
‐ How is it affecting your life? - How is your general health? SADMA
‐ Contraindications for HRT: ever been diagnosed with
stroke, TIA, migraine, hypertension, thyroid disease, Physical Examination
clots in legs or lungs, undiagnosed vaginal bleeding, - General examination
liver disease, personal or FHx of breast or endometrial - Vital signs and BMI
cancer? - Breast examination for lumps
‐ SADMA? - Pelvic examination
o Inspection: discharge, color, amount, smell,
Investigations scratch marks, visible prolapse,
‐ FBE with iron studies o Speculum: discharge, vaginal wall for pallor,
‐ Urinalysis dryness, thin, atrophic, rectocele or
‐ U&E, Ca and Vitamin D levels cystocele or prolapse, pap smear
‐ LFts, TFTs, Lipid profile o PV: cervical motion tenderness, adnexal
‐ Pap smear masses
‐ Mammography (all women before or after 3 months on - Urine dipstick and BSL
HRT)
‐ Hormone levels: FSH, LH, estradiol, progesterone, Diagnosis and Management
testosterone, PRL, - You have a condition called atrophic vaginitis. It is a
‐ Coagulation profile common condition in postmenopausal women
‐ TVS to check for endometrial thickness because at this age, there is lack of estrogen and the
‐ DEXA scan vaginal wall starts to have atrophic changes because
‐ Diagnostic hysteroscopy and endometrial biopsy (if of that.
with undiagnosed vaginal bleeding or increased - For this I will give you local estrogen creams.
thickness) - Also, we need to further assess the womb lining. I will
‐ Urodynamic studies for incontinence refer you to a gynecologist and arrange an ultrasound
to rule out any nasty changes in your womb.
Management - Lifestyle modification. More calcium.
‐ I have organized the investigations for you. At this - Reading material.
stage, since you have dry vagina, I will give you - Review.
estrogen creams. I would advise to have a healthy
lifestyle including exercises 30 minutes a day 5 days a Lichen Sclerosus et atrophicus
week, healthy diet with lots of calcium, pelvic floor
exercises, smoking cessation, advise on safe levels of You are a GP and a 68-year-old female came to your GP
drinking practice complaining of itching of the vulva for 1 year.
‐ Use evening primrose oil for breast tenderness
‐ For social issues: handle accordingly Task
‐ I would like to refer you to a gynecologist who may a. History: chronic itching x 1 year with pain/discomfort
consider starting you on HRT and I would like to b. Diagnosis based on picture given
review you once all the investigations are back and we c. Investigations and manage the case
may need to change some of the management
depending on the results. Biopsy: chronic Inflammatory changes dermatoses -- lichen
sclerosis
r/o MALIGNANCY!
70 
 
History ‐ 95% of patients improve with this treatment. Maintain
‐ Please tell me more about the problem? Is it present good genital hygiene. Avoid using any other creams in
all the time or does it come and go? Does it wake you that area. Try to avoid scratching. If required, you may
up at night? Any bleeding? Discharge? Any problem use emollient to keep the area moist
with passing water like burning sensation, frequency ‐ If not relieved, may use retinoids, tacrolimus, UV
of urination, any problems with the stream (scarring therapy
due to LS may cause problems with urination? ‐ Refer to gynecologist for treatment and followup
Previous infections or surgeries down there? Skin
allergies? Have you ever been diagnosed with DM? ABNORMAL PAP SMEAR
Or prolonged steroid use? When was your LMP? Did
you have symptoms of menopause like flushing, LSIL with HPV
palpitations, irritability, dry vagina? HRT use? For how
long? Any problems with that? Did you have any Case: You are a GP and 24-year-old female came to find out
bleeding or spotting since then? When was your last the result of her pap smear. This showed low-grade
pap smear? Are you sexually active? Stable intraepithelial squamous lesion and HPV infection.
relationship? Any problems during intercourse? Did
you have a mammogram recently? How many Task
children do you have? All NSVD? Complications? a. Explain result to patient
Change in weight? Appetite? Lumps around body? Do b. Management accordingly
you feel tired most of the time? Any FHx of
gynecological cancers or similar conditions? Any Guidelines for Pap Smear Results for Asymptomatic Females
PMHx or surgical conditions? SADMA? Results Action
Picture: white shiny plaques on both vulva with lace- Negative Repeat x 2 years
like patterns w/ or w/o bleeding; may bleed when LSIL Repeat in 1 year
scratched HSIL Colposcopy and biopsy
Unsatisfactory Repeat in 6-12 weeks
Investigations Glandular cells Colposcopy and biopsy
‐ BSL, pap smear, swab if with discharge
‐ Multiple punch biopsy of lesion ¾ LSIL
o Mild dyskariosis/dysplasia or HPV infection
Differential diagnosis o Repeat in 12 mos
‐ lichen sclerosis ƒ If normal Æ repeat in 12 mos. Æ
‐ candidiasis if normal then every 2 years
‐ atrophic vaginitis ƒ If (+) LSIL Æ colposcopy and
‐ vulvar Cancer biopsy
‐ psoriasis ƒ If colposcopy shows LSIL Æ
‐ diabetes ablation (laser, cryotherapy,
‐ paget disease diathermy or surgical excision)
‐ leukoplakia ¾ HSIL
‐ vulvovaginitis ƒ Moderate to severe dyskariosis
‐ trauma ƒ Do Colposcopy and biopsy
‐ Eczema ƒ If colposcopy shows intracellular
(LSIL) lesions Æ ablation (laser,
Management cryotherapy, diathermy or surgical
‐ You have a condition called lichen sclerosis (genital excision)
pruritus + genital soreness+ white wrinkled plaques). It ƒ If colposcopy is positive for
is a chronic inflammatory skin condition. The exact invasive lesion Æ do cone biopsy
cause is unknown, but there is a genetic pattern and it o NO cone biopsy in pregnancy
is linked to certain immune-mediated conditions, e.g. ƒ Complications: bleeding, cervical
Autoimmune thyroid disease, vitiligo, psoriasis, incompetence, cervical stenosis
pernicious anemia, alopecia ƒ May affect further pregnancy:
‐ Please don't worry. This is not an infection and this is premature labor or premature
not cancer. It is not contagious. It usually presents as rupture of membranes
itching, vulvar pain, bleeding with scratching, o In pregnancy:
sometimes blister formation. ƒ If LSIL Æ wait and can do
‐ It is important to treat the condition to prevent ablation after pregnancy
scarring. 5% of these patients may develop cancers ƒ If HSIL:
within the scar. • <20 weeks: offer
‐ Is it because of my menopause? (up to now there has termination and
been no association proven between lack of estrogen aggressive cancer
and appearance of the condition) treatment
‐ Treatment with steroids (clobetasol propionate) -- • >20 weeks: up to
apply 2x a day for x 1 month, then once every night x mother to decide
1 month then 2x weekly x 3 months then once weekly • >35 weeks: continue
until asymptomatic then PRN pregnancy and do
‐ Inform about risk of steroid therapy: thinning of skin, cesarean section and
redness, fungal infections aggressive cancer
treatment
‐ Any active problems with cervix/abnormal pap smear
in pregnancy is a contraindication to vaginal delivery
71 
 
HPV ‐ We are planning to have a baby, can I fall pregnant?
‐ Around 200 types There is a 10% that the baby might acquire the
‐ 40 are found within anogenital area infection during labor only. It usually goes into the
‐ Spread by sexual contact and skin-to-skin contact baby’s throat (respiratory papillomatosis) causing
‐ Type 6 and 11: low-risk HPV Æ responsible for 90% warts. The baby may or may not be able to get rid of
of genital warts; not related in any way to cancers the infection on its own, but we can give certain
‐ Types 16 and 18: high-risk HPV Æ responsible for medications to help him. It is important to practice safe
70% of cervical cancers all over the world sex from now onwards.
‐ Causes microabrasions within cervical epithelium ‐ Review. Reading material.
‐ Are extremely common within the first 10 years of
sexual life but majority are transient Pap Smear (CIN I)
‐ Body is able to get rid of the virus on its own but might
take up to 12 months to clear the infection Case: Katharin aged 25 years presents to your surgery for result
‐ Also known as “common cold” of sexual activity of her Pap smear which you did last week. The result shows
changes consistent with CIN 1. Her last Pap test was two years
Counseling ago and that was normal. She is otherwise well and had no
‐ I have the results of the test with me. May I ask a few previous medical or any surgical problems. Katharine lives by
relevant questions? herself and works in a local bar. She smokes on average 10-15
‐ When did you have your last pap smear? What was cigarettes per day and drink socially.
the result? I understand you are sexually active, are
you in a stable relationship at the moment? How many Task
partners have you had previously? Did you always a. Explain result of pap smear
practice safe sex with the use of condoms? Have you b. Further relevant history
or your partner ever been diagnosed with a STI c. Management
(warts)? At the moment, do you have any symptoms,
vaginal discharge, bleeding, or itching? Any pain History
during intercourse? Any chance you might be - Multiple partners? Smoker? What age of coitarche?
pregnant now? Have you ever been pregnant before? Practicing safe sex? Promiscuity? What is your work?
Any miscarriages? When was your LMP? Cycles Low socioeconomic status? FHx of cancers?
regular?
‐ How is your general health? SADMA? Do you have a Management
family history of gynecological cancers or breast - Offer HPV vaccination
cancer? - Repeat after 1 year (or 6 months if age >30 and pap
smear >2 years ago)
Counseling - Counsel against risk factors and safe sex
‐ As you know, pap smear is a screening test for early
asymptomatic cervical cancer. We usually detect for Abnormal Pap smear with Actinomyces
the presence of abnormal cells in the cervix. At the
moment, your results showed that there are some Case: You are a GP and a 38-year-old female comes in with
cells that look different from normal. We call it LSIL. pap smear showing abnormal cells + Actinomyces. She has
Basically it means that there are minor changes within IUCD for 5 years.
the lining of the cervix, which could be because of the
presence of a coexisting infection with HPV. This virus Task
induces temporary changes in the lining of the cervix. a. History (periods regular, IUCD checks monthly,
What is important is that LSIL has a very low but 2children, NSVD, STI -, DM + Grandmother,
definite risk of transforming into cancer. We need to b. Physical examination (can see string of pap smear
repeat the test within 12 months time. There are two BSL 5.5)
possibilities: If pap smear is normal, we will repeat it c. Diagnosis and Management
again in 1 year time and if still normal then go back to
2-yearly regime. The other possibility is persistent History
LSIL or HSIL. If this happens, I will have to refer you ‐ I know you have come to see me because you want to
to a specialist for colposcopy and biopsy. It is a discuss your pap smear result.
process where we introduce a small tube with a ‐ Prior to our discussion, can I ask a few symptoms?
camera into the cervix to look at the lining. If there is a How are you feeling? Have you noticed any low
suspicious lesion, then a piece of tissue will be taken abdominal pain or discomfort? Any unusual discharge
out. If not, acetic acid will be applied and a suspicious or bleeding?
area will turn white and a sample will be taken. ‐ Periods:
‐ Regarding HPV infection, the body will be able to clear ‐ Are you in a stable relationship? Have you ever been
off the infection in majority of cases. It is very difficult diagnosed with STD or PID? Is it your first IUCD?
to find out how and when you got this infection What type of IUCD do you have? Have you ever been
because it can happen even in stable relationships. It pregnant? How many children have you had? Have
is important for you to be vaccinated with gardasil to you ever had an abnormal pap smear in the past
protect you from the other 3 subtypes of HPV. If you (No)? When was it? What was it? What was done for
like, we can check you for other STDs. that? When was your last pap smear apart from this
one? How’s your general health? SADMA?
‐ FHx
72 
 
Physical examination - Gardasil vaccine does not protect against other STDs.
‐ General appearance It doesn’t encourage girls to start sexual life earlier.
‐ Vital signs The main purpose of the HP V vaccine is to protect
‐ Abdomen: tenderness and masses them against cervical cancer and genital warts.
‐ Pelvic: inspection/speculum: appearance of cervix, However, it doesn’t give 100% protection. All girls
any abnormal discharge, thread of IUCD? PV: size of need to be screened for cervical pathology using pap
uterus, adnexal masses/tenderness, cervical smear from the age of 18 or 2 years after they
excitation/CMT become sexually active (whichever comes later).
‐ Urine dipstick and BSL - Can be given to boys but not included in the
immunization program.
Diagnosis and Management - Pregnant women? No, but you can give them after
‐ Your pap smear result showed abnormal cells and labor even while breastfeeding.
actinomyces. Actinomyces is a gram positive bacteria - It is no longer beneficial after the age of 27.
and is relatively common to find smears positive for

Actinomyces in women who use IUCDs. I want to refer


you to a gynecologist for further assessment and
management. Usually, with symptomatic Actinomyces,
IUCD should be removed, threads cut, and IUCD sent
for microscopy and culture. If it is positive, prolonged
antibiotic treatment with penicillin for 6 weeks is done.
After treatment, pap smear should be repeated in 6-12
weeks because it might be due to the coexisting
infection with the bug.

Gardasil Vaccine

Case: You are a GP and your next patient is a 45-year-old Mr.


Walker wants to know about Gardasil vaccination. His 15 year
old daughter will receive vaccine in school and he is worried that
it will encourage early sexual life.

Task
a. Respond to patient inquiry

- Hello Mr. Walker. I understand you have come to see


me to discuss Gardasil vaccine. How much do you
know about this vaccine? Have you ever heard about
HPV infection?
- Gardasil vaccine was designed to prevent HPV
infection and it doesn’t promote early sexual life.
Gardasil is effective against 4 types of HPV. There are
40 types of HPV that affect the genital tract. This
vaccine is against types 16 and 18 causative agents in
70-80% of all cervical cancer and types 6 and 11,
which are associated with 90% of genital warts. HPV
infection is transmitted by sexual intercourse. That is
why this vaccine is given to young girls (9-26) since
most of them hasn’t started sexual life and haven’t
been infected and thus will benefit the most. However,
even sexually active girls can benefit from gardasil
vaccine. Majority of them will not yet be infected, or
may be infected by 1 or 2 types and get protection
against others.
- Gardasil vaccine is part of the school immunization
program. It is free and given within 6 months. It is
administrated by intramuscular injection usually in the
shoulder. The only absolute contraindication to HPV
vaccination is severe allergic reaction (anaphylaxis)
following a previous dose of the vaccine.
- Gardasil contains virus-like particles which are non-
infectious and do not have any cancer-causing
potential. This vaccine is generally safe and well-
tolerated.
- Possible side effects: Injection site pain, swelling, and
redness.

You might also like